Download as pdf or txt
Download as pdf or txt
You are on page 1of 128

CAP II Paper 1: Advanced Accounting

CHARTERED ACCOUNTANCY PROFESSIONAL II


(CAP-II)

Suggested Answer
June 2018

The Institute of Chartered Accountants of Nepal

The Institute of Chartered Accountants of Nepal 1


Suggested Answer - June 2018

Paper 1: Advanced Accounting

The Institute of Chartered Accountants of Nepal 2


CAP II Paper 1: Advanced Accounting

Maximum Marks - 100


Total No. of Questions - 6 Total No. of Printed Pages -14
Time Allowed - 3 Hours
Marks
Attempt all questions. Working notes should form part of the answer.
1. A and B were carrying on business sharing profits and losses equally.
The firm's Balance Sheet as at 31.03.2074 was:
Liabilities Rs. Assets Rs.
Capital Accounts: Plant 1,60,000
A 1,50,000
B 1,30,000 2,80,000
Sundry Creditors 80,000 Building 48,000
Bank Overdraft 45,000 Debtors 75,000
Stock 70,000
Joint Life Policy 6,000
Profit & Loss A/c 30,000
Drawing Account: 16,000
A 9,000
B 7,000
Total 4,05,000 Total 4,05,000
The operations of the business were carried on till 30.09.2074. A and B both withdrew in
equal amount half the amount of profit made during the current period of six months
after charging depreciation at 10% per annum on plant and after writing off 5% on
building . During the current period of six months, creditors were reduced by Rs. 20,000
and bank overdraft by Rs. 5,000.
The life policy was surrendered for Rs. 6,000 before 30th Poush, 2074. Stock was valued
at Rs. 84,000 and debtors at Rs. 68,000 on 30th Poush, 2074. The other items remained
the same as 31.03.2074.
On 30.09.2074, the firm sold its business to AB Ltd. The value of goodwill was estimated
at Rs. 1,30,000 and the remaining assets were valued on the basis of the balance sheet as
on 30.09.2074.
AB Ltd. paid the purchase consideration in equity shares of Rs. 10 each.
You are required to prepare; 20
a) Balance Sheet of the firm as at 30.09.2074,
b) Realization account,
c) Partners' Capital Accounts showing the final settlement between them.
Answer:
a) Balance Sheet of the firm as at 30.09.2074
Liabilities Rs. Assets Rs.

The Institute of Chartered Accountants of Nepal 3


Suggested Answer - June 2018

Capital Account : Plant :


A's Capital 1,33,800 Opening Balance 1,60,000
B's Capital 1,15,800 Less: Depreciation @ 10% 8,000 1,52,000
Creditors 60,000 Building :
Bank Overdraft 40,000 Opening Balance 48,000
Less: Written-off @ 5% 2,400 45,600
Debtors 68,000
Stock 84,000

Total 3,49,600 Total 3,49,600

b) Realisation Account
Dr.
Cr.
Particular Amount Particular Amount
To Sundry Assets: By Creditors 60,000
Plant 1,52,000 By Bank Overdraft 40,000
Building 45,600
Stock 84,000 By AB Ltd A/c 3,79,600
Debtors 68,000 (working note 2 )
To Profit:
A's Capital A/c 65,000
B's Capital A/c 65,000
------------- ---------------
4,79,600 4,79,600
c) Partner's Capital Accounts
Date Particular A (Rs.) B (Rs.) Date Particular A (Rs.) B (Rs.)
01.04.74 To Profit & 01.04.74 By balance b/d 1,50,000 1,30,000
Loss A/c 15,000 15,000
01.04.74 To Drawing A/c 9,000 7,000 30.09.74 By Profit 15,600 15600
30.09.74 To Drawing A/c 7,800 7,800 (W.N.1)
(W. N.1)
30.09.74 To Balance c/d 1,33,800 1,15,800 ---------- ----------
- --
Total 1,65,600 1,45,600 Total 1,65,000 1,45,600
30.09.74 By Balance b/d 1,33,800 1,15800

30.09.74 To shares in AB 1,98,800 1,80,800 30.09.74 By Realisation 65,000 65,000


Ltd. A/c (Profit)
1,98,800 1,80,000 1,98,800 1,80,000

Working Notes:
(1) Ascertainment of profit for the period of 6 Months ended 30.09.2074
Amount (Rs)
Closing Assets :

The Institute of Chartered Accountants of Nepal 4


CAP II Paper 1: Advanced Accounting

Stock 84,000
Debtors 68,000
Plants less Depreciation 1,52,000
Building Less Written off 45,600
Total 3,49,600
Less: Closing Liabilities :
Creditors 60,000
Bank Overdraft 40,000 1,00,000
Closing Net Assets 2,49,600
Less: Opening adjusted Capitals
A(Rs. 1,50,000 - Rs. 15,000 - Rs. 9,000) 1,26,000
B(Rs. 1,30,000 - Rs. 15,000 - Rs. 7,000) 1,08,000 2,34,000
Profit Net of drawings 15,600
Actual Profit for six months before drawings (half of profit )=15,600x2 31,200
Combined Drawing during six months (half of profit) 15,600
(2) Ascertainment of purchase consideration
Rs.
Closing Net Assets (As above) 2,49,600
Add: Goodwill 1,30,000
Total Purchase consideration 3,79,600

2.
a) The following is the Balance Sheet of a concern on 31st Ashadh, 2073:
Liabilities Rs. Assets Rs.
Capital 10,00,000 Fixed Assets 4,00,000
Creditors (Trade) 1,40,000 Stock 3,00,000
Profit & Loss A\c 60,000 Debtors 1,50,000
Cash & Bank 3,50,000
12,00,000 12,00,000
The management estimates the purchases and sales for the year ended 31st Ashadh,
2074 as under:
Particulars Upto 32.2.2074 (Rs.) Ashadh 2074 (Rs.)
Purchases 14,10,000 1,10,000
Sales 19,20,000 2,00,000
It was decided to invest Rs. 1,00,000 in purchases of fixed assets, which are
depreciated @ 10% on cost.
The time lag for payment to Trade Creditors for purchase and receipt from Sales is
one month. The business earns a gross profit of 30% on turnover. The entire sales and
purchases are taken on credit basis. The expenses against gross profit amount to 10%
of the turnover. The amount of depreciation is not included in these expenses.
Draft a Balance Sheet as of 31st Ashadh, 2074 assuming that creditors are all Trade
Creditors for purchases and debtors for sales and there is no other item of current
assets and liabilities apart from stock and cash and bank balances. 10

The Institute of Chartered Accountants of Nepal 5


Suggested Answer - June 2018

b) From the following Balance Sheets of Mr. Ram, prepare a Cash Flow Statement as per
NAS 3 for the year ended 31.03.2074:

Balance Sheet of Mr. Ram


As on 1.4.2073 As on 31.03.2074
Liabilities:
Ram‘s Capital Account 5,00,000 6,12,000
Sundry creditors 1,60,000 1,76,000
Mrs. Ram‘s loan 1,00,000 -
Long term loan from bank 1,60,000 2,00,000
9,20,000 9,88,000

Assets:
Land 3,00,000 4,40,000
Plant & Machinery 3,20,000 2,20,000
Stock 1,40,000 1,00,000
Debtors 1,20,000 2,00,000
Cash 40,000 28,000
9,20,000 9,88,000
Additional information:
A machine costing Rs. 40,000 (accumulated depreciation there on Rs. 12,000) was
sold for Rs. 20,000. The provision for depreciation on 1.4.2073 was Rs. 1,00,000 and
on 31.03.2074 was Rs. 1,60,000. The net profit for the year ended on 31.03.2074 was
Rs. 1,80,000. 10

Answer:
a)
Projected Balance Sheet of
As on 31st Ashadh, 2074
Liabilities Rs Assets Rs
Capital 10,00,000 Fixed Assets 4,00,000
Profit & Loss Additions 1,00,000
Account as on
1st Shrawan, 60,000 5,00,000
2073
Add: Profit for 3,74,000 4,34,000 Less: Depreciation (50,000) 4,50,000
the year
Creditors 1,10,000 Stock in trade 3,36,000
(Trade)
Sundry Debtors 2,00,000
Cash & Bank Balances 5,58,000
15,44,000 15,44,000
Working Notes:
Projected Trading and Profit and Loss Account

The Institute of Chartered Accountants of Nepal 6


CAP II Paper 1: Advanced Accounting

For the year ended 31st Ashadh, 2074


Particulars Rs Particulars Rs
To Opening Stock 3,00,000 By Sales 21,20,000
To Purchases 15,20,000 By Closing Stock 3,36,000
(balancing figure)
To Gross Profit c\d (30% on 6,36,000
sales)
24,56,000 24,56,000
To Sundry Expenses (10% on 2,12,000 By Gross Profit b\d 6,36,000
sales)
To Depreciation 50,000
To Net Profit 3,74,000
6,36,000 6,36,000
Cash and Bank Account
1st Shrawan, 2073 to 31st Ashadh, 2074
Rs. Rs
.
To Balance b/d 3,50,000 By Sundry Creditors 15,50,000
To Sundry Debtors 20,70,000 (Rs1,40,000+Rs 14,10,000)
(Rs 1,50,000 + Rs By Expenses 2,12,000
19,20,000)
By Fixed Assets 1,00,000
By Balance c/d 5,58,000
24,20,000 24,20,000

b) Cash Flow Statement of Mr. Ram for the year ended 31.03.2074
Rs.
(i)Cash flow from operation activities
Net Profit (given) 1,80,000
Adjustment for Depreciation on Plant & Machinery 72,000
(W.N.2) 8,000 80,000
Loss on sale of Machinery (W.N.1) 2,60,000
Operating profit before working capital changes 40,000
Decrease in Stock (80,000)
Increase in Debtors 16,000 ( 24,000)
Increase in Creditors 2,36,000
Net cash from operating activities
(ii) Cash flow from investing activities: 20,000
Sale of Machinery (1,40,000)
Purchase of Land(4,40,000-3,00,000) (1,20,000)
Net cash used in investing activities
(iii) Cash flow from financing activities: (1,00,000)
Payment of Mrs. Ram‘s Loan (68,000)
Drawings(W.N. 3) 40,000
Loan from bank (1,28,000)

The Institute of Chartered Accountants of Nepal 7


Suggested Answer - June 2018

Net cash used in financing activities (12,000)


Net decrease in cash 40,000
Cash balance as on 1.4.2073 28,000
Cash balance as on 31.03.2074

Working Notes:
1 Plant & Machinery A/c
Rs. Rs.
To Balance b/d 4,20,000 By Bank-sales 20,000
( 3,20,000+1,00,000) By Provision for
depreciation A/c 12,000
By Profit & Loss A/c-loss
on sale(40,000-20,000- 8,000
12,000) 3,80,000
4,20,000 By balance c/d 4,20,000
(2,20,000+1,60,000)
2 Provision for depreciation on Plant & Machinery A/c
Rs. Rs.
To Plant & Machinery A/c 12,000 By Balance b/d 1,00,000
To Balance c/d 1,60,000 By Profit & Loss 72,000
1,72,000 A/c(Bal.fig) 1,72,000

3.Mr. Ram‘s Drawing


Rs.
Opening Capital 5,00,000
Add: Net profit 1,80,000
6,80,000
Less Closing Capital (6,12,000)
Drawings 68,000
3.
a) Pashupati Stores Pvt Ltd with its head office at Kathmandu, invoiced goods to its
branch at Gajuri at 20% less than the list price which is cost plus 100%, with
instruction that cash sales were to be made at invoice price and credit sales at list
price.
From the following particulars available from the branch, prepare Branch Stock
Account, Branch Adjustment A/c, Branch Profit and Loss A/c and Branch Debtors
A/c for the year ending 31st Chaitra, 2074. 10
Particulars Rs. Rs.
Stock on 1st Baishakh, 2074 ( invoice price) 6,000
Debtors on 1st Baishakh, 2074 5,000
Goods received from Head Office (invoice price) 66,000
Sales
Cash 23,000

The Institute of Chartered Accountants of Nepal 8


CAP II Paper 1: Advanced Accounting

Credit 50,000 73,000


Cash received from debtors 42,817
Expenses at branch 8,683
Debtors on 31st Chaitra, 2074 12,183
Stock on 31st Chaitra, 2074 (invoice price) 8,800
Remittance to Head Office 60,000

b) Following balances relating to loan and advances are extracted from records of a ‗A‘
class Commercial Bank.
Amount in lakhs
Categories of Loan Insured Amount Other Amount Total Amount
Pass 3,335 161,528 164,863
Restructured/Rescheduled 775 4,464 5,239
Substandard 138 706 844
Doubtful 217 1,927 2,144
Loss 662 3,539 4,201
Total 5,127 172,164 177,291

While scrutinizing the records of the bank, it was found that the bank has provided a
credit of Rs. 5,560 lakhs in excess of SOL. The bank has categorized this loan as
Pass. Find out the amount of loan loss provisions as per NRB Directives. 5
Answer:
a) In the Books of Pashupati Stores Pvt Ltd:
Dr Branch Stock Account Cr
Particulars Amount Particulars Amount
To Balance b/d 6,000 By Bank ( Cash Sales) 23,000
To Goods sent to Branch 66,000 By Branch Debtor 50,000
To Branch Adjustment( W.N-1) 10,000 By Shortage in stock (W. N-2) 200
By Balance c/d 8,800
Total 82,000 Total 82,000

In the Books of Pashupati Stores Pvt Ltd


Dr Branch Adjustment Account Cr
Particulars Amount Particulars Amount
To Shortage in stock (W.N-2) 75 By Branch Stock(W.N-1) 10,000
To Stock Reserve (W.N-5) 3,300 By Stock Reserve (W.N-3) 2,250
To Gross Profit & Loss 33,625 By Goods sent to Branch (W.N-4) 24,750
Total 37,000 Total 37,000

The Institute of Chartered Accountants of Nepal 9


Suggested Answer - June 2018

In the Books of Pashupati Stores Pvt Ltd


Dr Branch Profit and Loss Account Cr
Particulars Amount Particulars Amount
To Branch Expenses 8,683 By Gross Profit 33,625
To Shortage in sales (cost) 125
To General Profit and Loss 24,817
Total 33,625 Total 33,625

In the Books of Pashupati Stores Pvt Ltd


Dr Branch Debtors Account Cr
Particulars Amount Particulars Amount
To Balance b/d 5,000 By Bank 42,817
To Branch Stock 50,000 By Balance c/d 12,183
Total 55,000 Total 55,000
Working Notes
1) Let the Cost price= X
List price = 100% of cost price
=100 +100= Rs.200
Invoice price = 20% less than List price
=200-(20% of 200)
=Rs.160
Cash sales are made at invoice price i.e.Rs.160 and credit sales are made at list
price i.e.Rs.200
Excess amount charged on credit sales= Rs.200-Rs.160= Rs.40 i.e.
40/200*100%= 20 % of list price.
Good sold on credit= Rs.50, 000
Amount charged in excess of the invoice price = 20% of list price
= 20% of Rs.50, 000
= Rs.10, 000
2) If the cost =Rs.100 then invoice price = Rs.160
Therefore loading on invoice price = 60/160*100%=37.5%
Loading on Shortage =37.5% of Rs.200= Rs.75
= Rs.200-Rs.75= Rs.125
3) Loading of Opening stock
=37.5% of Rs.6, 000=Rs.2, 250
4) Loading on Goods sent to Branch
=37.5% of Rs.66, 000=Rs.24, 750
5) Loading on closing stock
=37.5% of Rs.8, 800=Rs.3, 300
b)
Computation of Loan Provisions Amount
(Amount in Lakhs)

The Institute of Chartered Accountants of Nepal 10


CAP II Paper 1: Advanced Accounting

Categories Insured Rate Provision Other RateProvision Total


of Loan Amount Amount Amount Provision
Amount
Pass 3,335 0.25 8 161,528 1% 1615 1624
Restructured 775 3.13 24 4,464 12.5% 558 582
/Rescheduled
Substandard 138 6.25 9 706 25% 177 185
Doubtful 217 12.5 27 1,927 50% 964 991
Loss 662 25 166 3,539 100% 3539 3705
Total 5,127 234 172,164 6852 7086
Additional 100% Provision for Credit Provided in Excess of SOL (B) =5560
Total Loan Loss Provisions (A+B) = 12646
Working Note:
Provisioning Rate for Insured Loan i.e. = 25% of prescribed rate
1×0.25=0.25%
12.5×0.25=3.125
20×0.25=6.25%
50×0.25=12.5%
100×0.25=25%

4.
a) Retirement Benefit Plan of ABC Bank Ltd. purchased preference shares of XYZ
Bank Ltd. face value of Rs. 1,000,000 at Rs. 1,037,000 on 1st Shrawan, 2069. The
coupon rate of the preference shares is 12% and maturity date of the shares is 31 st
Ashadh, 2074. You are required to prepare the investment account in the books of
retirement benefit plan up to the year ended 31st Ashadh, 2074. The effective interest
rate of the investment is 11%. 10
b) From the following information relating to M/s Genuine General Insurance find out the
revenue profit for the period ended 31.12.2074. 5
in Lakhs
Gross premium (unrealized Rs. 120) 8,000
Claim paid (including cheque issued but not collected Rs. 95) 1,680
Other administrative expenses 1,455
Unexpired risk reserve (opening) 2,450
Unexpired risk reserve (closing) 2,760
Outstanding claim liability (opening) 1,440
Outstanding claim liability (closing) 1,230
As per company's strategy 35% business is covered under reinsurance treaty,
which provides average 6.5% commission.

a)
Retirement Benefit Plan of ABC Bank Ltd.
Investment Account
Date Particulars Amount Date Particulars Amount

The Institute of Chartered Accountants of Nepal 11


Suggested Answer - June 2018

1,04.2069 To Bank A/c 10,37,000 31.03.2070 By Bank A/c 5,930


31.03.2070 By Balance C/d 10,31,070

Total 10,37,000 Total 10,37,000


1,04.2070 To Balance B/d 10,31,070 31.03.2071 By Bank A/c 6,582
31.03.2071 By Balance C/d 10,24,488

Total 10,31,070 Total 10,31,070


1,04.2071 To Balance B/d 10,24,488 31.03.2072 By Bank A/c 7,306
31.03.2072 By Balance C/d 10,17,181

Total 10,24,488 Total 10,24,488


1,04.2072 To Balance B/d 10,17,181 31.03.2073 By Bank A/c 8,110
31.03.2073 By Balance C/d 10,09,071

Total 10,17,181 Total 10,17,181


1,04.2073 To Balance B/d 10,09,071 31.03.2074 By Bank A/c 9,071
31.03.2074 By Bank A/c 10,00,000

Total 10,09,071 Total 10,09,071


Amortization Schedule
Year (A) Beginning (B) Dividend (C) Reported (D) (E) End-of-
of period and maturity dividend Amortization period
amortized cost cash inflow income of Premium amortized
[=(A)×11%] [=(C) – (B)] cost[=(A)+(D)]
2069- 10,37,000 1,20,000 1,14,070 5,930 10,31,070
70
2070- 10,31,070 1,20,000 1,13,418 6,582 10,24,488
71
2071- 10,24,488 1,20,000 1,12,694 7,306 10,17,181
72
2072- 10,17,181 1,20,000 1,11,890 8,110 10,09,071
73
2073- 10,09,071 11,20,000 1,10,929 9,071 10,00,000
74
b)

Statement of calculation of Revenue Profit


For the period ended 31.12.2074
in Lakhs
Gross premium (less unrealized Rs. 120) 7,880.00

The Institute of Chartered Accountants of Nepal 12


CAP II Paper 1: Advanced Accounting

Less: reinsurance portion (35%) (2,758.00)


Reinsurance commission (6.5% of Rs. 2,758) 179.27
Claim paid (1,680.00)
Other administrative expenses (1,455.00)
Unexpired risk reserve (opening) 2,450.00
Unexpired risk reserve (closing) (2,760.00)
Outstanding claim liability (opening) 1,440.00
Outstanding claim liability (closing) (1,230.00)
Revenue Profit for the period 2,066.27

Alternate Solution
Statement of calculation of Revenue Profit
For the period ended 31.12.2074
in Lakhs
Gross premium (less unrealized Rs. 120) 7,880.00
Less: reinsurance portion (35%) (2,758.00)
Reinsurance commission (6.5% of Rs. 2,758) 179.27
Claim paid (1,680.00)
Other administrative expenses (1,455.00)
Unexpired risk reserve (opening) 2,450.00
Unexpired risk reserve (closing) (2,760.00)
Outstanding claim liability (opening) 115% 1,656.00
Outstanding claim liability (closing) 115% (1,414.50)
Revenue Profit for the period 2,097.77

5.
a) The following information are related with Purple Nepal Ltd.
i) Goods of Rs. 60,000 were sold on 20-3-2074 but at the request of the buyer these
were delivered on 10-4-2074.
ii) On 15-3-2074 goods of Rs. 1,50,000 were sent on consignment basis of which
20% of the goods unsold are lying with the consignee as on 31-3-2074.
iii) Rs. 1,20,000 worth of goods were sold on approval basis on 1-12-2073. The
period of approval was 3 months after which they were considered sold. Buyer
sent approval for 75% goods up to 31-1-2074 and no approval or disapproval
received for the remaining goods till 31-3-2074.
iv) Apart from the above, the company has made cash sales of Rs. 7,80,000 (gross).
Trade discount of 5% was allowed on the cash sales.
You are required to advise the accountant of Purple Nepal Ltd. with valid reasons, the
amount to be recognized as revenue in above cases in the context of NAS -18 and
also determine the total revenue to be recognized for the year ending 31-3-2074. 5
b) M/s Dalima Ltd. is in a dispute with the competitor company. The dispute is
regarding the alleged infringement of copyrights. The competitor has filed a suit in
the court seeking damages of Rs. 325 lakhs.

The Institute of Chartered Accountants of Nepal 13


Suggested Answer - June 2018

Directors are of the view that the claim can be successfully resisted by the company.
How the matter be dealt in the financial statements of the company in the light of
NAS 37. Explain in brief giving reasons for your answer. 5
th
c) Gorkha Company Ltd. imported raw materials worth USD 9,000 on 24 Jestha, 2074,
when the exchange rate was Rs.104 per USD. The transaction was recorded in the
books at the above mentioned rate. The payment of the transaction was made on 10th
Shrawan, 2074, when the exchange rate was Rs.108 per USD. At the year end 31st
Ashadh, 2074, the rate of exchange was Rs.109 per USD.
The Account Officer of the company passed an entry on 31st Ashadh, 2074 adjusting
the cost of the raw material consumed for the difference between Rs.108 and Rs.104
per USD. Discuss whether this treatment is justified as per the provision of NAS-21. 5
Answer:
a) As per NAS 18 "Revenue" is a transaction involving the sale of goods, performance
should be regarded as being achieved when the following conditions are fulfilled:
(a) The seller of goods has transferred to the buyer the property in the goods for a
price or all significant risks and rewards of ownership have been transferred to the
buyer and seller retains no effective control of the goods transferred to a degree
usually associated with ownership : and
(b) no significant uncertainty exists regarding the amount of the consideration that will
be derived from the sales of the goods.
In case (i):
The sale is complete but delivery has been postponed at buyer's request. Purple Nepal
Ltd. should recognize the entire sale of Rs. 60,000 for the year ended 31 st Ashadh,
2074.
In case (ii):
20% goods lying unsold with consignee should be treated as closing inventory and
sales should be recognized for Rs. 1,20,000 (80% of Rs. 1.50,000). In case of
consignment sale revenue should not be recognized until the goods are sold to a third
party.
In case (iii):
In case of goods sold on approval basis, revenue should not be recognized until the
goods have been formally accepted by the buyer or the buyer has done an act
adopting the transaction or the time period for rejection has elapsed or where no time
has been fixed, a reasonable time has elapsed. Therefore in case (iii) revenue should
be recognized for the total sales amounting Rs.1,20,000 as the time period for
rejecting the goods had expired.
In case (iv):
Trade discounts given should be deducted in determining revenue. Thus Rs. 39,000
should be deducted from the amount of turnover of Rs.7,80,000 for the purpose of
recognition of revenue. Thus, revenue should be Rs. 7,41,000.
Thus total revenue amounting Rs. 10,41,000 (60,000+1,20,000+1,20,000+7,41,000)
will be recognized for the year ended 31st Ashadh, 2074 in the books of Purple Nepal
Ltd.
b) As per NAS 37 "Provisions, Contingent liabilities and Contingent assets" a provision

The Institute of Chartered Accountants of Nepal 14


CAP II Paper 1: Advanced Accounting

should be recognized when


a) an entity has a present obligation as a result of a past event.
b) it is probable that the outflow of resources embodying economic benefits will be
required to settle the obligation and
c) a reliable estimate can be made of the amount of obligation.
if these conditions are not met, no provision should be recognized.
In the given situation, since the directors of the company are of the opinion that the
claim can be successfully resisted by the company, therefore there will be no outflow
of resources. Hence no provision is required. The company can disclose the same as
contingent liability by way of following note.
Litigation is in the process against the company relating to dispute with the
competitor who alleged that the company has infringed copyrights and seeking
damages of Rs. 325 lakhs. However the director are of the opinion that the claim can
be successfully resisted by the company.

c) As per NAS-21, ―The effects of changes in foreign exchange rates‖,


(i) Initial recognition of a foreign currency transaction is done in the functional
currency, by applying the spot exchange rate between the functional currency and
foreign currency at the date of the transaction.
(ii) At the end of each reporting period, foreign currency monetary items shall be
translated using the closing rate.
(iii)Exchange difference arising on settlement of monetary items or on translating
monetary items at rates different from those at which they were translated on initial
recognition during the period or in previous financial statement shall be recognized
in profit and loss in the period in which they arise.
In the given case, at the date of transaction the raw material purchased and its
creditors will be recorded at USD 9,000×Rs.104= Rs. 936, 000.
At Balance Sheet date such transaction is reported at closing rate of exchange,
hence it will be valued at the closing rate i.e. Rs.109 per USD (USD
9,000×Rs.109=Rs.981, 000)
The difference of exchange rate between the closing date and transaction date is
Rs.5 per USD (i.e. Rs. 109-Rs.104). The difference of Rs.45,000 (USD 9,000×5)
will be shown as an exchange loss in the profit and loss account for the year ended
31st Ashadh, 2074 and will not be adjusted against the cost of raw materials.
At the settlement date, the company would recognize or provide in the profit and
loss account an exchange gain of Rs. 9,000 (i.e. at the rate Rs. 1 per USD, the
difference of exchange rate between the balance sheet date and the date of
settlement, i.e. Rs.109 and Rs.108 per USD).
Hence the accounting treatment adopted by the Account officer is not as per NAS
21.
6. Write short notes on: (5×3=15)
a) Life Insurance Fund
b) PEARLS System of Monitoring the Cooperative.
c) ‗Other Comprehensive Incomes‘ as per NFRS.

The Institute of Chartered Accountants of Nepal 15


Suggested Answer - June 2018

d) When can a company change its accounting policy?


e) Calculation of Profit from the Notional profit.
Answer:
a) Life Insurance Fund
As per section 22 of the Insurance Act, 2049, every Insurer shall maintain a reserve fund
as specified by the Insurance Board for the liability relating to its Insurance Business
within the state of Nepal. Further, section 21 of the Act states that an Insurer shall
maintain separate fund for each category of Insurance Business and amount to be
received from each Insurance Business shall be deposited in the concerned fund. The
fund maintained for one category of Insurance Business shall not be utilized to bear the
liabilities relating to other category of Insurance Business.
The following amount shall be transferred to the Life Insurance Fund:
 An amount not less than the total liability as specified by the Actuary on the basis of the
Insurance Policies published by the Insurer of the Life Insurance Business within the
state of Nepal.
 In case of fiscal year where no Actuary Valuation is done, the whole surplus amount of
Revenue Account; and
 Amount specified by Insurance Board for Solvency Margin from time to time.
The amount kept under Life Insurance Fund shall be distributed as per Actuary
Valuation Report following the directives of the Insurance Board on this regard.
b) PEARLS System of Monitoring the Cooperative
PEARLS is defined as a set of financial ratios to monitor the financial stability of the
credit unions within World Council of Credit Union's developing movement projects.
These ratios provide credit unions, project staff, national federations and regulators
with essential tools for monitoring, planning, standardizing, ranking and facilitating
supervisory control in credit unions. Each letter in the word PEARLS measures the
key areas of credit union operations: Protection, Effective financial structure, Asset
quality, Rate of return and Costs and Liquidity and Sign of growth. PEARLS system
is designed as a management tool that goes beyond the simple identification of
problems. It helps managers find meaningful solutions to serious institutional
deficiencies. Use of the system permits managers quickly and accurately pinpoint
troubled areas, and to make the necessary adjustments before problems become
serious. In essence, PEARLS is an "early warning system" that generates invaluable
management information.
c) ‗Other Comprehensive Incomes‘ as per NFRS
Other comprehensive income comprises items of income and expenses (including
reclassification adjustments) that are not recognized in profit and loss as required or
permitted by other NFRSs.
The components of other comprehensive income include;
1. Changes in revaluation surplus
2. Re-measurements of defined benefit plans
3. Gains and losses arising from translating the financial statements of a foreign operation
4. Gains and losses from investments in equity instruments measured at fair value through
other comprehensive income in accordance NFRS related with financial instruments

The Institute of Chartered Accountants of Nepal 16


CAP II Paper 1: Advanced Accounting

5. The effective portion of gains and losses on hedging instruments in a cash flow hedge
For particular liabilities designed as at fair value through profit or loss, the amount of the
change in the fair value that is attributable to changes in the liability‘s credit risk.
d) A change in accounting policy should be made in the following conditions:
(i) If the change is required by some statute or for compliance with an Accounting
Standard.
(ii) Change would result in more appropriate presentation of the financial statement.
Change in accounting policy may have a material effect on the items of financial
statements. For example, if cost formula used for inventory valuation is changed from
weighted average to FIFO, or if interest is capitalized which was earlier not in practice,
or if proportionate amount of interest is changed to inventory which was earlier not the
practice, all these may increase or decrease the net profit. Unless the effect of such
change in accounting policy is quantified, the financial statements may not help the
users of accounts. Therefore, it is necessary to quantify and disclose the effect of
change on financial statement items like assets, liabilities, profit/loss.

e) Calculation of Profit from the Notional profit


If the contract account shows a loss, such loss must be fully provided. However, when
the contract account discloses a profit, all such profit cannot be deemed as profit since,
in a subsequent accounting period there may be escalation of costs and such other
contingencies. Therefore, only a proportion of the notional profit is deemed to be
profit and balance is credited to work in progress account. There are certain rules of
thumb regarding the portion of profit to be provided for contingencies. They are given
below:-
Completion stage Profits to be provided for contingencies
Below 25% Full
25% to less than 50% Two –Thirds
50% to less than 95% One- Third
95% and above Nil
The profits to be taken, as earned for the accounting period, must further be reduced
on what is called ‗cash basis‘. Cash basis is the proportion of cash received to work
certified. For example, if 40% of contract is completed, Rs. 60,000 is the notional
profit and 80% of work certified is received in cash from the contractee, profit earned
would be calculated as shown below:
Profit earned = Notional profit X 1/3 X80%
= Rs. 60,000 X 1/3 X80%
= Rs. 16,000
In the above example, out of the notional profit of Rs. 60,000, a sum of Rs. 16,000 will
be deemed to be the profit for the accounting period and the balance of Rs. 44,000 will
be carried forward towards contingencies.
Alternate Solution
If the contract account shows a loss, such loss must be fully provided. However, when

The Institute of Chartered Accountants of Nepal 17


Suggested Answer - June 2018

the contract account discloses a profit, all such profit cannot be deemed as profit since,
in a subsequent accounting period there may be escalation of costs and such other
contingencies. Therefore, only a proportion of the notional profit is deemed to be
profit.
As per the provision of NAS 11, profit to be recognized during a particular year will
be calculated as follows:

Total Contract Revenue xxx


Total Estimated Cost xxx
Total Estimated or Notional Profit xxx

Profit to be recognized during a particular year= Total Estimated Profit* Stage of


completion- Profit recognized up to previous year

Where, Stage of completion can be calculated by any of the following method

i. the proportion of contract costs incurred for work performed to date bear to the
estimated total contract costs;
ii. surveys of work performed; or
iii. completion of a physical proportion of the contract work

The Institute of Chartered Accountants of Nepal 18


CAP II Paper 1: Advanced Accounting

Specific Comments on the performance of the students


Batch: - June 2018
Level: - CAP-II
Subject: Advanced Accounting

Question No. 1
Calculation of Partners' capital and depreciation on Building is not done well.
Calculation of profit is also not done by majority of students. Generally, students had
problem in determining purchase consideration and showing profit made during
period. Most of the students randomly attempted the question; however, none could
solve it well. Some brought different way than model answer. Almost all students
confused in calculating 6 months profit. Few were confused on written off and
depreciation of Building, computation of purchase consideration. Students were not
able to calculate profit and drawing.
Question No. 2
(a) Calculation of Debtors and Creditors is wrong in some cases. Most of the students
confused on one month credit policy.
(b) Majority of students are lack of knowledge while preparing CFS. Cash flow
Statement was correctly answered by few students only. Most of the students were
confused on drawing depreciation.
Question No. 3
(a) Loading on stocks not accurate in some cases and also calculation of shortage of
stock is not done by majority of students. Most of the students confused on
calculation profit on debtor amount and shortage/loss of stock.
(b) Knowledge of provisioning is lacking. Few are confused to apply rate on insured
and SOL.
Question No. 4
(a) Majority of students fails to understand the question. Almost nil students
answered Investment account. Students failed to compute the amortizing schedule
Calculation of reinsurance portion is not done right by majority of students.
Question No. 5
(a) Goods on approval basis are not calculated by majority students. Few confused on
delivery concept, consignment, and sales by approval.
(b) Difference between provision and contingent liability is not clear.
(c) Calculation of Ex. Gain/Loss is not done due to lack of concept.
Question No. 6
(a) Majority of students fails to define life insurance fund.
(b) Almost none of the students fails to attempt the question.
(c) Majority of students did not have knowledge about OCI.
(d) Lack of concept.

The Institute of Chartered Accountants of Nepal 19


Suggested Answer - June 2018

Paper 2:

Audit & Assurance

The Institute of Chartered Accountants of Nepal 20


CAP II Paper 2: Audit & Assurance

Maximum Marks - 100

Total No. of Questions- 7 Total No. of Printed Pages- 10

Time Allowed - 3 Hours


Marks
Attempt all questions.
As an auditor, give your opinion with explanations on the following cases: (45=20)
a) AJ Limited has an investment worth NPR 1,000,000 in its financial statements at 31st
Ashadh 2074. Due to the continuing recession, the investment reduced in value to NPR
900,000 by 15th Shrawan 2074.
b) MNS Ltd. (The Company) is engaged in manufacturing business. The book value of
plant & machinery of the company was Rs. 900 million as on Ashadh end 2073
(purchased at Rs. 1,000 million on 1st Shrawan 2072). It provided depreciation on
straight line basis at 10% per annum based on useful life of the plant & machinery.
Imported asset of Rs. 100 million, the component of above plant & machinery was
acquired on 1st Shrawan 2073 that would be obsolete in 2 years. The company wants to
write off this asset over 2 years. Can the company do so?
c) X Ltd. entered into an agreement with Y Ltd. to dispatch goods valuing Rs. one lakh
every month for six months upon receipt of entire payments. Y Ltd. accordingly made
the payment. In third month due to a natural calamity Y Ltd. requested X Ltd. not to
dispatch until further notice.

d) During the financial year 2073/74, Y & R Private Limited, a service providing company
purchased generator of Rs. 2 million for smooth functioning of its office. The accountant
claims that there is no necessity to provide for depreciation in respect of generator as it
was kept standby but not used at all during the financial year.

Answer:
a) NAS 10 Events after the Reporting Period provides guidance whether an entity should
adjust its financial statements or shall disclose for the events after reporting period.
Since reduction in investment value occurred only after the reporting period, it is
indicative of condition that arose after the reporting period which is a non-adjusting event
as per para 3 of NAS 10. An entity shall not adjust the amounts recognised in financial
statement to reflect non-adjusting events after the reporting period. The decline in fair
value does not normally relate to the condition of the investments at the end of the
reporting period, but reflects circumstances that have arisen subsequently.
Therefore, the entity does not update the amounts disclosed for the investments as at the
end of the reporting period, however it may need to give additional disclosure about the

The Institute of Chartered Accountants of Nepal 21


Suggested Answer - June 2018

nature of event and an estimate of its financial effect , or a statement that such an estimate
cannot be made.

b) As per Nepal Accounting Standard 16 (Property, Plant and Equipment), each part of an
item of property, plant and equipment with a cost that is significant in relation to the total
cost of the item shall be depreciated separately.
An entity shall allocate the amount initially recognised in respect of an item of property,
plant and equipment to its significant parts and depreciate separately each such part. To
the extent that an entity depreciates separately some parts of an item of PPE it also can
depreciate separately the remainder of the item. As it appears that imported assets of Rs.
100 million, which is component of plant and machinery, is having independent useful
life.
Therefore, the company can choose to depreciate the significant parts at 10% p.a. and
remainder imported assets over two years.

c) NAS 18 ―Revenue‖ specifies that revenue from sale of goods should be recognized when
following conditions have been fulfilled:
i) The seller of the goods has transferred all significant risks and rewards of ownership
to the buyer.
ii) the seller retain no effective control of the goods sold usually associated with
ownership;
iii) The amount of revenue can be measured reliably.
iv) It is probable that the economic benefits associated with the transaction will flow to
the entity and
v) The cost in respect of the transaction can be measured reliably.

In this case X Ltd had transferred the significant risk and rewards of the property at an
agreed price. As such sale has been fully completed because upon receipt of the entire
payment. X Ltd was required to dispatch goods valuing Rs 100,000 for six month out of
its inventory. However, in the third month, Y Ltd requested to stop dispatch until further
intimation due to a natural calamity. X Ltd had transferred the goods at an agreed price
and all significant risks and rewards. The delivery was to be effected as per the schedules
indicated by Y Ltd. As per NAS 18, Revenue, mere postponement of delivery at buyers
request does not alter the period in which revenue should be recognized. Accordingly X
Ltd should recognize the entire 600,000 as Sales.

d) As per para 55 of NAS 16 "Property, Plant and Equipment", depreciation of an assets


begin when it is available for use, i.e. when it is in the location and condition necessary
for it to be capable of operating in the manner intended by the management.
Depreciation is a measure of the wearing out, consumption or other loss of value of a
depreciable asset arising from use, efflux of time or obsolescence through technology and

The Institute of Chartered Accountants of Nepal 22


CAP II Paper 2: Audit & Assurance

market changes. Thus, depreciation has to be charged even in case of these assets which
are not used at all during the year but by mere efflux of time provided such assets qualify
as depreciable assets.
When the generator was kept ready for use as stand-by, it means it was intended to be
used for the purpose of business. Depreciation in respect of this generator would have
been provided in the accounts for the year ended 31st Ashadh 2074. If there is an
intention to use an asset, though it may not have actually been used, it is a 'constructive'
or 'passive' use and eligible for charging depreciation.

2. Give your comments on the following cases: (45=20)


a) Mr. KC, Partner of CA firm involved in Audit of X Limited was offered with luxury
car by X Limited for his personal use till financial statements is approved in AGM.
b) Your firm has been appointed as the statutory auditor of Super Express Bank Ltd. for
the financial year 2074/75. You, as the engagement partner, are in the process of
drafting audit plan of the said audit. When obtaining an understanding and
performing a preliminary assessment of the internal audit function for drafting your
audit plan, what are the important criteria to be considered.
c) While auditing accounts of a public limited company for the year ended 31st Ashadh
2074, an auditor found out an error in the valuation of inventory, which affects the
financial statement materially.
d) Auditor of Maya Limited was unable to confirm the existence and valuation of
imported inventory lying with the transporter and accepted a certificate from the
management without obtaining audit evidence. The inventory lying with the
transporter is material to the financial statements.
Answer:
a) A professional accountant in public practice or an immediate or close family member may
be offered gifts and hospitality from a client that may create threats to compliance with
the fundamental principles of code of ethics.
When a professional accountant in public practice or an immediate or close family
member is offered gift and hospitality, the situation shall be evaluated. In the instant case,
an offer of using luxury car for personal use may influence the opinion of professional
accountant in public practicee. Self-interest threats to objectivity or confidentiality are
created when gifts and hospitality is made to unduly influence actions or decisions,
encourage illegal or dishonest behavior, or obtain confidential information. Intimidation
threats to objectivity or confidentiality are created if such gifts & hospitality is accepted
by the professional accountant in public practice or an immediate or close family
member.
A professional accountant in public practice shall evaluate the significance of such
threats and apply safeguards when necessary to eliminate the threats or reduce them to an

The Institute of Chartered Accountants of Nepal 23


Suggested Answer - June 2018

acceptable level. Accordingly, when the threats cannot be eliminated or reduced to an


acceptable level through the application of safeguards, such offer shall not be accepted. .

b) As per NSA 610; "Using the work of internal auditors" the statutory auditor should
consider the activities of internal auditors and their effect, if any, on statutory audit
procedures. In the light of aforesaid provision of NSA, the following aspects should be
considered for drafting the audit plan of Super Express Bank Ltd. for the financial year
2074/75:
1. Organizational Status: specific status of internal auditing in the entity and the effect
this has on its ability to be objective. In the ideal situation, internal auditing will
report to the highest level of management and be free of any other operating
responsibility. Any constraints or restrictions placed on internal auditing by
management would need to be carefully considered. In particular, the internal auditors
should be free to communicate fully with the external auditor.

2. Scope of Function: the nature and extent of internal auditing assignments performed.
The external auditor would also need to consider whether management acts on
internal audit recommendations and how this is evidenced.

3. Technical Competence: whether internal auditing is performed by persons having


adequate technical training and proficiency as internal auditors. The external auditor
may, for example, review the policies for hiring and training the internal auditing staff
and their experience and professional qualifications.

4. Due Professional Care: whether internal auditing is properly planned, supervised,


reviewed and documented. The existence of adequate audit manuals, work programs
and working papers would be considered.

c) NSA 450 ―Evaluation of Misstatements identified during the audit‖ deals with the
auditor‘s responsibility to evaluate the effect of identified misstatements on the audit and
of uncorrected misstatements, if any, on the financial statements. The auditor should
consider requesting the management to adjust the financial information or consider
extending his audit procedures. If the management refuses to adjust the financial
information and the results of extended audit procedures do not enable the auditor to
conclude that the aggregate of uncorrected misstatements is not material, the auditor
should express a qualified or adverse opinion, as appropriate.
In the instant case, the auditor has detected the material errors affecting the financial
statements; the auditor should communicate his findings to the management on a timely
basis, consider the implications on true and fair view and think about modifying the
report.

d) As per NSA 580 ―Written Representations‖ auditor may rely on the representation by
the management but he should seek corroborative audit evidence. The management

The Institute of Chartered Accountants of Nepal 24


CAP II Paper 2: Audit & Assurance

representation cannot substitute other evidence that the auditor could reasonably expect
to be available to the auditors.
Also, NSA 501 ―Audit Evidence‖ Specific Consideration for Selected items‖ requires
obtaining sufficient and appropriate evidence regarding the existence and condition of
inventory lying with third party if material to the financial statement.

Supporting evidences can be obtained from inside or outside sources. The audit evidence
for verification of inventory lying with the transporter - say purchase order, invoice,
custom clearance certificate, inspection, confirmation from transporter etc. are available
evidences which auditor should verify.

Just because the management had confirmed the existence and valuation of imported
inventory lying with the transporter the auditor cannot shrink his responsibility. This is
negligence on his part.

3. Answer the following: (35=15)


a) What are the provisions on ‗Timing of Liaison and Coordination‘ amongst internal
audit and external audit in NSA 610?
b) Mr. Shyam was appointed as the auditor of M/s Himalayan Ltd. and intends to apply
the concept of materiality for the financial statements as a whole. Please guide him as
to the factors that may affect the identification of an appropriate benchmark for this
purpose.
c) Explain the concept of True and Fair View.

Answer:
a) When planning to use the work of internal auditing, the external auditor will need to
consider internal auditing‘s tentative plan for the period and discuss it at as early a stage
as possible.
Where the work of internal auditing is to be a factor in determining the nature, timing
and extent of the external auditor‘s procedures, it is desirable to agree in advance the
timing of such work, the extent of audit coverage, test levels and proposed methods of
sample selection, documentation of the work performed and review and reporting
procedures.
Liaison with internal auditing is more effective when meetings are held at appropriate
intervals during the period. The external auditor would need to be advised of and have
access to relevant internal auditing reports and be kept informed of any significant
matter that comes to the internal auditor‘s attention which may affect the work of the
external auditor. Similarly, the external auditor would ordinarily inform the internal
auditor of any significant matters which may affect internal auditing.

The Institute of Chartered Accountants of Nepal 25


Suggested Answer - June 2018

b) Use of benchmark in determining Materiality: NSA 320 Materiality in planning and


performing an audit prescribes the use of benchmarks in determining materiality for the
Financial Statements as a Whole. Accordingly determining materiality involves the
exercise of professional judgment. A percentage is often applied to a chosen benchmark
as a starting point in determining materiality for the financial statements as a whole.
Factors that may affect the identification of an appropriate benchmark include the
following:
i. The elements of the financial statements (for example, assets, liabilities, equity,
revenue, expenses);
ii. Whether there are items on which the attention of the users of the particular entity‘s
financial statements tends to be focused (for example, for the purpose of evaluating
financial performance users may tend to focus on profit, revenue or net assets);
iii. The nature of the entity, where the entity is at in its life cycle, and the industry and
economic environment in which the entity operates;
iv. The entity‘s ownership structure and the way it is financed (for example, if an entity is
financed solely by debt rather than equity, users may put more emphasis on assets,
and claims on them, than on the entity‘s earnings); and
v. The relative volatility of the benchmark.

c) 'True and fair view' is a phrase usually auditors use to express audit opinion on the
financial statements of an entity. It implies that the financial statements are presented
fairly in all materials respect; the position, performance, cash flows and changes in
equity of the entity. The auditor expresses such opinion upon assessment of the internal
control system of the entity and test checking the financial transactions carried out
during the fiscal year. The auditor's act is guided by the provisions set forth in the Nepal
Standards on Auditing together with the Code of Ethics applicable to the professional
accountants.

As per NSA 200 ―Overall Objectives of the Independent Auditor and the Conduct of an
Audit in accordance with NSA‖, the auditor‘s expression of true and fair view is
supposed to be received as only the ―reasonable assurance and not the absolute
assurance‖ of the state of the financial statements. This implies that the users are not
supposed to absolutely rely on auditor‘s judgment for making their financial decisions
relating to the entity. This is because the auditor is not expected to, and cannot, reduce
audit risk to zero and cannot therefore obtain absolute assurance that the financial
statements are free from material misstatement due to fraud or error. This is because there
are inherent limitations of an audit, which result in most of the audit evidence on which
the auditor draws conclusions and bases the auditor‘s opinion being persuasive rather than
conclusive.
Broadly speaking, the financial statements are considered as presenting to true and fair
view if:
 The information contained in them are not materially misstated;

The Institute of Chartered Accountants of Nepal 26


CAP II Paper 2: Audit & Assurance

 There is an appropriate application of Nepal Accounting Standards, with additional


disclosure in the case of companies registered under Companies Act. In the case of
other entities there is an appropriate application of generally accepted accounting
principles as is applicable; and
 They comply with the provisions of applicable laws and regulations of the company.

4. Answer/Comment on the following: (35=15)


a) What is ‗Independence of Mind‘ and ‗Independence in Appearance‘?
b) You are the statutory auditor of PQR Ltd., while carrying out the audit you found
existence of certain threats to objectivity at significant level. What is your duty in
such situation? Give the examples of safeguards you will apply in such situation?
c) Explain the provision relating to 'Conflicts of Interest' with reference to Part B of
Code of Ethics.

Answer:
a) In case of audit engagements, it is in the public interest and, therefore, required by section
290 0f Code of ethics, that members of audit teams, firms and network firms shall be
independent of audit clients.
Independence of Mind
Independence is a state of mind and personal character and an enlightened view of the
professional duties involved. The state of mind that permits the expression of a
conclusion without being affected by influences that compromise professional judgment,
thereby allowing an individual to act with integrity and exercise objectivity and
professional skepticism.
Independence in Appearance
Independence of auditor must not only exist in fact, but should also appear to exist to all
reasonable persons. The avoidance of facts and circumstances that are so significant that a
reasonable and informed third party would be likely to conclude, weighing all the specific
facts and circumstances, that a firm‘s, or a member of the audit team‘s, integrity,
objectivity or professional skepticism has been compromised.

b) As per Section 280 (4) of ICAN Code of Ethics, I should evaluate the significance of such
threats and should apply safeguards when necessary to eliminate them or reduce them to
an acceptable level. The safeguards that should be applied are:
i. Withdrawing from the engagement team;
ii. Supervisory procedures;
iii. Terminating the financial or business relationship giving rise to the threat;
iv. Discussing the issue with higher levels of management within the firm;
v. Discussing the issue with those charged with governance of the client.

The Institute of Chartered Accountants of Nepal 27


Suggested Answer - June 2018

If safeguards cannot eliminate or reduce the threat to an acceptable level, I should decline
or terminate the audit engagement of PQR Ltd.

Section 220 of Code of Ethics contains the provision relating to Conflict of Interest. A
professional accountant in public practice may be faced with a conflict of interest when
performing a professional service. A conflict of interest creates a threat to objectivity and
may create threats to other fundamental principles.

Such threats may be created when:


 The professional accountant provides a professional service related to a particular
matter for two or more clients whose interests with respect to that matter are in
conflict or
 The interests of the professional accountant with respect to a particular matter and the
interests of the client for whom the professional accountant provides a professional
service related to that matter are in conflict.

A professional accountant shall not allow a conflict of interest to compromise


professional or business judgement. The professional accountant in public practice shall
apply safeguards, when necessary, to eliminate the threats to compliance or reduce them
to acceptable level.

5. Answer the following: (25=10)


a) What are the matters to be audited in view of propriety as per Audit Act,
2048 as amended in 2073?
b) Describe the functions and duties of auditors as per Companies Act, 2063.

Answer:
a) Section 5 of the Audit Act 2048 (As amended in 2073) states that ; the Auditor General
shall audit following matters considering the propriety thereof :
(i) On the propriety of any expenditure and its authorization, if in the opinion of the
Auditor General such expenditure is a reckless one or is an abuse of national property,
whether fixed or current, despite that the expenditure conforms to the authorization,
and
(ii) On the propriety of all authorizations issued in respect of any grant of national
property whether fixed or current, or underwriting of any revenue, or any contract,
license or permits relating to mining, forest, water resources, etc. and any other act of
abandoning fixed or current assets of the nation.
Though the Auditor General may not include in his report minor items of discrepancy and
other items deemed by him as insignificant in view of their propriety which were
observed during the audit of income and expenditures.

The Institute of Chartered Accountants of Nepal 28


CAP II Paper 2: Audit & Assurance

b) The function and duties of auditors as per section 115 of the Companies Act,
2063 are as follows:
(1) The auditor shall, addressing the shareholders or the appointing authority, submit to the
company his/her report, certifying the balance sheet, profit and loss account and cash
flow statement based on the books of account, records and accounts audited by
him/her.
(2) The audit report shall be prepared in accordance with the prevailing law or in
consonance with the audit standards prescribed by the competent body; and such
report shall state the matters to be set out under this Act, as per necessity.
(3) The audit report as referred to in Sub-section (2) shall also indicate the following
matters, inter alia:
(i) Whether such information and explanations have been made available as were
required for the completion of audit;
(ii) Whether the books of account as required by this Act have been properly
maintained by the company in a manner to reflect the real affairs of its business;
(iii) Whether the balance sheet, profit and loss account and cash flow statements
received have been prepared in compliance with the accounting standards
prescribed under the prevailing law and whether such statements are in agreement
with the books of account maintained by the company;
(iv) Whether, in the opinion of the auditor based on the explanations and information
made available in the course of auditing, the present balance sheet properly
reflects the financial situation of the company, and the profit and loss account and
cash flow statement for the year ended on the same date properly reflect the profit
and loss, cash flow of the company, respectively;
(v) Whether the board of directors or any representative or any employee has acted
contrary to law or misappropriated any property of the company or caused any
loss or damage to the company or not;
(vi) Whether any accounting fraud has been committed in the company
(vii) Suggestion, if any

6. Write short notes on the following: (42.5=10)


a) Audit Risk
b) Audit Committee
c) Cut Off Procedure
d) Corporate Governance
Answer:
a) Audit Risk
The risk that the auditor expresses an inappropriate audit opinion when the financial
statements are materially misstated. Audit risk is a function of the risks of material
misstatement and detection risk. Risk of material misstatement is a product of inherent

The Institute of Chartered Accountants of Nepal 29


Suggested Answer - June 2018

risk and control risk whereas detection risk is a situation where auditors audit procedures
fail to detect material misstatement. To reduce audit risk at acceptable level, the auditor
should appropriately respond to the assessed risk.

b) Audit Committee
As per Section 164 of Companies Act as amended, a listed company with paid up capital
of thirty million rupees or more or a company which is fully or partly owned by the
Government of Nepal shall form an audit committee under the Chairpersonship of a
director who is not involved in the day-to–day operations of the company and consisting
of at least three members. At least one member of the audit committee shall be an
experienced person having obtained professional certificate in accounting or a person
having gained experience in accounting and financial field after having obtained at least
bachelor’s degree in accounts, commerce, management, finance or economics. The
committee is responsible to review internal control systems of the company.

c) Cut Off Procedure


Cut-off procedures mean procedures employed to ensure the separation of transactions at
the end of one year from those in the commencement of the next year. For the cut off
procedure of inventory, the auditor should satisfy himself by examination and test check
that these procedures adequately ensure that:
 Goods purchased for which property has passed to the client have in fact been
included in inventories and that the liability if any, has been provided for.
 Goods sold have been excluded from the inventories and credit has been taken for
sales.
The auditor may examine a sample of documents evidencing the movement of stocks into
and out of stores, including documents pertaining to period shortly before and shortly
after the cutoff date, and check whether the stocks represented by those documents were
included or excluded, as appropriate, during the stock-taking.
d) Corporate Governance
Corporate Governance is a system by which the business organizations are directed and
controlled. It is a set of processes, customs, policies, laws, and institutions affecting the
way an organization is directed, administered or controlled. The corporate governance
structure specifies the allocation of rights and responsibilities among board, managers,
shareholders, employees, suppliers, customers, government etc. and spells out the rules
and procedures for making decisions on corporate affairs.

Corporate governance is thus 'an internal system encompassing policies, processes and
people, which serves the needs of shareholders and other stakeholders, by directing and
controlling management activities with corporate fairness, transparency, independence,
integrity and accountability. Further, it influences how the objectives of the company are
set and achieved, how risk is monitored and assessed, and how performance is optimized.
It encourages companies to create value (through entrepreneurism, innovation,
development and exploration) and provide accountability and control systems
commensurate with the risks involved

The Institute of Chartered Accountants of Nepal 30


CAP II Paper 2: Audit & Assurance

7. Distinguish between: (25=10)


a) Cost Accounting and Financial Accounting
b) Reserves and Provisions
Answer:
a) Distinction between Cost Accounting and Financial Accounting
Cost Accounting is a close follower of financial accounting. It is not independent of
financial accounting. Though there are common grounds between the two, the important
differences are given below:
(i) Reporting: The major objective of financial accounting is external reporting whereas the
focus of cost accounting has been essentially internal i.e. management.
(ii) Flexibility: Financial accounting is mostly historical or after the event while cost
accounting is much more flexible and open minded and includes in both retrospective
and anticipatory calculations.
(iii)Nature: Financial accounting classifies records, presents and interprets in terms of
money transactions whereas cost accounting classifies, records, presents and interprets
in a significant manner the material, labour and overhead costs involved in
manufacturing and selling each product.
(iv) Financial accounting uses Relevant Accounting Standards while recording, classifying
summarizing and reputing business transactions whereas cost accounting is not bound
to use such Accounting Standards and it can use any technique or practice which
generates useful information.
(v) Time Span: Financial accounting data are developed for a definite period, usually a
year, half year or a quarter, but cost accounting reports and statements can be prepared
whenever needed.
(vi) Accounting Method: Financial Accounting follows the double-entry system for
recording, classifying and summarizing business transactions.
The data under Cost Accounting can be gathered for small or large segments or activities
of an organization and monetary as well as other measures can be used for different
activities in the organization.

b) Reserves and Provisions


(i) Reserve is an appropriation of profit whereas provision is a charge against Profit.
(ii) Reserves are not intended to meet specific liability, contingency or diminution in the
value of assets. Provisions are made to provide for depreciation, renewal or a known
liability or a disputed claim.
(iii) Reserves cannot be created unless there is a profit except revaluation reserve and
capital subsidy. Provisions must be created whether or not there is profit.
(iv) Reserves are generally optional except in certain situations – Capital Redemption
reserve, Debenture Redemption Reserve, etc. Provisions are not optional and have to

The Institute of Chartered Accountants of Nepal 31


Suggested Answer - June 2018

be made as per generally accepted accounting principles.


(v) Reserves are shown on the liability side. Provisions for depreciation and provision for
doubtful debts are shown as deduction from respective assets. Provision for liability is
shown on the liability side.

The Institute of Chartered Accountants of Nepal 32


CAP II Paper 2: Audit & Assurance

Specific Comments on the performance of the students


Batch: - June 2018
Level: - CAP-II
Subject: Audit and Assurance
Question No. 1
Most of the students did not understand the question about non-adjusting event. They are
unaware about NAS PPE(16). Students are aware about the related accounting standard and
provision. However, while answering the question they are not correctly given the
justification. It seems the student has surface knowledge only. Most of the students were not
able to relate Q.no. 1 (c) with revenue recognition. Some students cannot answer in line with
NAS, other irrelevant NASs are referred. 50% answer are irrelevant, could not linked with
NAS - 18
Question No. 2
(c) part is confusing, so mostly internal audit function has been ignored only focused on audit
plan. 2(b) most of the student wrote about audit planning instead of preliminary assessment
of internal audit function. The question is not understood by student properly. 'Self- int.
threat' could not be referred by majorities. Majorities 90%, fail to link the audit plan with
"internal audit function". No student can refer NSA 450.
Question No. 3
Satisfactorily. Instead of writing about timing of liaison and coordination most student write
on using the work of internal Auditor.
Question No. 4
Students found difficulty due to generalization of question.
Question No. 5
Not referred as per audit act, attempted definition of propriety audit. It seems student does
not understand the concept of proprietary audit properly.
Question No. 6
Most of all answered satisfactorily. There is confusion regarding Audit committee for some
students. They wrote the detail of Audit committee of ICAN, instead Audit committee as per
company Act.
Question No. 7
Differences are not specific. Attempted by most of the students and answering properly.
However, some student repeated same point again and again in different language.

The Institute of Chartered Accountants of Nepal 33


Suggested Answer - June 2018

Paper 3:

Corporate & Other Law

The Institute of Chartered Accountants of Nepal 34


CAP II Paper 3: Corporate & Other Law

Roll No……………. Maximum Marks - 100

Total No. of Questions - 7 Total No. of Printed Pages -12


Time Allowed - 3 Hours
Marks
Attempt all questions.
1. Answer the following questions: (5×5=25)
a) During the course of Audit of Nepal Commercial Bank Limited, auditor of the bank
found some irregularities during the course of audit. He would like the shareholders
to know about these irregularities. Therefore, he wants to have the bank‘s extra
ordinary general meeting (EGM) convened and he approaches you for consultation
for convening EGM. Could you explain how the EGM of the bank can be convened
pursuant to the Companies Act, 2063?
b) Mr. Ramesh Mahato, a renowned businessman is planning to carry out insurance
business in Nepal by establishing a private company. Give your suggestion/opinion to
Mr. Mahato on his plan by citing the relevant provision of the Companies Act, 2063.
c) Mr. 'A' is appointed as an auditor of Prime Development Bank Ltd. in this Fiscal
Year. He is willing to continue in this company for F/Y 2076/2077. Mr. B, who had
been a partner of A before 3 years, is also willing to be an auditor in this company. In
spite of this, company is planning to substitute another auditor. In this circumstances,
is the willingness of A and B is justified? If the company has planned to remove Mr.
A then how it can be removed? Justify your answer with reference to the Companies
Act, 2063.
d) Mentions the circumstances where special resolution shall be presented in the general
meeting of a company for decision.
e) A resolution is passed by the general meeting of Alliance Insurance Ltd. to appoint
Rajan as an independent Director. After three months an action is filed against him as
he was convicted for 1 year jail imprisonment in a case of assault, hence, disqualified.
Advise the company regarding :
i) The disqualifications of an independent director?
ii) Is Rajan is disqualified in this respect?
Answer
a) As per the section 82(2) of the Companies Act 2063, if in the course of auditing the
account of the bank, it is deemed necessary to call an extra ordinary general meeting
for discussion of the irregularities at the EGM, the auditor may request the board of
directors to call such meeting; and if the board of directors fails to call the meeting
accordingly, the auditor may make an application, setting out the matter, to the
Office of Company Registrar and if an application is so made, the office may call the
extra-ordinary general meeting of the company.
b) Besides incorporation of a company as a public company certain companies can be
incorporated as a public company only. According to the section 12 of the
companies Act 2063 a company carrying on the business of banking, financial

The Institute of Chartered Accountants of Nepal 35


Suggested Answer - June 2018

transactions, insurance business related transaction, stock exchange business,


provision fund etc. as prescribed. Thus Mr. Ramesh Mahato, if interested to carryout
insurance business in Nepal, it is mandatory to be registered company as public
company only.

c) According to section 110 of the Companies Act, 2063, every company must appoint
an auditor to audit its accounts.
The companies Act, 2063, in section 111, has provided different rules as to
appointment of auditor. An auditor, from the amongst the auditors registered or
licensed under the existing law, can be appointment, in case of a public company, by
the General Meeting and in the case of a private company as provided in its
Memorandum of Association, Articles of Association and consensus agreement or by
the General Meeting in the absence of such arrangements.
The prime authority to appoint an auditor is general meeting. The company registrar
office also performs an appointing authority with reference of Board of Director due
to fail to appoint an auditor by AGM and fails to be held meeting and ceases to hold
the office for any reason.
The auditor appointed above remains in the office until next AGM is held. No auditor
or his/her partner or ex-partner or employee or ex-employee shall be appointed as
auditor for more than three consecutive terms to perform the audit of a public
company.

Both candidates are eligible in the eye of prevailing law. In regard to the willingness
of Mr. A, he can be continued up to 3 consecutive terms. However, the issue is
subject to the approval and consent of the shareholders meetings. Further, Mr. B is
also eligible to be appointed as an auditor of the said company. He has fulfilled
requirement as Mr. A has owned.
Regarding the removal process of an auditor, the companies Act, 2063, it its section
119, has mentioned, that, an auditor should be removed after completion of the audit
accounts of such F/Y. Auditor can be removed after fulfilling following conditions:
 Breaching the code of conduct of auditors or does any act against the interest of
the company which has appointed him as the auditor, or commits ant act contrary
to the prevailing law.
 Auditor may be removed through same process whereby he/she was appointed as
auditor.
 Prior information should be given to Nepal Chartered Accountants Institute.
 Approval should be taken from regulatory authority as provided by the prevailing
law.
 Approval from the office in the time while failing such authority.
 The auditor should be provided with a reasonable opportunity to defend
him/herself.
d) In a general meeting of the Company, it is required to submit matters to be decided
as resolution which may be either ordinary or special resolution. However Section
83 of the Companies Act prescribes the subject matter which requires to be
presented as special resolutions in the general meeting of a company for decision.

The Institute of Chartered Accountants of Nepal 36


CAP II Paper 3: Corporate & Other Law

i) Increasing the authorized capital of the company,


ii) Decreasing or altering share capital of the company,
iii) Altering the name or main objectives of the company,
iv) Amalgamating one company into another company,
v) Issuing bonus share,
vi) Buying back of own shares by the company,
vii) Selling shares at a discount ,
viii) Converting a private company into a public company or vice versa,
ix) Such other matter in respect of which the company is required by this Act or
the articles of association to adopt a special resolution.

e)
i) Pursuant to Sub-section (2) of Section 89 of the Companies Act, 2063, the following
persons shall not be eligible to be appointed to the office of independent director:
1. A person as referred to in Sub-section (1) of section 89.
2. A shareholder of the concerned company.
3. Who has not obtained at least bachelor degree in a subject that is related to the
business to be carried on by the concerned company and gained at least ten
years of experience in the related field or in the company management affairs or
who has not obtained at least bachelor degree in finance, economics,
management, accounts, statistics, commerce, trade or law and gained at least ten
years of experience in the related field.
4. An officer, auditor or employee of the concerned company or a period of three
years has not lapsed after his/her retirement from any such office.
5. The close relative of the office of the concerned company;
6. An auditor of the concerned company or his/her partner.
ii) As per Sub-section 1 of Section 89 of the Companies Act, 2063, A person who is
convicted of an offense of corruption or of an offense involving moral turpitude and
convicted of an offense of theft, fraud, forgery or embezzlement or misuse of goods
or funds entrust to him/her, in an authorized manner, and sentenced in respect thereof,
a period of three year has not elapsed from the expiry of the sentence he cannot be
appointed as an independent director of a public limited company. In the above case,
Mr. Rajan was convicted in an offense of assault which does not involve moral
turpitude and does not come within the categories as mentioned in the Act. Therefore,
he can be appointed as an independent director of the company.

2. Answer the following questions: (3×5=15)


a) Mr. Ramesh Raj a promoter of the Gorkha Bank Ltd approached CEO of the bank
and requested that he wanted to take loan from the bank. CEO of bank refused to
allow the loan stating that there are various acts that cannot be done by Banks &
Financial Institution. Mr. Raj was confused why he could not obtain loan from his
own bank and approached you and asked you, what are the acts prohibited to be done
by Banks & Financial Institution as per the Banks & Financial Institutions Act, 2073.

The Institute of Chartered Accountants of Nepal 37


Suggested Answer - June 2018

b) The board of directors of the ABC Bank Ltd has decided to distribute bonus share
from the amount accumulated in the General Reserve Fund for the fiscal year
2073/74. As an internal auditor of the bank, give your opinion on the decision of the
board of directors referring the relevant provision of the Bank and Financial
Institution Act 2073.
c) Mr. Dhakal is a CEO of a Nepal Commercial Bank Ltd. established and worked for
Five Years in Nepal. As he heard that NRB provides loan and refinance to the bank
like this. You are there in a position of economic advisor. Give suggestion regarding
the loan and refinance provision as prescribed by the NRB Act, 2058.
Answer
a) Section 50 of the Banks& Financial Institutions Act, 2073 prescribes the provision
of the act prohibited to be done by Banks & Financial Institutions. According to it
Banks & Financial Institutions are not allowed to carry out, or cause to be carried out
the following activities:
1.Purchasing or selling goods for commercial purpose or constructing building or
purchasing any immovable property except when it is required for its own use;
2.Advancing credit against the security of its own share.
3.Supplying credit or facility to any directors, person who have subscribed one percent
or more of its share, chief executive or any family members of such persons,
managing agents or any firms, companies or institutions which are entitled or
nominate or appoint directors; or any firm companies or institutions in which the
institution has a financial interest.
4.Supplying credit or facility in an amount exceeding such percentage of capital fund
as may be prescribed by the Nepal Rastra Bank to a single customer, company and
companies or partnership firm of single group.
5.Supplying any type of credit to any person, firm, company or institution against
gurantee given by the promoters, directors, or chief executive.
6.Making investment in the securities of Bank or financial institutions of Classes A, B
and C classified by the Nepal Rastra Bank.
7.Making investment of an amount exceeding the limit prescribed by the Nepal Rastra
Bank in the share capital of any other institutions.
8.Indulging with other licensed institutions to mutually create any type of monopoly or
any other type of controlled practice in the financial transactions.
9.Doing any kind of act which is capable of creating any artificial obstruction in the
competitive environment of the financial sector, with the intention of deriving undue
advantage.
10. Doing such other acts prohibited from being done by a bank or financial institutions
as may be prescribed by the Nepal Rastra Bank.

b) As per Section 44 of the Banks and Financial Institution Act, 2073;

The Institute of Chartered Accountants of Nepal 38


CAP II Paper 3: Corporate & Other Law

(1) A licensed institution must maintain a general reserve fund at least twenty per cent
of the net profits of each year shall be kept on being credited to such fund until the
amount of such fund doubles the paid-up capital and must be continued adding at
least ten percent each year thereafter
(2) The amount credited to the reserve fund of a licensed institution under Sub-section
(1) may not be invested or transferred to any other head without the prior approval of
the Nepal Rastra Bank.
So, the decision taken by the board of directors of the ABC bank is against the
provision of BAFIA which is not executable.

c) NRB is a banker of the bank. It inspects and supervises the mobilization of financial
resources whether it has collected through public or from loan and refinance or its
promoters. As this matter is concerned, NRB may make available loan and refinance
to commercial bank and financial institution under the provision of section 49 of
NRB Act. The provision is as follows:
(1) The Bank may, subject to the terms and conditions prescribed by it, make available
loan and refinance to commercial banks and financial institutions for a maximum
period of one year against the security of the following assets:-
(a) International negotiable instrument referred to in Clause (e) of Sub-section (1)
of Section 66;
(b) The debt bond issued by Government of Nepal payable within Nepal;
(c) The deposits accumulated in the Bank or the gold and precious metals, which
the Bank may transact under this Act;
(d) The bill of exchange or the promissory notes referred to in Sub-section (1) of
Section 48;
(e) Other securities as prescribed.
(2) Notwithstanding anything contained in Sub-section (1) of the Bank may provide any
type of credit to a commercial bank and financial institution for a maximum period
of one year in cases where Government of Nepal has, for the sake of public interest
and welfare, deemed it appropriate to provide loan and has requested the Bank
therefore and Government of Nepal has given a guarantee of securities of prevailing
market rate for such loan or in extraordinary circumstances where the Bank has to
work as a lender of the last resort.
(3) Loan, as provided in accordance with the provision of sub section (1) or (2), shall be
renewed for the period of One year subject to the terms and conditions prescribed by
it.

3. Answer the following questions: (2×5=10)


a) Under what circumstances, the registration certificate of insurers cannot be renewed.
Explain in the light of relevant provisions of the Insurance Act, 2049.
b) Securities Act, 2063 has made provision for Compensation Fund to be established by
a Stock Exchange and should made rule for the operation of such Compensation
Fund. Enumerate the provisions that should be included in such Rule.
Answer

The Institute of Chartered Accountants of Nepal 39


Suggested Answer - June 2018

a) Section 11A of Insurance Act, 2049 mentions few circumstances that can lead to
non-renewal of certificate of registration of an insurer. These circumstances are as
follows:
i) In case it doesn't submit the balance sheet according to section 23
ii) in case it doesn't submit the statement of income according to section 24
iii) In case it doesn't submit auditor's report according to section 25
iv) In case it doesn't submit the actuary's report according to section 26
v) In case it doesn't pay the insurance service fee according to section 40
vi) In case it has been prohibited from engaging in the insurance business according
to section 12A.

In case the certificate of registration of an insurer cannot be renewed because of any


of the circumstances mentioned in the above, the Board shall notify the insurer
accordingly within 15 days from the date of emergence of such circumstances.

b) Section 53 of the Securities Act, 2063 has made provision for Compensation Fund to
be established by a Stock Exchange to protect investor against possible loss or
damage.
As per the section 54 of the Securities Act, 2063 made provisions shall be made in the
Rules in relation to the operation of the compensation fund as follows:
(a) Provisions relating to the deposit of money to the fund,
(b) Maximum amount to be paid as compensation from the fund,
(c) Provisions relating to the accounts and audit of the fund,
(d) Conditions for making claim to obtain amount from the compensation fund and
procedures from making such a claim,
(e) Conditions where any claim cannot be made on the compensation fund,
(f) Procedures for taking action and making decision on payment of money as
claimed from the compensation fund,
(g) Maximum limit of amount payable as compensation to one person,
(h) Other necessary matters in relation to the examination of compensation claims,
(i) Provisions to be made in the event of the revocation of the license of a stock
exchange,
J. Other necessary provisions in relation to compensation.

4. Answer the following questions: (2×5=10)


a) Krishna Tower Ltd. is going to distribute bonus of F/Y 2074/75. In this year some
of the employee has involved in riots and indisciplinery acts. Company is in
dilemma and confusion how bonus is to be distributed to such employee? Suggest in
following query in accordance with the Bonus Act, 2030.
i) What are the eligibility criteria for distributing bonus?
ii) Is there any restriction to have or to distribute bonus?
b) What are the provisions prescribed by the Labour Act, 2074 regarding working
hours and over time work?

The Institute of Chartered Accountants of Nepal 40


CAP II Paper 3: Corporate & Other Law

Answer
a)
i) Bonus Act, 2030, section 6, has mentioned the eligibility criteria of bonus as
follows:
(1) An employee who has worked for the half period to be worked in a fiscal year,
shall be entitled to obtain bonus under to this Act.
Provided that, no employee shall be entitled to obtain Bonus who has worked
casually or in a shift basis.
(2) For the purpose of Sub-section (1), the following periods shall also be computed
as a period where an employee has worked.
(a) A period kept on reserve under any contract or under Section 11 of the
Labour Act, 2048 (1991).
(b) A period under which an employee is on any leave with salary.
(c) A period of disablement caused by accident arising in course of business of
the enterprise.

ii) There are restriction provisions to have or to distribute bonus in the Bonus Act,
2030. Section 8 has prescribed the restriction provision that an employee shall
not be entitled to obtain bonus under this Act, if he/she is punished or dismissed
from service for committing any act as follows:
(a) Theft of the property of the enterprise or any damage to such property.
(b) Illegal strike or abetment to other for such strike,
(c) Riots or breaching of discipline.
Provided that, this Section shall not be deemed to be prejudiced to obtain in the
case of the bonus for a period before committing such a punishable act.

b) Section 28, 29 and 30 of the Labor Act, 2074 prescribed provision of working hours,
and overtime work of the labor.
As per the provision of employees or workers are allowed to work as follows:
i. Employer cannot engage any labor more than 8 hour a day and 48 hours a week.
ii. Labor should be allowed recess of half an hour after 5 hours of continuous work.
iii. If the nature of work is of continuous and cannot be stop, arrangement of recess
should be made in rotation basis.
iv. Recess period will be calculated in working hour.
v. Labor shall not be forced to work over time. However if non completion of work
affect Life, security or health of any person or employer might incur heavy loss,
labor could be engage for overtime work.
vi. Overtime work as above shall not be more than 4 hours a day and 24 hours a
week.
In case of women workers, they can be engaged only from 6 a.m to 6.pm. If they are
to be engaged beyond 6 p.m i.e at night time, it required to take their consent by
arranging necessary safety measures and transportation facilities. In case of minor
workers it is prohibited to engage them from 6 p.m. to 6 a.m and is allowed to work 6
hours a day and 36 hours a week.

The Institute of Chartered Accountants of Nepal 41


Suggested Answer - June 2018

5. Answer the following questions: (2×5=10)


a) There is complaint lodged into the Council against Mr. Dikpak, the member of the
council, alleging that he used to share or distribute the profit the auditing fees or
remuneration with any other person other than a member of the Institute. How and
why this act is regulated by the existing law.
b) Discuss the provisions of the punishment as prescribed by the Nepal Chartered
Accountants Act, 2053?
Answer
a) As mentioned in the question that Mr. Dipak has alleged for the breach of conduct
prescribed for the member of the Council that he used to share or distribute the profit
the auditing fees or remuneration with any other person other than a member of the
Institute as mentioned in the section 34(3) Nepal Chartered Accountants Act 2053. It
is clear that the matter is concerned with the breach of conduct.
Regarding the regulation of such act, the Council is the final authority to punish the
alleged member. However, before punish to such allegation, the recommendation of
Disciplinary Committee have necessary. As provided in the section 14, the
Disciplinary Committee, has to recommend the Council to take necessary actions
after investigation upon complains lodged against any action, contrary to this Act or
Regulations by any member. The Disciplinary Committee have the authority similar
to a judicial court in respect of summoning concerned person and investigating
evidences and witnesses. This Committee shall recommend to the Council, along with
its opinion and finding, for necessary action against a member, if found guilty. The
Council may, considering such a recommendation, impose any of the following
punishment according to the degree of offense:-
(a) Reprimanding,
(b) Removing from the membership for a period up to five years,
(c) Prohibiting from carrying on the accounting profession for any particular period,
(d) Cancellation of the Certificate of Practice or membership.
Any Council member against whom the Disciplinary Committee, after investigating
upon the complaint of his action contrary to the Act or Regulations, Bye-laws or code
of conduct framed under the Act, has decided to recommend the Council to take
necessary action, shall not be allowed to attend and to vote at the Council meeting
where the Council is hearing at such recommendation. Before imposing a punishment
referred in above, the Council shall provide reasonable opportunity to the concerned
members to submit their clarification. The concerned member may, if he is not
satisfied with the decision referred to in above, file an appeal in the Appellate Court.

b) Section 41 of the Nepal Chartered Accountants Act, 2053 has prescribed punishment
as follows:
(1) A person, who carries out audit without obtaining a Certificate of Practice, pursuant
to this Act, shall be liable of punishment with a penalty of maximum two thousand
rupees or with an imprisonment for a maximum period of three months or with both.
(2) A person, who in contravention of Section 6 uses the name or the seal of the Institute
or exercises any type of authority bestowed to the Institute, shall be punished with a

The Institute of Chartered Accountants of Nepal 42


CAP II Paper 3: Corporate & Other Law

penalty of one thousand rupees maximum on first conviction, and on any subsequent
conviction thereafter, a maximum penalty of five thousand rupees or imprisonment
for a maximum period of six months or both.
Provided that this sub-section shall not apply to the organizations or university
established under their own legislation or the units within the Institute.
(3) A person, who has not obtained a Certificate of Practice and is proved to have signed
any document in capacity of the member holding Certificate of Practice, shall be
liable to punishment with a penalty up to two thousand rupees or imprisonment for a
period of up to three months or both.
(4) A member, who commits any act contrary to the provisions of this Act or Regulations
framed under this Act other than the provisions of this section, shall be suspended for
a maximum period of five years and shall be liable of punishment with a maximum
penalty of two thousand rupees or imprisonment for a maximum period of three
months or both.
(5) A complainant who lodges a complaint, without any reasonable cause to make
complaint and it is proved that the complaint was made with an intention to harass a
member, shall be liable to punishment with fine up to one thousand rupees.
(6) The complaint cases, except those to be heard under Section 14, lodged in the Council
against any member, pursuant to Section 35, shall be instituted in the concerned
Appellate Court.

6. Answer the following questions: (5×4=20)


a) When the drawer, acceptor or endorser of a Negotiable Instrument shall be
discharged from his/her liability?
b) State the meaning of Foreign Investment as per the Foreign Investment and
Technology Transfer Act, 2049. Explain repatriation facilities provided to the foreign
investors under the Act.
c) Explain the provisions relating to 'specific performance' of a contract under the
Contract Act, 2056.
d) Explain "Sick Industry" under Industrial Enterprises Act 2073.
e) Write notes on Special Program relating to Social Welfare as mentioned in the Social
Welfare Act, 2049.

Answer
a) As per section 56 of the Negotiable Instrument Act 2034; the drawer, acceptor or
endorser of a Negotiable Instrument shall be discharged from his/her liability in the
following conditions:-
I. If the Holder intentionally cancels the name of the acceptor or endorser with
intent to discharge him/her to such Holder and all parties claiming through such
Holder.
II. If a Holder thereof who otherwise discharges such acceptor or endorser or Drawer
and to all parties deriving title under such Holder after notice of such discharge.

The Institute of Chartered Accountants of Nepal 43


Suggested Answer - June 2018

III. If the Negotiable Instrument is payable to bearer or has beenindorsed in blank and
such Drawer, acceptor or endorser makes Payment in due Course of the amount
due thereon is discharged to all concerned parties thereto related with such
Negotiable Instrument.

b) Section 2(1) of the Foreign Investment and Technology Transfer Act, 2049 has given
the definition of the term foreign investment. It defines- the term foreign investment
means the following investment made by any foreign investor in any industry of
Nepal:
a. Investment in share capital.
b. Re-investment of income received from the investment in share capital.
c. Investment as a loan or loan facilities.
There are numbers of facilities given by the Act to attract the foreign investment in
Nepal. Pursuant to section 5 of the Act the following facilities are provided to the
foreign investors:
The foreign investors can repatriate the following amount in the foreign currencies
from Nepal:
a) Amount received from sale of whole or some part of share.
b) Amount received as profit or dividend of foreign investment.
c) Amount received as interest of loan or the payment of interest.
d) Amount received under the technology transfer agreement.

c) Section 86 of the Contract Act, 2056 provides for ‗Specific Performance‘. Accordingly
in case the cash compensation paid in consideration of the actual loss or damage
suffered by the aggrieved party as a result of breach of contract is not reasonable or
adequate, the aggrieved party may demand the execution of the contract as stipulated
specific performance instead of making a claim for compensation.
Notwithstanding anything contained above, no claims for execution of the contract as
stipulated specific performance shall be heard in any of the following circumstances;
i. In case the amount paid in cash as compensation for breach of contract is
adequate;
ii. In case the court cannot supervise whether or not the work to be performed under
the contract has been actually performed;
iii. In case the contract has been signed for providing services relating to personal
expertise, skill or knowledge;
iv. In case the situation is -such that the contract cannot be executed as stipulated;
v. In case the party violating the contract him/herself demands that the contract be
executed as stipulated.

d) Section 37 of Industrial Enterprises Act 2073, prescribes the provision about the
Sick Industry.
i. if any industry is being operated at least five years and in loss for a consecutive
period of last 3 years due to operate in thirty percent or below of the total
production capacity due to condition out of control and not being due to

The Institute of Chartered Accountants of Nepal 44


CAP II Paper 3: Corporate & Other Law

negligence or weakness of management, following the procedure mentioned


under prescribed guidelines, Government of Nepal may identify as sick industry.
ii. On the basis of contribution made by the industry before being sick industry on
generation of employment, import substitution or earn foreign exchange by export
promotion and can be re-operated if provided fixed concession, facility or rebate,
Nepal Government may take necessary action to restructure, reformation and
management of such industry.

e) As per the section 4 of the Social Welfare Act, 2049; Government of Nepal may
operate special Programs, relating to the social welfare activity and social service,
in the following matters:
(a) To serve interest and render welfare to the children, oldage, helpless or disabled
people.
(b) To foster participation in development and to promote and protect the welfare,
rights and interest of the women.
(c) To rehabilitate and help to lead a life of dignity to the victims of social mischief's
and also to juvenile delinquency, drug addicts and similar people involved in other
kind of addictions.
(d) To help to lead a life with dignity to the jobless, poor and illiterate people.
(e) To manage religious places and the activities of the trust Guthi institutions.
(f) To take effective management and actions for the welfare of the backward
communities and classes.

7. Write short notes on the following: (2×5=10)


a) Formation of the World Trade Organization (WTO) and its objectives.
b) List out the various leave and holidays facilities prescribed by the Labour Act, 2074.
Answer
a) It was officially constituted on January 1, 1995 which took the place of GATT as an
effective formal, organization. Contrary to the temporary nature of GATT, WTO is a
permanent organization which has been established on the basis of Multilateral
Agreements approved by participating countries. The WTO has nearly 164 members
accounting for over 98% of world trade. Around 23 others are negotiating membership.
Decisions are made by the entire membership. This is typically by consensus. The
WTO‘s top level decision-making body is the Ministerial Conference which meets once
in very two years.

The WTO secretariat, based in Geneva, has around 600 staff and is headed by a
Director-General. Its annual budget is roughly 160 million Swiss Francs. It does not
have branch offices outside Geneva. Since decisions are taken by the members
themselves, the secretariat does not have the decision making the role that other
international bureaucracies are given. The secretariat's main duties are to supply
technical support for the various councils and committees and the ministerial
conferences, to provide technical assistance for developing countries, to analyze world
trade and to explain WTO affairs to the public and media.

The Institute of Chartered Accountants of Nepal 45


Suggested Answer - June 2018

The important objectives of WTO are:


1. To improve the standard of living of people in the member countries.
2. To ensure full employment and broad increase in effective demand.
3. To enlarge production and trade of goods.
4. To increase the trade of services.
5. To ensure optimum utilization of world resources.
6. To protect the environment.
7. To accept the concept of sustainable development.

b) Labour Act, 2074 under sections 40 to 48 has provided the leave and holidays facilities
as follows.
 Weekly holiday- 1 day every week
 Public Holiday- 13 days including May Day and additional 1 day to female
employees including International Women's Day.
 Home Leave- 1 day for every 20 worked days.
 Sick Leave- up to 12 days fully paid annually.
 Mourning Leave- 13 days in case of death of persons.
 Leave in lieu- For the laborers put in work on public holiday or weekly off will be
provided accordingly.
 Maternity Leave- 98 days. Fully paid up to 60 days either before or after the
delivery.
 Paternity Leave- 15 days. Fully paid.

The Institute of Chartered Accountants of Nepal 46


CAP II Paper 3: Corporate & Other Law

Specific Comments on the performance of the students


Batch: - June 2018
Level: - CAP-II
Subject: Corporate & Other Laws
Question No. 1
Case laws and practical questions are satisfactorily solved by the students. Most of the
students have not suggested or gave answer as it is required to answer. It shows the little
preparation and not understanding of Company law. Most of the students could not answer of
the question 1(c) could not write general concept of Sections 111 and 119 of Company Act,
2063. Most students found confusion whether moral turpitude tall in offense or not.

Question No. 2
Most of the students have not answered the questions relating to the Banking laws. This
question requires the NRB and BAFIA's legal provision. However, they submit their answer
with their logic. Most of the students have written mode of the distribution of bonus which is
not relevant part of the answer.

Question No. 3
Students have not answered the questions in well manner. They are lack of knowledge of
relevant legal provisions. Students focused on benefit of compensation fund than its
operation process.

Question No. 4
Most of the students have answered this question in well manner. Few students were in
confusion about criteria of bonus distribution.
Question No. 5
Students are confused to Section 34 and 41 of NCA Act, 2053. They have mentioned 31
instead of 41 or vice-versa.
Question No. 6
Answer to 6 (a) and 6 (c) is less than standard as they had no idea of the concerned laws.
Their answers are not well prepared in other cases too. Holder of the Negotiable Instrument –
concept, meaning is not clear in answer. Most of the students have confusion regarding
holder-drawer-acceptor and endorse.

Question No. 7
Students have only common knowledge about the questions. Few students did not answer the
holidays stating the particular days.

The Institute of Chartered Accountants of Nepal 47


Suggested Answer - June 2018

Paper 4:

Financial Management

The Institute of Chartered Accountants of Nepal 48


CAP II Paper 4: Financial Management

Roll No……………. Maximum Marks - 100


Total No. of Questions – 7 Total No. of Printed Pages –15
Time Allowed – 3 Hours
Marks
Attempt all questions.
Working notes should form part of the answer. Make assumptions wherever
necessary.
1. After extensive research and development, Goodweek Tires Ltd., has recently developed a
new tire, the Super Tread, and must decide whether to make the investment necessary to
produce and market it. The tire would be ideal for drivers doing a large amount of wet
weather and off-road driving in addition to normal freeway usage. The research and
development costs so far have totaled about Rs. 10 million. The Super Tread would be put
on the market beginning this year, and Goodweek expects it to stay on the market for a
total of four years. Test marketing costing Rs. 5 million has shown that there is a significant
market for a Super Tread-type tire. Goodweek must initially invest Rs. 120 million in
production equipment to make the Super Tread. This equipment can be sold for Rs. 51
million at the end of four years. Goodweek intends to sell the Super Tread to two distinct
markets:
a. The original equipment manufacturer (OEM) market: The OEM market consists
primarily of the large automobile companies that buy tires for new cars. In the OEM
market, the Super Tread is expected to sell for Rs. 3,600 per tire when introduced. The
variable cost to produce each tire is Rs. 1,800 in first year of production.
b. The replacement market: The replacement market consists of all tires purchased after the
automobile has left the factory. This market allows higher margins; Goodweek expects
to sell the Super Tread for Rs. 5,900 per tire there. Variable costs are the same as in the
OEM market.
Goodweek intends to raise prices of its product in both markets at 5 percent every year as it
expects the same increase in variable costs. In addition, the Super Tread project will incur
Rs. 25 million in marketing and general administration costs during the first year. This cost
is expected to increase at 4 percent in the subsequent years.
Goodweek's corporate tax rate is 25 percent. The company uses a 16 percent discount rate
to evaluate new product decisions. Automotive industry analysts expect automobile
manufacturers to produce 20,000 new cars this year and production to grow at 2.5 percent
per year thereafter. Each new car needs four tires (the spare tires are undersized and are in a
different category). Goodweek expects the Super Tread to capture 11 percent of the OEM
market. Industry analysts estimate that the replacement tire market size will be 140,000
tires this year and that it will grow at 2 percent annually. Goodweek expects the Super Tread
to capture an 8 percent of this market share.
The appropriate depreciation schedule is as per SLM. The immediate initial working capital
requirement is Rs. 11 million. Thereafter, the net working capital requirements will be 15
percent of next year's sales. Except for the initial investment that will occur immediately,
assume all cash flows will occur at year-end.
Required: 20

The Institute of Chartered Accountants of Nepal 49


Suggested Answer - June 2018

Based on net present value analysis, recommend whether investing in the project is worth
taking.
Answer:

The Research & Development Cost and Test Marketing Cost incurred so far are sunk for
decision making.
A. Calculation of Initial Out Lay (Rs.)
Cost of Equipment 120,000,000
Initial W/C Infusion 11,000,000
131,000,000

B. Calculation of Annual Depreciation (Rs.)


Cost of the Machine 120,000,000
Estimated Salvage value 51,000,000
Depreciable Value 69,000,000
Useful Life 4 Yrs
Annual Depreciation 17,250,000

C. Calculation of Sales Volume (Rs.)


1 2 3 4
OEM Market
Total Market Size for the Car 20,000 20,500 21,013 21,538
Total Number of Tires Requirement 80,000 82,000 84,052 86,152
Share of Goodweek (A) (11%) 8,800 9,020 9,246 9,477
Replacement Market
Total Market of Replacement Tires 140,000 142,800 145,656 148,569
Share of Goodweek (B) (8%) 11,200 11,424 11,652 11,886
Total Size of Sales Volume (A+B) 20,000 20,444 20,898 21,363

D. Sales Piece Per Unit and Annual Sales Value (Rs.)


1 2 3 4
OEM Market 3600 3,780 3,969 4,168
Replacement Market 5900 6,195 6,505 6,830
36,697,37
Sales Value OEM Market 31,680,000 34,095,600 4 39,500,136
75,796,26
Sales Value Replacement Market 66,080,000 70,771,680 0 81,181,380
112,493,6
Total Sales Value 97,760,000 10,48,67,280 34 120,681,516

The Institute of Chartered Accountants of Nepal 50


CAP II Paper 4: Financial Management

E. Variable Cost Per Unit and Annual Total Variable Cost (Rs.)
1 2 3 4
Variable Cost for all market 1,800 1,890 1,985 2,085
Total Variable Cost 36,000,000 38,639,160 41,482,530 44,541,855

F. Estimation of Further Working Capital Requirement (Rs.)


Initial 1 2 3 4
Initial W/C
Requirement 11,000,000
W/C need at year end 15,730,092 16,874,045 18,102,227
Additional Infusion 4,730,092 1,143,953 1,228,182
Release of W/C 18,102,227

G. Terminal Cash Flows


Net Proceed from the sale (Rs.)
Cash Salvage Value 51,000,000
Book Salvage Value 51,000,000
Gain/Loss -
Tax on Gain/Loss -
Net proceed from Sale 51,000,000
Release of W/C 18,102,227
Total Terminal Flow 69,102,227
(Rs.)
H. Annual CFAT Y1 Y2 Y3 Y4
Sales 97,760,000 104,867,280 112,493,634 120,681,516
Variable Cost 36,000,000 38,639,160 41,482,530 44,541,855
Marketing & Administration
Cost 25,000,000 26,000,000 27,040,000 28,121,600
Depreciation 17,250,000 17,250,000 17,250,000 17,250,000
EBT 19,510,000 22,978,120 267,21,104 30,768,061
Tax@ 25% 4,877,500 5,744,530 6,680,276 7,692,015
EAT 14,632,500 17,233,590 20,040,828 23,076,046
Add: Depreciation 17,250,000 17,250,000 17,250,000 17,250,000
CFAT 31,882,500 34,483,590 37,290,828 40,326,046
Working Capital Adjustment (4,730,092) (1,143,953) (1,228,182) -
CFAT after WC Adjustment 27,152,408 33,339,637 36,062,646 40,326,046

I. Calculation of NPV (Rs.)

The Institute of Chartered Accountants of Nepal 51


Suggested Answer - June 2018

PVF
Particulars Time Cash Flow @16% Total PV
Initial Outlay 0 (131,000,000) 1.00 (131,000,000)
Recurring Cash Flow 1 27,152,408 0.8621 23,408,091
2 33,339,637 0.7432 24,778,018
3 36,062,646 0.6407 23,105,337
4 40,326,046 0.5523 22,272,075
Terminal Cash Flow 4 69,102,227 0.5523 38,165,160
Net Present Value 728,681

Since the NPV of the Project is positive, it should be implemented.

2.
a) AB & Co. has applied for working capital finance from a commercial bank. The
following is the firm‘s projected profit and loss account:-
Particulars Amount (Rs.)
Sales 2,247,000
Cost of goods sold 1,637,100
Gross profit 609,900
Administrative expenses, 149,800
Selling expenses 139,100 288,900
Profit before tax 321,000
Tax provision 107,000
Profit after tax 214,000
Total cost of goods sold (COGS) is calculated as follows:
Particulars Amount (Rs.)
Material used 898,800
Wages and other manufacturing expenses 668,750
Depreciation 251,450
1,819,000
Less: Stock of finished goods (10% product not yet sold) 181,900
Cost of goods sold 1,637,100
 The figures given above relate only to the goods that have been finished, and not
to work in progress.
 Goods equal to 15 percent of the year‘s production (in terms of physical units)
are in progress on an average requiring full material but only 40 percent of other
expenses.
 The firm has a policy of keeping two months‘ consumption of material in stock.
 All expenses are paid one month in arrears.

The Institute of Chartered Accountants of Nepal 52


CAP II Paper 4: Financial Management

 Suppliers of material grant one and half months‘ credit;


 Sales are 20 percent cash while remaining sold on two months‘ credit.
 70 percent of the income tax has to be paid in advance in quarterly installments.
Required: 10
Prepare an estimate of the working capital requirements of the firm on cash cost
basis. You may add 10 percent to your estimated figure to account for exigencies.
b) The Chitwan Krishi Company is planning to relax its credit policy to motivate
customers to buy on new credit terms. It is expected that the variable cost will remain
75 percent of sales. The incremental sales are expected to be sold on credit. For the
perceived increase in risk of liberalizing the credit terms, the company requires higher
return. The following table shows the projected information:
Credit Policy Required Return on Collection New Sales (Rs.)
investment Period(days)
A 20% 40 300,000
B on
25% 45 400,000
C 32% 55 500,000
D 40% 70 600,000
Required: 5
Which credit policy should the company pursue? Assume 360 days in a year.

Answers:
a)
STATEMENT SHOWING THE REQUIREMENTS OF WORKING
CAPITAL (ON CASH COST BASIS)
Particulars Rs.
A. Current Assets:
Stock of raw material (898,800+1,34,820)×2/12 172,270
Stock of Work in Progress WN II 174,945
Stock of Finished Goods WN III 156,755
Debtors 1,359,756 x 2/12 226,626
Total Current Assets 730,796
B. Current Liabilities
Creditors for Raw Materials 1,205,890 x 1.5/12 150,736
Creditors for Wages 708,875 x 1/12 59,073
Creditors for Office and Adm. Expenses 149,800 x 1/12 12,483
Creditors for Selling & Distribution 139,100 x 1/12 11,592
Expenses
Provision for Taxation 107,000 x 30/100 32,100
Total Current Liabilities 265,984
C. Net Working Capital A-B 464,612
D. Add: Safety Margin 464,612 x 10/100 46,461

The Institute of Chartered Accountants of Nepal 53


Suggested Answer - June 2018

E. Required Working Capital C + D 511,073


Working Notes:
I) Calculation of raw material consumed:
Material used in Finished goods = 898,800
Materials used in WIP = 15% ×898,800 = 134,820
Raw material consumed = 898,800 + 134,820 = 1,033,620
II) CALCULATION OF STOCK OF WORK-IN-PROGRESS
Particulars Rs.
Raw Material (898,800 x 15%) 1,34,820
Wages & Manufacturing Expenses (668,750 x 0.15 x 0.40) 40,125
Total 174,945
III) CALCULATION OF STOCK OF FINISHED GOODS AND COST OF SALES
Particulars Rs.
Direct Material Cost (898,800+134820) 1,033,620
Wages & Manufacturing Expenses (668,750+40,125) 708,875
Gross factory Cost 1,742,495
Less: Closing WIP (As per WN II) 174,945
Cost of Goods Produced 1,567,550
Less: Closing Stock (10% x 1,567,550) (156,755)
Cost of goods sold 1,410,795
Add: Office & Adm. Expenses 149,800
Add : Selling and Distribution Expenses 139,100
Total Cash cost of sales 1,699,695
Total Cash cost of Credit Sales (80% of 1,699,695) 1,359,756
OR: WN III
Valuation of Finished Goods
Raw material = 898,800×10% = 89,880
Wages and manufacturing exp.= 6,68,750×10% = 66,875
1,56,755
WN IV: Computation. Of Debtors
Debtors on sales value = ( 22,47,000×80%)×2/12 = 2,99,600
Component cost of debtors
Raw material (8,98,800 - 89,880)/22,47,000 = 36%
Wages & mgmf (6,68,750 - 66,875)/22,47,000 = 26.78%
Adm and selling exp. (1,49,800 + 1,39,100)/22,47,000 = 12.86%
Total cash cost % of Debtors = 75.6428%
Cash cost of Debtors = 2,99,600×75.6428 = 2,26,626

The Institute of Chartered Accountants of Nepal 54


CAP II Paper 4: Financial Management

IV) CALCULATION OF CREDIT PURCHASES & AMOUNT OF CREDITORS


Particulars Rs.
A. Raw Material Consumed 1,033,620
B. Add: Closing Stock 172,270
C. Opening Stock Nil
D. Purchases (A+B-C) 1,205,890
E. Creditors ( 1,205,890×1.5/12) 150,736

b)
Statement showing the Evaluation of Debtors Management Policies
Particulars Proposed Proposed Proposed Proposed
Policy A Policy B Policy C Policy D

A. Expected Profits
a) Credit Sales 300,000 400,000 500,000 600,000
b) Total Cost
i) Variable Cost 225,000 300,000 375,000 450,000
c) Expected profit (a)-(b) 75,000 100,000 125,000 150,000
B. Opportunity Cost of Investment in 5000 9375 18,333 35,000
Receivables (WN)
Net Benefits (a)-(b) 70,000 90,625 106,667 115,000
Recommendation: Policy D should be adopted since the net benefits under this policy
is higher than those under other policies.
Working Note: Calculation of opportunity cost of Investments in receivables.(Rs.)
Opportunity Cost = Total Cost × Collection Period/360 × Rate of Return/100
Policy A = 225,000×40/360×20% = 5,000
Policy B = 300,000×45/360×25% = 9,375
Policy C = 375,000×55/360×32% = 18,333
Policy D = 450,000×70/360×40% = 35,000

3.
a) Northern California Fruit Company‘s latest earnings are Rs. 2 per share. Earnings per
share are expected to grow at a 20 percent compounded annually for 4 years, at a 12
percent annually for the next 4 years and at 6 percent thereafter. The dividend-pay-out
ratio is expected to be 25 percent for the first 4 years, 40 percent for the next 4 years
and 50 percent thereafter. At the end of year 8, the price -earnings ratio for the
company is expected to be 8.5 times, where year 9‘s expected earnings per share are
used in the denominator.
Required: (4+4=8)
i) If the required rate of return is 14 present, what is the present
market price per share?

The Institute of Chartered Accountants of Nepal 55


Suggested Answer - June 2018

ii) If the present market price per share is Rs. 30, what is the stocks
expected return?
b) Following are the ratios of trading activities of East West Ltd. :-
Average collection Period 3 months Opening trade receivables Rs. 600,000
Inventory turnover ratio 1.5 times Gross profit Rs. 800,000
Average payment period 2 months Opening inventory Rs. 1,590,000
Gross profit ratio 25% Closing bills receivable Rs. 50,000
Closing bills payable Rs. 20,000
All sales are made in credit.
Required: (1+3+3=7)
i) Calculate revenue from operations
ii) Calculate closing trade debtors
iii) Calculate closing inventory
Answers:
a)
Growth Earning(Rs.) DP Ratio Dividend(Rs.)
Year 0 - 2.00 - -
Year 1 20% 2.40 25% 0.60
Year 2 20% 2.88 25% 0.72
Year 3 20% 3.46 25% 0.86
Year 4 20% 4.15 25% 1.04
Year 5 12% 4.65 40% 1.86
Year 6 12% 5.21 40% 2.08
Year 7 12% 5.84 40% 2.34
Year 8 12% 6.54 40% 2.62
Year 9 6% 6.93 50% 3.47

B. Price at the End of Eight Year


Given,
P8/E9 = 8.5 Times
P8 = 8.5 * E9
P8 = 58.90
Therefore, price per share at the end of year 8 ( P8) will be = Rs. 58.90
C. Calculation of Current Market Price Per Share

The Institute of Chartered Accountants of Nepal 56


CAP II Paper 4: Financial Management

2.34 2.62 58.90

Rs 26.72 / Share
ii) Calculation of Stock‘s Expected Return on Market Price of Rs. 30 Per Share
2.34

2.62 58.90

The expected rate of return needs to be calculated using interpolation technique.


Therefore we need to use Hit and Trial Method at different rate.
Try at 10 % of Discount Rate

2.34 2.62 58.90

P0 = Rs. 34.87
As given market price of Rs. 30 lies in between the price per share of Rs. 32.76 and
Rs. 25.25 calculated using the discount rate at 10% and 14% respectively; therefore
value can be interpolated in between 10% and 14%.
Through interpolation
Expected Rate of Return = LR X [HR-LR]
= 10% + 34.87 - 30 X [14%-10%]
34.87 - 26.72
= 10 + 4.87/8.15
= 12.39%
Therefore the expected return at current market Price of NRs 30 is
11.47%.
b) i) Revenue from operations

Gross Profit Ratio = Gross Profit * 100/Revenue from operation


25 = 800,000*100/ Revenue from operation
= Rs. 3,200,000

ii) Trade debtors

Average Colletion Period = 12/Debtor turnover ratio


3 months = 12/Debtor turnover ratio
So, Debtor turnover ratio = 4 times
Debtor turnover ratio = Credit Revenue/ Average trade receivables

The Institute of Chartered Accountants of Nepal 57


Suggested Answer - June 2018

4 = 3200,000/ Average trade Receivables


Average trade receivables = Rs. 800,000
(Opening + Closing Trade receivables)/2 = Rs. 800,000
600,000+Closing Receivables = 1600,000
Closing trade receivables = Rs. 1000,000
Closing trade debtors = Rs. 1000,000 – Bills receivables
= Rs. 950,000

iii) Closing inventory:


Cost of goods sold = Revenue – Gross Profit
= 3200,000-800,000
= Rs. 2,400,000
Inventory turn over ratio = Cost of Goods Sold/Average inventory
1.5 = Rs. 2,400,000/Average Inventory
Average inventory = Rs. 1,600,000
(Opening +Closing inventory)/2 = Rs. 1,600,000
Closing inventory = Rs. 1,610,000
4.
a) A company earns profit of Rs. 300,000 per annum after meeting its interest liability of
Rs. 120,000 on 12% debentures. The numbers of equity shares of Rs. 10 each are
80,000 and retained earnings amount to Rs. 1,200,000. The company proposes to take
up an expansion scheme for which a sum of Rs. 400,000 is required. It is anticipated
that, after expansion, the company will be able to achieve the same return on
investment as at present. The funds required for expansion can be raised either
through debt at the rate of 12% or by issuing equity shares at par. The tax rate is 25%.
Required: (7+1=8)
i) Compute the earnings per share, if:
 The additional funds were raised as debt.
 The additional funds were raised by issue of equity shares.
ii) Advise the company as to which source of finance is preferable.

b) The following is the capital structure of Simons Company Ltd. as on 31st Ashadh
2074:
Rs.
Equity shares (of Rs. 100 each) 1,000,000
10% Preference Shares (of Rs. 100 each) 400,000
12% Debentures 600,000
2,000,000
The market price of the company`s share is Rs. 110 and it is expected that a dividend
of Rs. 10 per share would be declared after 1 year. The dividend growth rate is 6%.
The company is in the 25% tax bracket.
Required: (3+4=7)
i) Compute the weighted average cost of capital.

The Institute of Chartered Accountants of Nepal 58


CAP II Paper 4: Financial Management

ii) Assuming that in order to finance an expansion plan, the company intends to
borrow a fund of Rs. 1 million bearing 14% rate of interest, what will be the
company`s revised weighted average cost of capital? This financing decision is
expected to increase dividend from Rs. 10 to Rs. 12 per share. However, the
market price of equity share is expected to decline from Rs. 110 to Rs. 105 per
share.
Answers:
a)

i)
Capital Employed Before expansion plan Rs.
Equity Shares 800,000
Debentures (Rs. 1,20,000/12)ˣ100 1,000,000
Retained Earnings 1,200,000
Total Capital Employed 3,000,000
Earnings Before the payment of Interest and Tax (EBIT)
Rs.
Profit 300,000
Interest 120,000
EBIT 420,000

Return on Investment (ROI)


ROI = (EBIT×100)/Capital Employed
= Rs. 420,000ˣ100/Rs. 3,000,000
=14%
Earnings before Interest and Tax (EBIT) After Expansion
Capital Employed after expansion = Rs. 3,400,000
(3,000,000+400,000)
Desired EBIT = 3,400,000 ˣ 14/100 = Rs. 476,000
Statement showing EPS under Present and Anticipated Expansion
Scheme
Expansion Scheme
Particulars Present Situation Additional Funds raised as
Debt Equity
EBIT (i) 420,000 476,000 476,000
Interest - Old Debt 120,000 120,000 120,000
Interest - New Debt 0 48,000 0
Total Interest (ii) 120,000 168,000 120,000
EBT (i)-(ii) 300,000 308,000 356,000
Less: Tax @ 25% 75,000 77,000 89,000
EAT 225,000 231,000 267,000
No. of Equity Shares 80,000 80,000 120,000
EPS 2.81 2.89 2.23

The Institute of Chartered Accountants of Nepal 59


Suggested Answer - June 2018

ii) If the company raises additional funds as debt the EPS would be
greater. Hence, it is suggested to raise additional funds in the form of
debt.
b) (i) Computation of the Weighted Average Cost of Capital
Source Weight(W) C/C W*C/C
Equity share 0.5 15.09% 7.55%
10% Preference share 0.2 10% 2.00%
12% Debentures 0.3 9% 2.7%
Weighted Average Cost of Capital 12.25%
ii) Computation of Revised Weighted Average Cost of Capital
Source Weight(W) C/C W*C/C
Equity share 0.333 17.43% 5.81%
10% Preference share 0.133 10% 1.33%
12% Debentures 0.200 9% 1.80%
14% Loan 0.333 10.5% 3.5%
Weighted Average Cost of Capital 12.44%
Working Notes:
1) Cost of equity shares (Ke) at present,
D1
Ke = --------------- + g = 10/110+0.06= 0.1509= 15.09%
P0
2) Revised Cost of Equity shares (Ke),
Ke = 12/105+0.06= 17.43
5.
a) Horizon Enterprises is a manufacturer and exporter of woolen garments to most of the
European countries. Its business is expanding day by day, and in the previous
financial year the company has registered a 25 percent growth in export business.
The company is in the process of considering a new investment project. It is an all
equity financed company with 1,000,000 equity shares of Rs. 50 per share. The
current issue price of this share is Rs. 125 ex-dividend. Annual earnings of Rs. 25 per
share, in the absence of new investments, will remain constant in perpetuity. All
earnings are distributed at present. A new investment is available which will cost Rs.
17,500,000 in one year's time and will produce annual cash inflows of Rs. 5,000,000
thereafter. The new investment is financed through internal accruals.
Required: 5
Analyze the effect of the new investment on dividend payments and the share price.
b) Ms. Smile currently holds two equity shares X and Y in equal proportion with the
following risk and return characteristics:
Return (RX) 24% Return (RY) 19%
σX 28% σY 23%
The returns of these securities have a positive correlation of 0.6.
Required: (3+2=5)

The Institute of Chartered Accountants of Nepal 60


CAP II Paper 4: Financial Management

i) Calculate the portfolio return and risk.


ii) How much should the correlation coefficient be to bring the portfolio risk to her
desired 15% level?
c) An analyst working with McKinsey, Singapore is trying to figure out the cost of capital
for a Hydro Power Project in Nepal for his client based at USA. As the proposed
investment is cross-border, and country risk is significantly high for Nepal in
comparison to USA, he wants to include a country risk premium in his estimate of the
cost of equity for the project. The analyst has compiled the following information for
his analysis;
 Nepali US dollar denominated 10-Year Government Bond Yield = 8.8%
 10 Year US Treasury Bond yield = 5%
 Annualized Standard Deviation of NEPSE= 32%
 Annualized Standard Deviation of Nepali US Dollar denominated 10-Year
Government Bond= 18%
 Beta = 1.25
 Expected Market Return= 10.4%
 Risk Free Rate = 4.2%
Required: 5
Calculate the Country Risk Premium and the Cost of Equity for the Hydro Power
Project in Nepal.
Answers:
a)
Calculation of Cost of Equity (ke)
Ke = D/P ˣ 100
= 25/125 ˣ 100
=20%
Earnings per share = Rs. 25
Total Earnings = 1,000,000 shares @ Rs. 25 each =Rs. 25,000,000
New project cost = Rs. 17,500,000
Dividend per share in 1st year
Since the investment is financed out of internal accruals, the amount available for
dividend at the end of first year is Rs:
=1st Year's earnings - Project cost
=Rs. 25,000,000-Rs. 17,500,000
=Rs. 7,500,000
Since all the earnings are distributed, earnings per share and dividend per share are
same.
Dividend per share in 1st year = Rs. 7,500,000/1,000,000 shares = Rs. 7.50
Dividend per share in 2nd year and will remain constant in perpetuity
=Rs 25,000,000+Rs. 5,000,000
1,000,000 shares
=Rs. 30 per share
The present value of new share price after the new investment is taken up

The Institute of Chartered Accountants of Nepal 61


Suggested Answer - June 2018

P = (Rs. 7.5/1.2) +(Rs 30/0.2 ˣ 1/1.2)


= 6.25+125
= Rs. 131.25
It is evident that the dividend payment and the price of share will increase
after new investment.

b) i) The portfolio return and risk are as under:


Portfolio Return [E(RP)] = RX × ProportionX + RY × ProportionY
= 24% × 50% + 19% × 50%
= 12% + 9.5%
= 21.5%


Portfolio risk [σP] = ơX2× ProportionX2 +
2 2
ơY ×ProportionY + 2 × ơX ×
ProportionX × ơY × ProportionY ×
CorrelationXY

= 282 × 0.502 + 232 × 0.502 + 2 × 28 × 0.50 × 23 ×


0.50 × 0.6 √
= √ 521.45 = 22.84%
ii) If Ms Smile desires the portfolio standard deviation to remain at 15%, then
correlation of equity shares X and Y shall be -0.321 as below:
152 = 282 × 0.502 + 232 × 0.502 + 2 × 28 × 0.50 × 23 × 0.50 × CorXY
225 = 328.25 + 322 CorXY
CorXY = (225 - 328.25)/322
= - 0.321
c)
i. Calculation of Country Risk Premium
Country Risk Premium (CRP) = (Yield on Nepali bond - Yield on US Bond)×(SD of NEPSE/SD
of Nepali bond)
=[0.088-0.05]*[0.32/0.18] = 0.06756 = 6.76%

ii. Cost of Equity


KCE = RF + ß [RM – RF + CRP]
= 0.042+1.25[0.104-0.042+0.06756]
= 20.4%

6. Write short note/ answer on: (4×2.5=10)


a) Assumptions of economic lot size technique
b) Cost and benefits of factoring
c) NEPSE Index
d) Venture capital

The Institute of Chartered Accountants of Nepal 62


CAP II Paper 4: Financial Management

Answers:
a) Following are the assumptions of economic lot size technique:
 Constant Annual requirement of Cash
 Constant rate of demand of cash
 Constant Transaction costs
 Constant holding costs, and
 Zero conversion period
b) The cost of factoring includes:
 Factoring commission
 Interest charged by factor on advance granted
The benefits of factoring includes:
 Saving in costs of in house credit collection department
 Saving in bad debt losses
 Saving in cost of funds invested in receivables due to reduction in average
collection period
 Saving in cash discount allowed (if any)
c) The NEPSE Index is a value weighted index of all shares listed at the Nepal Stock
Exchange and calculated on a daily basis (for the days market remain open) at the
closing price. The calculation of the NEPSE index is based on the concept of the
market capitalization which is the sum of the market capitalization of all the company
listed in the Nepal Stock Exchange. If the ratio of current period market capitalization
to the base period market capitalization is multiplied by the multiplier 100, we get
NEPSE index. This method of index calculation is called value weighted method.
Total Market Capitalization of all the Companies Listed
NEPSE Index= -------------------------------------------------------------------------------* 100
Total Base Year`s Market Capitalization

However in reality, the number of the listed companies keeps on changing, and the
number of the outstanding shares also keeps on changing as the company issues right
shares or bonus shares or common shares at the time of capital needs. The actual
practice to adjust the base period is as follows:
Adjusted Base Period =(New Market Capitalization including new listing/New
Market Capitalizations excluding new listing )* Base Year's Market Capitalization.
d) Venture Capital is long term equity investments in such business which has the
potentiality for significant growth and high return on capital. The main futures of
venture capital investment are:
(i) Providing finance to entrepreneurial talents;
(ii) Providing capital to persons having management skills;
(iii) Expecting a high return in the form of capital gain.
The venture capital schemes are designed to promote technological advancement and
innovation through introduction of new products, process or plants and equipment's.
The activities, which are in general needs venture capital support, are:
 Commercial production of viable new process products
 Technological up gradation including adoption of imported technology
suitable to local conditions.

The Institute of Chartered Accountants of Nepal 63


Suggested Answer - June 2018

 Energy conservation with innovative technology.


 Commercial exploitation of proven technology.
Thus venture capital involves in risky ventures in technology development and long
gestation technology development projects. Venture capital normally enters at the
different stages in the projects viz.
 Early Stage
Seed Capital is provided to entrepreneur for concept formulation or start-up capital
for commercial exploitation of proven product.
 Expansion Financing
Finance is provided either for major expansion of the commercial production or for
working capital support. The firm of investment takes many shapes. The most
common forms are equity purchase, conditional loans, income notes and participation
debentures.
7. Distinguish between: (4×2.5=10)
a) NPV and IRR
b) Horizontal analysis and Vertical analysis
c) Accounting break-even and Financial break-even
d) Digital money and Crypto currencies
Answers:
a) Evaluation of Non-conventional investments-Both are non-equivalent as regards the
acceptance/rejection of non-conventional investments if the projects differ in their (a)
expected lives or (b) estimated cash outflows or (c) timings of cash flows.
Reinvestment Rates-IRR assumes that intermediate cash inflows are reinvested at
IRR while NPV assumes that intermediate cash inflows are reinvested at required rate
of return (i.e. firm‘s cost of capital).
Multiple/Negative Rates-IRR can yield negative rates/multiple rates under certain
circumstances while there is no such possibility under NPV method.
b) Horizontal analysis: This technique is also known as comparative analysis. It is
conducted by setting consecutive balance sheet, income statement of statement of
cash flow side by side and reviewing changes in individual categories on year-to-year
or multi year basis. The most important item revealed by comparative financial
statement analysis is trend. The horizontal financial statements analysis is done by
restating amount of each item or group of items as a percentage.
Vertical analysis: Vertical/ Cross-sectional/ Common size statements came from the
problems in comparing the financial statements of firms that differ in size. The
vertical analysis represents the relationship of different items of financial statements
with some common item by expressing each item as a percentage of the common
item. For example:
 In the balance sheet, the assets as well as the liabilities and equity are each
expressed as a 100% and each item in these categories is expressed as a
percentage of the respective totals.
 In the common size income statement, turnover is expressed as 100% and every
item in the income statement is expressed as a percentage of turn over.
c) Accounting break-even method is the most common form of the analysis done and
one of the easiest. It is calculated as being the number of units that need to be sold in

The Institute of Chartered Accountants of Nepal 64


CAP II Paper 4: Financial Management

order to produce zero profit. More formally, the number of units required can be
calculated as total fixed cost divided by the difference between unit price and variable
cost. The difference between unit price and variable cost can be considered the profit
per unit produced and sold and a business must sell enough units to cover its fixed
costs before it can become profitable.
Accounting BEP
Financial break-even is a similar concept to accounting break-even but uses very
different measurements. It is the level of earnings needed before a firm's earnings per
share is equal to zero. Here, earnings is defined as earnings before interest and taxes,
or gross profit minus cost of sales and operating expenses and earnings per share is
most often defined as being earnings divided by the number of outstanding common
shares.

Financial BEP
d) Digital money exists only in the digital form. It doesn‘t have any physical equivalent
in the real world. Nevertheless, it has all the characteristics of traditional money.
Digital currencies don‘t have geographical or political borders; transactions might be
sent from any place and received at any point in the world. Actually, digital accounts
and digital wallets may be regarded as bank deposits.
Whereas, cryptocurrency is an asset used as a means of exchanging. It is considered
reliable because it‘s based on cryptography. It creates and analyzes the algorithms
and protocols so no information is changed or interrupted during the conversation by
third parties. Cryptocurrencies use Blockchain and a decentralized ledger. It means
that no supervisory authority controls all the actions in the network. This comes at
the expanse of all the users.
Though cryptocurrency is a type of digital currency, there are some fundamental
differences.
Structure: Digital currencies are centralized; there is a group of people and
computers that regulates the state of the transactions in the network.
Cryptocurrencies are decentralized, and the regulations are made by the majority of
the community.
Anonymity: In general, digital currencies require user identification. One needs to
upload a photo and some documents issued by the public authorities. Buying,
investing and any other processes with cryptocurrencies do not need require any of
that. Nevertheless, cryptocurrencies are not fully anonymous. Though the addresses
don‘t contain any confidential information such as name, residential address, etc.,
each transaction is registered; the senders and the receivers are publicly known.
Thus, all the transactions are tracked.
Transparency: Digital currencies are not transparent. One cannot choose the address
of the wallet and see all the money transfers. This information is confidential.

The Institute of Chartered Accountants of Nepal 65


Suggested Answer - June 2018

Cryptocurrencies are transparent. Everyone can see any transactions of any user,
since all the revenue streams are placed in a public chain.
Transaction manipulation: Digital currencies have a central authority that deals with
issues. It can cancel or freeze transactions upon the request of the participant or
authorities or on suspicion of fraud or money-laundering. Cryptocurrencies are
regulated by the community.
Legal aspects: Most countries have some legal framework for digital currencies, but
official status for cryptocurrency is not defined. The establishment of the legal
framework is only in the process.

The Institute of Chartered Accountants of Nepal 66


CAP II Paper 4: Financial Management

Specific Comments on the performance of the students


Batch: - June 2018
Level: - CAP-II
Subject: Financial Management
Question No. 1
Since question is too long students are not able to understand it properly. So, most of the
student are not able to solve it fully even if they try to attempt it. Failed to calculate CFAT.
Lacks of concept on Sunk cost. Failed to calculate working capital movement and its effect
on working capital.

Question No. 2
Most of the students who have attempted to solve part a of the question lacks conceptual
knowledge. Tax payable treatment is also not correct. Part b of the question was solved by
most of the students satisfactorily. Majority of students fails to calculate the working capital
cash cost basis. Some students could not calculate the opportunity cost.

Question No. 3
(a) Sub part of this question not able to solve by the most of student.
(b) Most solve the question but some made mistake in using COGS for calculating Inventory
used sales formula. Most of the students failed to calculate price of the share using P/E
ratio for year 8. Further almost all of them failed to apply IRR technique to arrive at the
required rate of return. Calculation of trade debtors is wrong in majority of cases.

Question No. 4
(a) Student fall to compute the EPS for the present condition and decision made without
considering present EPS. In some case working are missing. Some students fails to
calculate future earnings.
(b) Most solve this question, but some used market value weight. Calculation of weight is
wrong in some cases.

Question No. 5
(a) Not able to solve by most of the student. Calculation of new share price is wrong.
(b) Can solve by the most of the student but some made calculation error. Calculation of
portfolio risk and correlation coefficient is wrong.
(c) Being concept not practice by the student. Majority of students did not understand the
question.

Question No. 6
(a) Most of student writes on the basis of inventory management instead of cash
management.
(b) Solve by most of the student
(c) Not able to write to the point.
(d) Not able to write to the point.
Question No. 7

The Institute of Chartered Accountants of Nepal 67


Suggested Answer - June 2018

Answer not specific.

The Institute of Chartered Accountants of Nepal 68


CAP II Paper 5: Cost & Management Accounting

Paper 5:

Cost & Management Accounting

The Institute of Chartered Accountants of Nepal 69


Suggested Answer - June 2018

Maximum Marks - 100


Total No. of Questions: 6 Total No. of Printed Pages - 15
Time Allowed - 3 Hours

Marks
All questions are compulsory. Working notes should form part of the
answer.
Make assumptions wherever necessary.
1. Pappu Manufacturing Ltd. manufactures two products A and B. The manufacturing
division consists of two production department P1 and P2 and two service departments
S1 and S2. Budgeted overhead rates are used in the production departments to absorb
factory overhead to the products. The rate of department P1 is based on direct machine
hours, while the rate of department P2 is based on direct labour hours. In applying
overheads, the predetermined rates are multiplied by actual hours. For allocating the
service department costs to production departments, the basis adopted is as follows:
(i) Cost of department S1 to department P1 and P2 equal and
(ii) Cost of department S2 to department P1 and P2 in the ratio of 2:1 respectively
Annual profit plan data:
Factory Overheads budgeted for the year:
Departments P1 P2 S1 S2
Amount (Rs.) 27,85,000 22,55,000 7,50,000 5,10,000
Budgeted output of product A and B are 50,000 units and 30,000 units respectively.
Budgeted raw material cost per unit for product A and B are Rs.120 and Rs.150
respectively. Budgeted time required for production per unit are as follows:
Product A Product B
Department P1 1.5 machine hours 1.0 machine hours
Department P2 2 direct labour hours 2.5 direct labour hours
Average wage rates budgeted in Department P2 are: Product A – Rs. 72
per hour and Product B – Rs. 75 per hour.
All materials are used in Department P1 only.
Actual data (for the month of July 2017)
Units actually produced:
Product A 4,000 units
Product B 3,000 units.
Actual direct machine hours worked in Department P1:
On Product A - 6,100 hours, Product B - 4,150 hours.
Actual direct labour hours worked in Department P2:
On Product A - 8,200 hours, Product B - 7,400 hours.
Costs actually incurred:
Product A Product B
Raw Materials 5,10,000 4,80,000

The Institute of Chartered Accountants of Nepal 70


CAP II Paper 5: Cost & Management Accounting

Wages 5,80,000 5,50,000


Factory Overheads:
Departments P1 P2 S1 S2
Amount 2,81,000 2,25,000 72,000 51,000
Required: 20
i) Compute the predetermined overhead rate for each production department.
ii) Prepare a Statement showing Budgeted and Actual costs for the month of July, 2017.
Answer:
i) Computation of predetermined overhead rate for each production department for budgeted
data

Particulars Production Departments Service Departments


P1 (Rs.) P2 (Rs.) S1 (Rs.) S2 (Rs.)
Budgeted overhead for the year 27,85,000 22,55,000 7,50,000 5,10,000
Allocation of Service department 3,75,000 3,75,000 (7,50,000) -
S1's cost to Production Dept. P1
and P2 equally
Allocation of Service department 3,40,000 1,70,000 - (5,10,000)
S2‘s cost to Production Dept.
P1and P2 in the ratio of 2:1
Total 35,00,000 28,00,000 Nil Nil
Budgeted Machine hours in 1,05,000
department P1(working note 1)
Budgeted Direct labour hours in 1,75,000
department P2 (working note 1)
Budgeted Machine/ Direct labour Rs. 33.33 Rs. 16
hour rate

ii) Statement showing Budgeted and Actual Costs for the month of July 2017.

Particulars Budgeted (Rs.) Actual (Rs.)


Raw Materials used in Department P1
A (4,000 units x Rs.120) 4,80,000 5,10,000
B (3,000 units x Rs.150) 4,50,000 4,80,000
Direct Labour Cost on the basis of labour hours worked
in
department P2 5,76,000 5,80,000
A (4,000 x 2 hrs x Rs. 72) 5,62,500 5,50,000
B (3,000 x 2.5 hrs x Rs. 75)
Factory Overheads:
On machine hour basis in Department P1
A (4,000 x1.5 hrs x Rs. 33.33) 1,99,980 2,08,888
B (3,000 x 1hr x Rs. 33.33) (working note 3) 99,990 1,42,112
On Direct labour hour basis in Department P2
A (4,000 x 2 hrs x Rs. 16) 1,28,000 1,46,128

The Institute of Chartered Accountants of Nepal 71


Suggested Answer - June 2018

B (3,000 x 2.5 hrs x Rs. 16) (working note 3) 1,20,000 1,31,872


Total 26,16,470 27,49,000

Working Notes:
Product A Product B Total
1. Budgeted output (in units) 50,000 30,000
Budgeted Machine hours in 75,000 hrs 30,000 hrs 1,05,000
Department P1 (50,000 x 1.5 (30,000 x 1 hrs
hrs) hr
Budgeted Direct labour hour in 1,00,000 hrs 75,000 hrs 1,75,000
Department P2 (50,000 x 2 (30,000 x 2.5 hrs
hrs) hrs)
2. Actual output (units) 4,000 3,000
Actual Machine hours utilized in 6,100 4,150 10,250
Department P1
Actual Direct labour hours utilized in 8,200 7,400 15,600
Department P2
Working Notes: 3
Computation of actual overhead rates for each production department from actual data.
Particulars Production Departments Service Departments
P1 (Rs.) P2 (Rs.) S1 (Rs.) S2 (Rs.)
Actual factory overhead for July 2,81,000 2,25,000 72,000 51,000
2017
Allocation of Service department 36,000 36,000 (72,000) -
S1's
cost to Production Dept. P1 and P2
equally
Allocation of Service department 34,000 17,000 - (51,000)
S2's
cost to Production Dept. P1 and P2
in
the ratio of 2:1
Total 3,51,000 2,78,000 Nil Nil
Actual Machine hours in 10,250
department P1 (Working note 2)
Actual Direct labour hours in 15,600
department P2 (Working note 2)
Machine hour rate Rs. 34.2439
(Rs. 3,51,000 /10,250)
Direct Labour hour rate Rs. 17.8205
(Rs. 2,78,000 / 15,600)
Overhead Absorbed
Product A 2,08,888 1,46,128
(Rs.34.2439 (Rs.17.8205
x 6,100) x 8,200)

The Institute of Chartered Accountants of Nepal 72


CAP II Paper 5: Cost & Management Accounting

Product B 1,42,112 1,31,872


(Rs.34.2439 (Rs.17.8205
x 4,150) x 7,400)
Total 3,51,000 2,78,000

2.
a) The Margos company has just completed operation for the year 2017. The company‘s
Assistant Accountant(who is very inexperienced) prepared the following Profit and
Loss Account for the years activities:
Rs.
Sales 32,00,000
Operating Expenses: Rs.
Insurance 40,000
Gas Electricity and water 1,00,000
Direct Labour Cost 6,00,000
Indirect Labour Cost 1,20,000
Depreciation of Factory Equipment 1,60,000
Raw Materials purchased during the year 12,00,000
Rent 4,00,000
Selling and Admn. Overheads 3,20,000
29,40,000
Net Profit 2,60,000
You have been asked to assist the company in preparing correct Profit & Loss
Account for the year 2017. The following additional information is available:
1) The company is a manufacturing firm that produces a product for sale to outside
customers.
2) 80 percent of the rent paid applies to factory operation and the remainder to
Selling and Administration activities.
3) No raw materials were on hand on 1st January. However, raw material of the value
of Rs 1,50,000 purchased during 2017 were still on hand on 31st December. The
remainder was used in production during the year.
4) 70 percent of the Insurance and 90% of the Gas Electricity and Water paid apply
to factory operations the remainder apply to selling and Administration activities.
5) Work in Progress and finished goods inventories were
1st January 31st
December
Work in progress Rs. 4,20,000 Rs. 4,80,000
Finished goods Rs. 5,40,000 Rs. 4,00,000
Required: 10
i) A statement of cost of goods manufactured in 2017, and
ii) A corrected Profit & loss Account for the year ended 31st December, 2017.

b) Budhanilkantha is a Public School having 25 buses each plying in different directions


for the transport of its school students. In view of large number of students availing of
the bus service, the buses work two shifts daily both in the morning and in the

The Institute of Chartered Accountants of Nepal 73


Suggested Answer - June 2018

afternoon. The buses are garaged in the school. The workload of the students has been
so arranged that in the morning, the first trip picks up senior students and the second
trip plying an hour later picks up junior students. Similarly, in the afternoon, the first
trip takes the junior students and an hour later the second trip takes the senior students
home.
The distance travelled by each bus, one way is 16 kms. The school works 24 days in a
month and remains closed for vacation in Mansir and Kartik. The bus fee, however, is
payable by the students for all the 12 months in a year.
The details of expenses for the year 2073-2074 are as under:
Driver's salary -payable for all the 12 months Rs. 5,000 per month per driver
Cleaner's salary payable for all the 12 months
(one cleaner has been employed for every five Rs. 3,000 per month per cleaner
buses)
Licence Fees, Taxes etc. Rs. 2,300 per bus per annum
Insurance Premium Rs. 15,600 per bus per annum
Repairs and Maintenance Rs. 16,400 per bus per annum
Purchase price of the bus Rs 16,50,000 each
Life of the bus 16 years
Scrap value Rs. 1,50,000
Diesel Cost Rs. 18.50 per litre

Each bus gives an average of 10 kms per litre of diesel. The seating capacity of each bus
is 60 students. The seating capacity is fully occupied during the whole year.
The school follows differential bus fees based on distance travelled as under:
Students picked up and dropped within Bus fee Percentage of students
the range of distance from the school availing this facility
4 kms 25% of Full 15%
8 kms 50% of Full 30%
16 kms Full 55%
Ignore interest. The bus fees has to be based on average cost, you are
required to: 10
i) Prepare a statement showing the expenses of operating a single bus
and the fleet of 25 buses for a year.
ii) Work out average cost per student per month in respect of
(a) Students coming from a distance of upto 4 kms from the school;
(b) Students coming from a distance of upto 8 kms from the school; and
(c) Students coming from a distance of upto 16 kms from the school.
Answer:
a) Statement of Cost of Goods Manufactured in 2017.
Direct Material consumed :Purchased in Rs .12,00,000
Less: Closing Stock 1,50,000 Rs 10,50,000

The Institute of Chartered Accountants of Nepal 74


CAP II Paper 5: Cost & Management Accounting

Direct Labour 6,00,000


Manufacturing Overhead:
Indirect labour 1,20,000
Gas . Electricity and Water(90% of Rs. 1,00,000) 90,000
Insurance(70% of Rs. 40,000) 28,000
Depreciation 1,60,000
Rent(80% of Rs. 4,00,000) 3,20,000 7,18,000
23,68,000
Add: Opening Work –in-progress 4,20,000
27,88,000
Less: Closing Work –in – progress 4,80,000
Cost of Goods manufactured 23,08,000

(b) Corrected Profit & Loss Account


for the year ended December 31, 2017
Rs Rs
Sales 32,00,000
Less: Cost of Sales:
Cost of goods manufacture (as per ‗a‘) 23,08,000
Add: Opening stock of finished goods 5,40,000
28,48,000
Less: Closing stock of finished goods 4,00,000
Cost of goods sold 24,48,000
Add: Selling Adm:
Selling & Admn. Overhead 3,20,000
Rent (20% of Rs. 4,00,000) 80,000
Insurance (30% of Rs. 40,000) 12,000
Gas.Electricity & Wate 10% of Rs. 1,00,000) 10,000 4,22,000
Cost sales 28,70,000
Profit 3,30,000

i) Budhanilkantha Public School


Statement showing the expenses of operating a single bus and the fleet of 25
buses for a year
Particulars Fleet of 25
Per bus per annum buses per
annum
(Rs.) (Rs.)
Running costs : (A)
Diesel 56,832 14,20,800
(Refer to working note 1 )
Repairs & maintenance costs : 16,400 4,10,000
(B)
Fixed charges :
Driver's salary 60,000 15,00,000

The Institute of Chartered Accountants of Nepal 75


Suggested Answer - June 2018

Cleaners salary 7,200 1,80,000


Licence fee, taxes etc. 2,300 57,500
lnsurance 15,600 3,90,000
Depreciation 93,750 23,43,750
Total fixed charges : ( C ) 1,78,850 44,71,250
Total expenses: (A + B + C) 2,52,082 63,02,050

ii) Average cost per student per month in respect of students coming from a
distance of :

a) 4 kms. from the school


{Rs. 2,52,082 /(354 students x 12 months)} Rs. 59.34
(Refer to working note 2)
b) 8 kms from the school
(Rs. 59.34 x 2) Rs. 118.68
c) 16 kms from the school
(Rs. 59.34 x 4) Rs. 237.36

Working notes:
1. Calculation of diesel cost per bus;

No. of trips made by a bus each day : 4


Distance travelled in one trip both :
32 kms
ways
(16 kms x 2 trips)

Distance travelled per day by a bus :


128 krns
(32 kms x 4 shifts)
Distance travelled during a month : 3,072 kms
(128 kms x 24 days)
Distance travelled per year : 30,720 kms
(3,072 kms x 10 months)
No. of litres of diesel required per :
3,072 litres
bus per year
(30,720 kms /10 km)
Cost of diesel per bus per year : Rs. 56,832
(3,072 litres x Rs. 18.50)

2. Calculation of Equivalent Number of full fare paying student:


Distance % No Wt Equivalent
Within 4 km 15 18 25% 4.5
4-8 30 36 50% 18
8-16 55 66 100% 66
Total 120 88.5

The Institute of Chartered Accountants of Nepal 76


CAP II Paper 5: Cost & Management Accounting

Avg Cost/ full fare paying student = 252082


88.5x12
= 237.36
3.
a) Nepa Engineering produces one of the component ―Metal Shaft‖ from a single raw
material in economic lots of 2,000 units. The raw material cost is Rs. 2 per Metal
Shaft. Average annual demand is 20,000 units. The annual holding cost of material is
Re. 0.25 per unit and the minimum stock level is set at 400 units. Direct labour costs
for the component are Rs. 6 per unit, fixed manufacturing overhead is charged at a
rate of Rs. 3 per unit based on normal activity of 20,000 units. The company also
hires a machine on which the components are produced at a rate of Rs 200 per month.
Required: (5+3=8)
i) What will be the total annual manufacturing cost of 20,000 units of Metal Shaft?
ii) Nepa Engineering is considering the possibility of purchasing from a supplier the
components now it makes. The supplier will provide necessary components at a
unit price of Rs. 9. Transportation and storage cost would be negligible. Should the
company purchase the component instead of manufacturing?
b) A job can be executed either through workman M or N. M takes 32 hours to complete
the job while N finishes it in 30 hours The standard time to finish the job is 40 hours.
The hourly wage rate is same for both the workers. In addition workman M is entitled
to receive bonus according to Halsey plan (50%) sharing while N is paid bonus as per
Rowan plan. The works overheads are absorbed on the job at Rs. 7.50 per labour hour
worked. The factory cost of the job comes to Rs. 2,600 irrespective of the workman
engaged.
Find out the hourly wage rate and cost of raw materials input. Also show cost against
each element of cost included in factory cost. 8
c) How will you treat Idle Capacity Cost in Accounting? 4
Answer:
a) Working Note:
1. The cost of placing an order when the raw material is purchased is not given.
Let the cost of placing an order be X
_______________________________________
Economic Batch Quantity (EBQ) = 2 x Annual Consumption x Cost of placing an order
√ Cost of carrying one unit of inventory for one
year

Substituting the available Information


______________
2,000 = 2 x 20,000 x X
√ 0.25
X = 40,00,000/160,000
Or X = Rs. 25

The Institute of Chartered Accountants of Nepal 77


Suggested Answer - June 2018

Cost of placing an order = Rs. 25


2. Average stock level = Minimum stock level + ½ EOQ
= 400 + ½ (2000)
= 1,400 units

i) Calculation of total annual manufacturing cost


Rs.
Material Cost (20,000 x Rs.2) 40,000
Storage cost (1,400 x Re. 0.25) 350
Ordering Cost (20,000/2,000) x Rs. 25 250
Labour cost (20,000 x Rs. 6) 1,20,000
Rental Charges (Rs.200 x 12) 2,400
Fixed Manufacturing Overhead (20,000 x 3) 60,000
Total annual manufacturing cost 223,000

ii) Total annual manufacturing cost (calculated as above) 223,000


Less: Fixed Manufacturing Overhead (Sunk cost) 60,000
Manufacturing Cost 163,000
Purchase cost (20,000 x 9) 180,000
Fixed cost being sunk cost is not relevant for decision making. The company
should not purchase the component. Bying the components will be beneficial
only if there is alternative use of existing capacity and the opportunity cost
exceeds Rs. 17,000 i.e. Rs. 1,80,000 – Rs. 1,63,000.
b)
Working notes:
1. Time saved and wages:
Workmen M N
Standard time (hrs) 40 40
Actual time taken (hrs) 32 30
Time saved (hrs) 08 10
Wages paid @ Rs. x per hr. (Rs.) 32x 30x
2. Bonus Plan:
Halsey Rowan
Time saved (hrs) 8 10
Bonus (Rs.) 4x 7.5x
(8 hrs × Rs x) (10 hrs/40 hrs × 30 hrs × Rs x)
2
3. Total wages:
Workman M: 32x + 4x = Rs 36x
Workman N: 30x + 7.5x = Rs 37.5x
Let Material Cost be y
Statement of factory cost of the job
Workmen M N
Rs. Rs.
Material cost y y

The Institute of Chartered Accountants of Nepal 78


CAP II Paper 5: Cost & Management Accounting

Wages (Refer to working note 3) 36x 37.5x


Works overhead 240 225
Factory cost 2,600 2,600
The above relations can be written as follows:
36x + y + 240 = 2,600 …. (i)
37.5x+ y+ 225 = 2,600 …..(ii)
Subtracting (i) from (ii) we get
1.5x – 15 = 0
or 1.5 x = 15
or x = Rs. 10 per hour
On substituting the value of x in (i) we get y = Rs. 2,000
Hence the wage rate per hour is Rs. 10 and the cost of raw material input is Rs. 2,000 on
the job.
Statement of showing factory cost
Particulars M N
Material 2000 2000
Wages 360 375
Factory OH 240 225
Total 2600 2600
c)
Treatment of Idle capacity cost: Idle capacity costs can be treated in product costing, in the
following ways:
(i) If the idle capacity cost is due to unavoidable reasons such as repairs, maintenance,
changeover of job, etc, a supplementary overhead rate may be used to recover the idle
capacity cost. In this case, the costs are charged to the production capacity utilised.
(ii) If the idle capacity cost is due to avoidable reasons such as faulty planning, power
failure etc., the cost should be charged to profit and loss account.
(iii) If the idle capacity cost is due to seasonal factors, then, the cost should be charged to
the cost of production by inflating overhead rates.

4.
a) Xion Ltd. sold 3,00,000 units of its product at Rs. 40 per unit. Variable costs are Rs.
20 per unit (manufacturing costs of Rs. 14 and selling cost Rs. 6 per unit). Fixed costs
are incurred uniformly throughout the year and amount to Rs. 35,00,000 (including
depreciation of Rs. 15,00,000). There are no beginning or ending inventories.
You are required to calculate: 5
i) Break-even sales level quantity and cash break-even sales level quantity.
ii) P/V ratio.
iii) Number of units that must be sold to earn an income (EBIT) of Rs. 2,50,000.
iv) Sales level required to achieve an after-tax income (PAT) of Rs. 2,50,000.
Assume 40% corporate Income Tax rate.
b) Kalika Limited obtained a contract No. 999 for Rs. 50 lacs. The
following details are available in respect of this contract for the year
ended Ashad 31, 2074:

The Institute of Chartered Accountants of Nepal 79


Suggested Answer - June 2018

Rs.
Materials purchased 1,60,000
Materials issued from stores 5,00,000
Wages and salaries paid 7,00,000
Drawing and maps 60,000
Sundry expenses 15,000
Electricity charges 25,000
Plant hire expenses 60,000
Sub-contract cost 20,000
Materials returned to stores 30,000
Materials returned to suppliers 20,000
The following balances relating to the contract No. 999 for the year
ended on Ashad 31, 2073 and Ashad 31, 2074 are available:
as on 31st Ashad, 2073 as on 31st Ashad, 2074
Work certified 12,00,000 35,00,000
Work uncertified 20,000 40,000
Materials at site 15,000 30,000
Wages outstanding 10,000 20,000
The contractor receives 75% of work certified in cash.
Prepare Contract Account. 5
c) Moti manufacturers – a small scale enterprise, produces a single
product and has adopted a policy to recover the production overheads
of the factory by adopting a single blanket rate based on machine
hours. The annual budgeted production overheads for the year 2016-
17 are Rs. 44,00,000 and budgeted annual machine hours are
2,20,000.
For a period of first six months of the financial year 2016-2017,
following information were extracted from the books:
Actual production overheads Rs. 24,88,200
Amount included in the production overheads:
Paid as per court‘s order Rs. 1,28,000
Expenses of previous year booked in current year Rs. 1,200
Paid to workers for strike period under an award Rs. 44,000
Obsolete stores written off Rs. 6,700
Production and sales data of the concern for the first six months are as under:
Production:
Finished goods 24,000 units
Works-in-progress

The Institute of Chartered Accountants of Nepal 80


CAP II Paper 5: Cost & Management Accounting

(50% complete in every respect) 18,000 units


Sale:
Finished goods 21,600 units
The actual machine hours worked during the period were 1,16,000 hours. It is revealed
from the analysis of information that ¼ of the under/ over absorption was due to
defective production policies and the balance was attributable to increase/decrease in
costs.
Determine the amount of under/over absorption of production overheads for the six
months period of 2016-17 and point out accounting treatment of under/over absorption
of production overheads. 5
Answer:
a)
(i) Break-even Sales Quantity = (Fixed Cost /Contribution per unit)
= 35,00,000 / 20
= 1,75,000 units
Cash break even Sales Quantity = (Cash Fixed Cost/Contribution per unit)
= 20,00,000 /20
= 1,00,000 units
Where cash fixed cost = Fixed cost - Depreciation
= 35,00,000 - 15,00,000
= 20,00,000
(ii) PV Ratio = (Contribution/selling price per unit) x 100
= (20/40)*100
= 50%
(iii) No. of units that must be sold to earn an Income (EBIT) of Rs. 2,50,000
= ( Fixed Cost + Desired EBIT Level)/ Ccontribution per unit
= (35,00,000+2,50,000)/20
= 1,87,500 units
(iv) After Tax Income(PAT) = Rs. 2,50,000
Tax rate = 40%
Desired level of Profit before tax = (2,50,000/60)x100 = Rs. 4,16,667
Estimate Sales Level = (Fixed Cost + Desired Profit)/ PV ratio
= (35,00,000 + 4,16,667) / 50%
= Rs. 78,33,334

b) In the books of Kalika Ltd.


Contract No. 999 Account for the year ended 31st Ashad, 2074
Dr. Cr.
Amount
Particulars Particulars Amount (Rs.)
(Rs.)
By Material
To Work in progress bld: 30,000
returned to store
-Work certified 12,00,000 By Material 20,000

The Institute of Chartered Accountants of Nepal 81


Suggested Answer - June 2018

returned to suppliers
By Stock (Material)
-Work uncertified 20,000 30,000
c/d
15,000 By Work in
To Stock (Materials) bld
progress c/d:
To Material purchased 1,60,000 -Work certified 35,00,000
To Material issued 5,00,000 -Work uncertified 40,000
To Wages paid 7,00,000
Less: Opening 0/s (10,000)
Add: Closing 0/s 20,000 7,10,000
To Drawing and maps* 60,000
To Sundry expenses 15,000
To Electricity charges 25,000
To Plant hire expenses 60,000
To Sub- contract cost 20,000
To Notional profit cld
(balancing figure) 8,35,000
36,20,000 36,20,000
By Notional profit
To Costing P& L Alc (W N -1) 4,17,500 8,35,000
b/d
To WIP Reserve (balancing
4,17,500
figure)
8,35,000 8,35,000

*Assumed that expenses incurred for drawing and maps are used exclusively for this contract
only.
Working Note:
% Completion of contract = Work certified/Contract price×100%
= 35,00,000/50,00,000×100%
= 70%
Profit to be transferred to Costing Profit & Loss Account
=2/3*Notional profit* Cash received/Work certified
= 2/3*8,35,000*75/100
= Rs. 4,17,500
c) Amount of under/ over absorption of production overheads during the
period of first six months of the year 2016-2017:
Amount Amount
(Rs) (Rs)
Total production overheads actually
24,88,200
incurred during the period

Less: Amount paid to worker as per 1,28,000


court order

The Institute of Chartered Accountants of Nepal 82


CAP II Paper 5: Cost & Management Accounting

Expenses of previous year booked in


1,200
the current year

Wages paid for the strike period under


44,000
an award

Obsolete stores written off 6,700 (1,79,900)


23,08,300
Less: Production overheads absorbed
as per machine hour rate (1,16,000 23,20,000
hours × Rs. 20*)
Amount of over absorbed production 11,700
overheads

*Budgeted Machine hour rate (Blanket rate) = 44,00,000/2,20,000 hours


= Rs. 20 per hour

Accounting treatment of over absorbed production overheads:

The over/under absorption of overhead can be treated as


i) Charged to costing P/L A/C
ii) Carry forward to next Accounting Period
iii) Use of supplementary rate
Here the over/under absorbed overhead due to abnormal reason is charged to costing P/L
A/C and the balance under/over absorbed overhead due to normal reason is charged to
i) Cost of sales for units sold
ii) Closing stock for units unsold by calculating supplementary rate.

5.
a) From the following data of A and Co. Ltd. relating to budgeted and actual
performance for the month of March 2017, compute the Direct Material and Direct
Labour Cost variances. 7
Budgeted data for March:
Units to be manufactured 1,50,000
Units of direct material requires ( based on std. Rates) 4,95,000
Planned purchase of Raw Materials ( units) 5,40,000
Average unit cost of direct materials Rs. 8
Direct Labour hours per unit of finished goods 3/4 hr
Direct Labour Cost (total) Rs. 29,92,500
Actual data at the end of March:
Units actually manufactured 1,60,000
Direct Material Cost ( purchase cost based on units actually
issued) Rs. 43,41,900

The Institute of Chartered Accountants of Nepal 83


Suggested Answer - June 2018

Direct Material Cost ( purchase cost based on units actually


purchased) Rs. 45,10,000
Average unit cost of direct materials Rs. 8.20
Total Direct Labour hours for March 1,25,000
Total Direct Labour cost for March Rs. 33,75,000
b) It should be management‘s endeavor to increase inventory turnover but to reduce
labour turnover. Expand and illustrate the idea contained in this statement. 4
c) There is generally a divergence between financial profits and cost profits. Explain this
statement and give reasons for such divergence. 4
Answer:
a)
Direct
Material
Variance:
= Std. Cost of Actual Output - Actual Cost
= 1,60,000*3.3*8-43,41,900
DMCV = 42,24,000-43,41,900 = 1,17,900 (Adverse)
= Actual Qty.*( Std. Rates- Actual Rates)
DMPV = 5,29,500*(8-8.20) =Rs.1,05,900( Adverse)
= Std. Rate* (Std. Qty. for Actual Output- Actual Qty.)
= 8*(1,60,000*3.30-5,29,500)
DMUV =Rs. 12,000 (Adverse)

Direct
Labour
Variances
= Std. Cost for Actual Output- Actual Cost
= 1,60,000*3/4*26.60-33,75,000
DLCV = Rs. 1,83,000(Adverse)
DLRV =Actual time * (Std. Rate-Actual Rate)
=1,25,000*(26.60-27)
=Rs.50,000 (Adverse)
=Std. Rate* (Std. Time for Actual Output- Actual Time)
=26.60*( 1,60,000*3/4-1,25,000)
DLEV =Rs. 1,33,000 (Adverse)

Working Notes:
i) Standard Units of Direct Material required per unit of output:
= 4,95,000/1,50,000 = 3.30Units
ii) Total Actual Quantity of Direct Materials Used:
= 43,41,900/8.20 = 5,29,500 Units
iii) Standard Direct Labour Cost 29,92,500/ (1,50,000*3/4) =
per hour: Rs. 26.60
iv) Actual Direct Labour cost per hour:

The Institute of Chartered Accountants of Nepal 84


CAP II Paper 5: Cost & Management Accounting

=33,75,000/1,25,000 = Rs. 27
OR
Direct Material variance
Standard cost
Actual cost
SQ×SP SP×AQ
AQ×AP
(495000/150000×160000)×8 8×(4341900/8.2)
4224000 4236000
4341900

Material usage variance = 12000(A) Material Price variance =


105900(A)

Material Cost variance = 117900(A)


Where Actual quantity of material = 4341900/8.2 = 529500

Labor Variance
Standard cost
Actual cost SHr×SR SR×AHr
AHr×AR
(3/4×160000)×26.6 26.6×125000
3192000 3325000
3375000

Labor efficiency variance = 133000(A) Labor Rate variance = 50000(A)

Labour cost variance = 183000(A)


Where
Standard Rate hour = Standard Labour cost/Standard Labour Hour
= 2992500
150000×3/4
= Rs. 26.6
b)
Inventory turnover: It is a ratio of the value of materials consumed during a period to the
average value of inventory held during the period. A high inventory turnover indicates fast
movement of stock.
Labour turnover: It is defined as an index denoting change in the labour force for an
organization during a specified period. Labour turnover in excess of normal rate is termed as
high and below it as low turnover.

The Institute of Chartered Accountants of Nepal 85


Suggested Answer - June 2018

Effects of high inventory turnover and low labour turnover: High inventory turnover
reduces the investment of funds in inventory and thus accounts for the effective use of the
concern‘s financial resources. It also accounts for the increase of profitability of a business
concern. As against high labour turnover the low labour turnover is preferred because high
labour turnover causes-decrease in production targets; increase in the chances of break-down
of machines at the shop floor level; increase in the number of accidents; loss of customers
and their brand loyalty due to either non-supply of the finished goods or due to sub-standard
production of finished goods; increase in the cost of selection, recruitment and training;
increase in the material wastage and tools breakage.
All the above listed effects of high labour turnover account for the increase in the cost of
production/ process/ service. This increase in the cost finally accounts for the reduction of
concern‘s profitability. Thus, it is necessary to keep the labour turnover at a low level.
As such, it is correct that management should endeavour to increase inventory turnover and
reduce labour turnover for optimum and best utilization of available resources and reduce the
cost of production and thus increase the profitability of the organization.

c) Financial accounts are concerned with the ascertainment of profit or loss for the whole
operation of the organization or a relatively long period usually a year, without being too
much concerned with cost computation, whereas cost accounts are provided for ascertaining
the profit or loss made by manufacturing or product division/products for cost comparison
and preparation and use of variety of cost statements. As these two sets of accounts are
maintained in different forms or follow different approach, it is quite natural that their results
may also differ. Invariably, the profit and loss revealed by the financial accounts may not
agree with the profit or loss as per cost accounts.
Reasons for Disagreement
Disagreements between financial profits and cost profits may arise due to the following
reasons:
1. Items shown only in financial accounts: There are certain items which are included in
financial accounts but find no place in cost accounts. These may be–
(a) Purely financial charges: e.g., loss on sale of fixed assets, discount on issue of
shares, damages payable, etc.
(b) Purely financial income: e.g. profit on sale of fixed assets, interest received, transfer
fees, etc.
(c) Appropriation of profits: e.g. dividends, income-tax, transfer to reserves, etc.
2.Items included in cost accounts only: There are certain items which are included in cost
accounts but not in financial accounts, e.g. notional interest on capital, notional rent on
premises owned, etc.
3.Over or under absorption of overheads : In cost accounts overheads are charged to
production on pre-determined rates while financial accounts show the actual amount of

The Institute of Chartered Accountants of Nepal 86


CAP II Paper 5: Cost & Management Accounting

overheads. If the overheads charged are not equal to the amount of overheads incurred, the
difference gives rise to over or under absorption causing difference in profits.
4.Different bases of stock valuation: In financial books, stocks are valued at cost or market
price whichever is less. However, in cost accounts stock of materials may be valued on
FIFO, LIFO, average method, etc. and work-in-progress may be valued at prime cost or
works cost, thus, there is difference in profits.
5.Different methods of charging depreciation: The amount of depreciation charged may be
different in two sets of books either because of the different methods of calculating
depreciation or the rates adopted, hence the profits may be different.
6. Abnormal gains and losses: Abnormal gains and losses may completely be excluded from
cost accounts or may be taken to costing profit and loss account. If it is excluded, costing
profit/loss will differ from financial profit/loss and adjustment will be required.

6. Write short notes: (4×2.5=10)


a) Operating Costing
b) Uniform Costing
c) Standard Costing
d) Efficiency Rating
Answer:
a) Operating Costing
It is the method of ascertaining costs of providing or operating a service. This method of
costing is applied by those undertaking which provide service rather than production of
commodities. The emphasis under operating costing is on the ascertainment of cost of
services rather than on the cost of manufacturing a product. This costing method is usually
made use of by transport companies, gas and water works department, electricity supply
companies, canteens, hospitals, theatres, schools etc.

b) Uniform Costing
Uniform costing is not a separate method of cost accounting. The basic idea behind uniform
costing is that the different concerns in an industry should adopt a common method of
costing and apply uniformly the same principles and techniques for better cost comparison
and common good.
The main objectives of uniform costing are as follows:-
a) To facilitate the comparisons of cost and performance of different units in the same
industry; it provides objective basis
b) To eliminate unhealthy competition among the different units of an industry
c) To improve production capacity level and labor efficiency by comparing the production
cost of different units with each other.
d) To bring standardization and uniformity in the operation of participating units.
c) Standard Costing

The Institute of Chartered Accountants of Nepal 87


Suggested Answer - June 2018

Standard costing is defined as the presentation and use of standard costs, their comparison
with actual costs and the analysis of variances to their causes and points of incidence.
Standard costing, thus is a system of costing which can be used in conjuction with any
method of costing, like job costing, process costing etc. Standard costs are pre-determined by
using a careful analysis of production methods, physical conditions and price factors. They
represent achievable targets and help to build up budgets gauge performance and obtain
product costs. The actual costs will vary from month to month or even from day to day.
The basic objective, therefore, of standard costing system is to assists the departmental head
by identifying and describing the variances over which he has control.

d) Efficiency Rating
Efficiency is usually related with performance and may be computed by computing the time
taken with the standard time allotted to perform the given job/task. If the time taken by the
worker on a job is equal or less than the standard time, then he is rated efficient. In case he
takes more time than the standard time he is rated as inefficient. It may be computed as
follows;
Efficiency in Percent=Time allowed as per std*100/ Time taken

The Institute of Chartered Accountants of Nepal 88


CAP II Paper 5: Cost & Management Accounting

Specific Comments on the performance of the students


Batch: - June 2018
Level: - CAP-II
Subject: Cost & Management Accounting
Question No. 1
Majority of students fail to answer this question correctly. Presentation and working
note was not clear. Very few students get pass mark in this question. Only few
students were able to solve the problem. Students mainly face difficulty in calculating
the accrual and budgeted overheads.
Question No. 2
Almost all the students attempted this question. They spent more time on part a. Most
of the students did not able to calculate equivalent no. of full fare paying students.
Concept of cost sheet is not proper in number of student. Difficulty in calculation of
equivalent student and must be focus on format of running charge and standing
charge. Students were struggling to calculate the rate (full fare/ equivalent student).
Average performance is good. Few mistakes are seen in calculating direct and indirect
expenses cost of goods manufacture. Almost students not able to calculate direct cost
and cost per students. Most of the student could not calculate the average cost per
student. Lack of conceptual clarity in most of the students was noticed.
Question No. 3
Almost all the students attempted this question. No one answer correctly for 2nd part
of a. Conceptual knowledge regarding relevant costing is lacking. All most all of the
students are not able to get current ordering cost and storage cost. Negligence on
theory part. Most of the students failed to calculate the effective cost of purchase
without considering the fixed overhead. Not able to calculate ordering cost. Few
students did mistake while calculating wages because of lack of conceptual
knowledge.
Question No. 4
Majority of students answered 'a' and 'b' part correctly. For 'c' part only few students
answered correctly. Students did not properly understand the requirements of part
three. They have provided accounting treatment with entry instead of mentioning or
explaining the accounting treatment. Computed the wrong lab cost in Contract
costing. Students were not able to explain the treatment of over/under absorption of
OHS. Some of the students could not calculate the over absorbed amount.
Question No. 5
Average performance on this part. Students have tried for part two and three. But their
knowledge is poor to explain the given statements. Students were not able to compute
the variances. Student did not perform well because not able to calculate actual
quantity of material and standard rate per hour. Some of the students gave sketchy
answers.
Question No. 6
Few students even don't know the concept and just answer to attempt the question.
Students have little knowledge about efficiency rating. Theory was not up to the
mark.

The Institute of Chartered Accountants of Nepal 89


Suggested Answer - June 2018

Paper 6:

Business Communication & Marketing

The Institute of Chartered Accountants of Nepal 90


CAP II Paper 6: Business Communication & Marketing

Part A Business Communication

The Institute of Chartered Accountants of Nepal 91


Suggested Answer - June 2018

Roll No……………. Maximum Marks – 100


Total No. of Questions - 8 Total No. of Printed Pages -7
Time Allowed – 3 Hours
Marks
All questions are compulsory.
Section -'A'
1. Read the following case carefully and answer the questions given below: (4×5=20)
Dr. Chandani Barma is the CEO at one of the reputed banking companies in Nepal. She
was appointed for the post due to the rich international exposure and experience that she
had attained in the banking sector. Right from her appointment she has realized that the
company is not making desirable progress despite the proactive role of the entire team of
staff and directors. She has recently investigated from her preliminary survey that
succeeding at her workplace has a serious threat due to the subtle conflict among the
promoters of the bank that constitute the Board of Directors. The policies that they‘ve
adopted are not perfectly matching with the contexts of the new-era market. Neither have
they been able to launch the innovative and fascinating programs for their clients and
customers.
During informal communication, they often blame each other of not being ready for
taking risks for the promotion of the bank. But, in the Board meeting, neither of them
spell out clearly about the challenges and changes they desire to have in their company. In
this condition, the CEO, Dr. Barma has had a proposal approved from the BoD to carry
out a comprehensive survey among the shareholders of the company about their desires to
have challenges and changes in the company.
Now, Dr. Barma is required to develop research tools, collect data, analyze data and
prepare an analytical survey report with useful findings and recommendations.
Questions:
a) Dr. Barma is required to prepare an analytical report. What are the basic features of
this type of report? What are its major components?
b) Prepare a set of questionnaire for the survey as one of the tools of data collection.
c) Write two objectives and the statement of problem that Dr. Barma would present in
her report.
d) How is information organized in this report? Illustrate.

Answer:
a) An analytical report is usually a research report. It is also called investigative report. It
is prepared on the basis of the information obtained from respondents of the related
field. It requires basically the research tools such as questionnaires, interview, focused
group discussion, observation report, tests, discourse analysis, etc. Scientific analysis
and possible interpretations of the data are made in this type of report.
The basic components of an analytical report are:

The Institute of Chartered Accountants of Nepal 92


CAP II Paper 6: Business Communication & Marketing

 Introduction
 General background
 Statement of problem
 Objectives
 Methodology
 Analysis and interpretation
 Findings, conclusion and recommendations.

b) Questionnaire for the shareholders

Tick the best option. (SA: strongly agree, A: agree, NA: not agree)
Opinions/attitudes SA A NA
1. I prefer risks and challenges in business; they help to
grow the company.
2. Risks are useful to motivate me to work and
concentrate on duty.
3. I don‘t like to be tied up by business commitments and
relationships. It‘s good to keep on what is with us
conventionally.

4. I‘m ready to allow the BoD to invest excessively on new


technology and globalization of market.

5. I don‘t care whether one failure in business loses


everything. So, our bank must invest as required on the
innovative activities.

6. I‘d like to follow the same pattern of business since it


has to bear less or no risk.

7. …

c) The objectives of the report

 To present the accurate views of the shareholders of the bank towards taking
risks in the growth of business;
 To recommend the BoD about adopting useful strategies for business growth
with certain innovations
Statement of problem
Siddhi Bank Pvt. Limited has been launching a number of financial programs and
activities including saving, loans, e-remit, e-banking, etc. The recent perspectives and
practices of the banking corporate have been rapidly and vastly changing. In the fast
growing era of this sector this bank has not been able to attain relatively the expected
success. In this particular situation, a mini-research among the shareholders became
urgency of time so that it could give useful guidelines to the executives and Board of

The Institute of Chartered Accountants of Nepal 93


Suggested Answer - June 2018

Directors in achieving the corporate goals. This report has been prepared as a result of
this urgency. It has been prepared after a very systematic survey research.

d) The information can be organized systematically by categorizing it into different issue


based themes such as options for investment, desires to update the IT service, desires
to take risk, desire for changes, desires for no-risk, and so on. Then the information is
tabulated to ensure more systematic data for the report. The data can be organized and
analysed using tables, graphs, statistical tools such as percentage, mean, standard
deviation, etc. The irrelevant ideas/options are avoided from analysis. While
organizing the data, the extremely high level desires are arranged in one part, the
average level of remarks in the other, and the low level desires for changes and
challenges are organized in the different sub-section.

2. What are the prefatory parts of a report? Enumerate chronologically and


explain each of them in brief. 10

Answer:
Prefatory parts are front-end materials that provide key preliminary information in the
report so that readers can decide whether and how to read the report. Many of these
parts—such as the table of contents, list of illustrations and executive summary—are
easier to prepare after the text has been completed because they are based on the main
text of the report. Report prefatory parts are enumerated as:
1. Cover: Many companies have standard covers for reports, made of heavy paper and
imprinted with the company‘s name and logo. If a company doesn‘t have such covers,
you can usually find something suitable in a good stationery or office supply store.
Look for cover stock that is attractive, convenient, and appropriate to the subject
matter. Covers are typically labeled with the report title, the writer‘s name (optional),
and the submission date (also optional). Think carefully about the title. You want it to
be concise and compelling while still communicating the essence of the subject
matter.

2. Title Fly and Title Page: The title fly is a single sheet of paper with only the title of
the report on it. It adds a touch of formality, but it isn‘t really necessary, and it
consumes additional paper. The title page includes four blocks of information: (1) the
title of the report; (2) the name, title, and address of the person, group, or organization
that authorized the report (if anyone); (3) the name, title, and address of the person,
group, or organization that prepared the report; and (4) the date on which the report
was submitted. On some title pages, the second block of information is preceded by
the words Prepared for or Submitted to, and the third block of information is preceded
by Prepared by or Submitted by. In some cases, the title page serves as the cover of
the report, especially if the report is relatively short and is intended solely for internal
use.

3. Letter of Authorization and Letter of Acceptance: If you received written


authorization to prepare a report, you might want to include that letter of authorization
(or memo of authorization) in your report. If you wrote a letter of acceptance (or
memo of acceptance) in response to that communication, accepting the assignment

The Institute of Chartered Accountants of Nepal 94


CAP II Paper 6: Business Communication & Marketing

and clarifying any conditions or limitations, you might also include that letter here, in
the report‘s prefatory parts. In general, letters of authorization and acceptance are
included in only the most formal reports. However, consider including one or both if a
significant amount of time has passed since you started the project or if you do not
have a close working relationship with the audience. These pieces help make sure
everyone is clear about the report‘s intent and the approach you took to create it.

4. Letter of Transmittal: The letter of transmittal (or memo of transmittal), a specialized


form of a cover letter that is usually positioned right before the table of contents,
introduces your report to the audience. This piece says what you would say if you
were handing the report directly to the person who authorized it, so the style is often
less formal than the rest of the report. If your readers are likely to be skeptical of or
even hostile to something in your report, the letter of transmittal is a good place to
acknowledge their concerns and explain how the report addresses the issues they care
about. Also, if you need to convey sensitive information to selected audience
members, you can opt to include the letter in just those copies. Depending on the
nature of your report, your letter of transmittal can follow either the direct approach
for routine or positive messages or the indirect approach for negative messages. Open
by introducing the report and summarizing its purpose, with a statement such as ―Here
is the report you asked me to prepare on. . . .‖ The rest of the introduction includes
information about the scope of the report, the methods used to complete the study,
limitations, and any special messages you need to convey. You may also want to
acknowledge help given by others.

5. Table of Content: The table of contents indicates in outline form the coverage,
sequence, and relative importance of the information in the report. The headings used
in the text of the report are the basis for the table of contents. Depending on the length
and complexity of the report, you may need to decide how many levels of headings to
show in the contents; it‘s a trade-off between simplicity and completeness. Contents
that show every level of heading—down to the fourth or fifth level in detailed
reports—identify all the sections but can intimidate readers and blur the focus by
detracting from your most important message points. Where the detailed table of
contents could have dozens or even hundreds of entries, consider including two
tables: a high-level table that shows only major headings, followed by a detailed table
that includes everything. No matter how many levels you include, make sure readers
can easily distinguish between them.

6. List of Illustration: If you have more than a handful of illustrations in your report, or
if you want to call attention to them, include a list of illustrations after the table of
contents. For simplicity‘s sake, some reports refer to all visuals as illustrations or
exhibits.

7. Synopsis or Executive Summary: A synopsis is a brief overview of a report‘s most


important points, designed to give readers a quick preview of the contents. It‘s often
included in long informational reports dealing with technical, professional, or
academic subjects and can also be called an abstract. Because it‘s a concise
representation of an entire report, it may be distributed separately to a wide audience;

The Institute of Chartered Accountants of Nepal 95


Suggested Answer - June 2018

interested readers can then request a copy of the entire report.

3.
a) What is the communication process? Explain each of the processes. 5
b) Construct a resume for the position of Staff Accountant Analyst to
apply in one of the companies in the United States. In the resume
present the candidate's ability to contribute to a new employer. Make
sure that the employer could easily scan through the resume to find
sections of interest. 5
Answer:
a) Communication is ―the transmission of information and meaning from one individual
or group to another.‖ The crucial element in this definition is meaning.
Communication has as its central objective the transmission of meaning. The process
of communication is successful only when the receiver understands an idea as the
sender intended it. The communication process generally involves eight steps:

1. Sender has an idea: The form of the idea may be influenced by the sender‘s
mood, frame of reference, background, culture, and physical makeup, as well as
the context of the situation.
2. Sender encodes the idea in a message: Encoding means converting the idea into
words or gestures that will convey meaning. A major problem in communicating
any message verbally is that words have different meanings for different people.
That‘s why skilled communicators try to choose familiar words with concrete
meanings on which both senders and receivers agree.
3. The sender produces the message in a transmittable medium: with the appropriate
message to express an idea, the sender now needs a communication medium to
present that message to the intended audience. Media for transmitting messages
can be divided into oral, written, visual, and electronic forms.
4. Message travels over a channel: the medium over which the message is
transmitted is the channel. Messages may be sent by computer, telephone, letter,
or memorandum. They may also be sent by means of a report, announcement,
picture, spoken word, fax, or other channel. Because both verbal and nonverbal
messages are carried, senders must choose channels carefully. Anything that
disrupts the transmission of a message in the communication process is called
noise. Channel noise ranges from static that disrupts a telephone conversation to
spelling errors in an e-mail message. Such errors damage the credibility of the
sender.
5. The audience receives the message: if the channel functions properly, the message
reaches its intended audience. However, mere arrival at the destination is no
guarantee that the message will be noticed or understood correctly. Many
messages are either ignored or misinterpreted as noise.
6. Receiver decodes message: the person for whom a message is intended is the
receiver. Translating the message from its symbol form into meaning involves
decoding. Successful communication takes place only when a receiver
understands the meaning intended by the sender. Such success is often hard to

The Institute of Chartered Accountants of Nepal 96


CAP II Paper 6: Business Communication & Marketing

achieve because no two people share the same background. Success is further
limited because barriers and noise may disrupt the process.
7. The audience responds to the message: by crafting messages in ways that show
the benefits of responding, senders can increase the chances that recipients will
respond in positive ways. However, whether a receiver responds as the sender
hopes depends on the receiver (a) remembering the message long enough to act on
it, (b) being able to act on it, and (c) being motivated to respond.
8. Feedback travels to sender: the verbal and nonverbal responses of the receiver
create feedback, a vital part of the entire communication process. Feedback helps
the sender know that the message was received and understood. Senders can
encourage feedback by asking questions such as, ―Am I making myself clear?‖
and, ―Is there anything you don‘t understand?‖ Senders can further improve
feedback by delivering the message at a time when receivers can respond. Senders
should provide only as much information as a receiver can handle. Receivers can
improve the process by paraphrasing the sender‘s message.

b)
Rabindra Sing Yadav
137 Paneku Marga
Siphal, Kathamndu
Home: (0977) 01-4456789
Cell Phone: 9808967765
Email: rabindra_yadav@gmail.com

International Experienced international accountant and financial analyst with proven leadership, planning
negotiating, and intercultural communication skills. Demonstrated ability to improve process
Accounting efficiency and reduce operating costs.
Management
Staff Accountant Analyst
Inter-continental Imports: Kathmandu, Nepal
 Prepare accounting reports for wholesale giftware importer
Experience
 Adult financial transactions with suppliers in 8 countries of
SAARC
06/2010  Serve as project and team leader
to present  Created a computerized model to adjust accountants for
fluctuations in currency exchange rates
 Negotiated joint-venture agreements with major suppliers
in India and China
 Implemented electronic funds transfer for vendor
disbursements, improving cash flow and eliminating
payables clerk position

03/2007 Staff Accountant


to 06/2010 Royal Agricultural Chemicals: Biratnagar, Nepal
 Handled budgeting, billing, and credit-processing
functions
 Audited travel & entertainment expenses for 30-members
Nepali sales force
 Helped launch an online system to automated all
The Institute of Chartered Accountants of Nepal 97
accounting functions, improving reporting accuracy by 65
%
Suggested Answer - June 2018

Education Master of Business Administration with emphasis


08/2006 in International business
George Mason University, Fairfax, Virginia, USA
07/2004
Bachelor of Business Administration, Accounting
University of Texas, Austin, Texas, USA
06/2000
Chartered Accountant
The Institute of Chartered Accountants of Nepal

Special Skill Cultural  Fluent in English, Chinese and Hindi


 Extensive business contacts in South Asia
Technical  Proficient with a wide range of financial software
and systems, including Excel, Access, Microsoft
Dynamics, and SAP Business One

Reference Available on request

4. Write short notes on: (4×2.5=10)


a) Corporate social responsibilities
b) Ethnocentrism versus Cultural relativism
c) Audience-centered approach
d) Channel breakdowns

Answer:
a) Corporate social responsibilities
Corporate social responsibility (CSR) is an essential component of an ethical and
responsible business organization. It is a business movement in which commercial
organizations address the social issues identified through different sources, and run
the programs for social welfare. They invest certain amount of money so that their
business as well as the society where they have to survive can grow together.
Different infrastructural activities, educational programs, public health programs,
sports events etc. are conducted by the business organizations as their responsibility to
the society.

b) Ethnocentrism versus Cultural relativism


Ethnocentrism is the tendency to think of your own culture as the norm and to defend
your culture as the only valid worldview. It is a common phenomenon and it is found
in many cultures.

The Institute of Chartered Accountants of Nepal 98


CAP II Paper 6: Business Communication & Marketing

There is a simple way to explain ethnocentrism and how it works. Imagine a man who
drives his car every day and has been taught to stay on the right side of the street
when he does so. This man then goes to a country where people drive on the left side.
After he has returned home he goes to his friend's house for dinner and then he says,
―It is incredible, all the people drive in the opposite way. They all drive on the wrong
side!‖

Cultural relativism is somehow an opposite concept. We have seen that every culture
has the natural tendency to adopt a particular worldview. A cultural relativist is a
person who believes that every culture has to be understood in his particular context
and that cultures should not be thought as universally valid worldviews but as
particular expressions of a people.

c) Audience-centered approach
An audience-centered approach involves understanding and respecting the members
of audience and making every effort to get the message across in a way that is
meaningful to the audience. This approach is also known as adopting the ―you‖
attitude, in contrast to messages that are about ―me.‖ Learn as much as possible about
the biases, education, age, status, style, and personal and professional concerns of
your receivers. If you‘re addressing people you don‘t know and you‘re unable to find
out more about them, try to project yourself into their position by using common
sense and imagination. This ability to relate to the needs of others is a key part of
emotional intelligence, which is widely considered to be a vital characteristic of
successful managers and leaders. The more one knows about the people you‘re
communicating with, the easier it will be to concentrate on his/his needs—which, in
turn, will make it easier for them to hear your message, understand it, and respond
positively.

d) Channel breakdowns
Sometimes the channel simply breaks down and fails to deliver the message at all. A
colleague you were counting on to deliver a message to your boss might have
forgotten to do so, or a computer server might have crashed and prevented your blog
from updating. Everyone in an organization can help minimize barriers and
distractions. In any situation, a small dose of common sense and courtesy goes a long
way. Turn off that mobile phone before you step into a meeting. Don‘t talk across the
tops of other people‘s cubicles. Be sensitive to personal differences, too. For instance,
some people enjoy working with music on, but music is a huge distraction for others,
so use headphones. Finally, take steps to insulate yourself from distractions. Don‘t let
messages interrupt you every minute of the day. Instead, try to set aside time to attend
to messages all at once so that you can focus the rest of the time.

The Institute of Chartered Accountants of Nepal 99


Suggested Answer - June 2018

Part B : Marketing

The Institute of Chartered Accountants of Nepal 100


CAP II Paper 6: Business Communication & Marketing

Roll No……………. Maximum Marks – 50


Total No. of Questions - 4 Total No. of Printed Pages -6
Time Allowed – 3 Hours
Marks
All questions are compulsory.
Section -'B'

5. Read the following case carefully and answer the questions given below: (45=20)
XYZ supermarket stands out as one of the most valuable brands in Nepalese supermarket
industry in a very short span since its inception. The first store was launched in one of the
finest mall located in Kathmandu valley, covering an area of 11,000 Sq. Ft. It has been
recognized for its innovation in introducing super marketing to the Nepalese market with
a very different and organized approach. XYZ supermarket has all the material including
groceries and vegetables for the people to shop at one place it stands out as being the first
in Nepal to provide the VALUE AND SAVINGS concept to its shoppers. The XYZ
includes the following features.
 Easy shopping: Easy to search items at all pages
 Big save: every item has shopping mall and wholesale mart (B2B). Shopping mall is
for consumer. Wholesale mart is for merchants or bulk or discount shopping
 Easy to set up your store: set up a new store in our shopping mall by suing our
wholesale mart, valuable goods source. You do not have to spend time to find
wholesaler.
 low cost AD: advertise your goods or website at low cost (big size slide head banner
only), visit low cost AD page
 You can sell without fee: why pay so much selling fee? Advertising fee is free as long
as you buy from XYZ wholesale network visit free AD page.
 Easy access: easy access to xyz.com through Google, Bing, Yelp, Face book,
LinkedIn
XYZ department store is planning a major expansion drive in the valley after three years
of successful run in Kathmandu. The retail store is planning to open 25 stores within
2018. XYZ has become one of the well-known retailer stores. "Within the first three
years of its operation, XYZ has evolved as one of the most trusted brands in the Nepali
market. We have been receiving encouraging response from our consumer and this has
prompted us to extend our service areas, "said Antim Ranjit, marketing and branding
manager at XYZ. He added that the company had identified probable places for its
expansion. "XYZ has been known for the availability of quality foods and grocery
product. Therefore, we will maintain our focus on this segment," Ranjit said.
The new store will feature around 25,000 varieties of product in the initial stage. Apart
from this, the store will have a separate compartment for meat from where people can
buy mutton and chicken.
An ATM lounge and food stalls in the 'Thela" concept will be other added attractions.
"We will put on our best effort to make sure that the store becomes successful in catering

The Institute of Chartered Accountants of Nepal 101


Suggested Answer - June 2018

to their needs, ' said Ranjit. After having selected a smaller area for the new stores, the
company said that it was planning to expand in every small nook and corner of the
Kathmandu valley to cater to the demand of people from all economic groups. 'Compared
to our previous two stores, we have chosen a smaller space, "said Ranjit. XYZ is also
planning to open in other locations. "We will have relatively larger stores in these
places," Ranjit said. The company has so far distributed 45,000 privilege cards to its loyal
customers and the figure is increasing daily. Even though XYZ refused to disclose its
investment in the upcoming projects, it is said that it would be spending substantially on
these world class stores.
a) What are the major retail marketing strategies adopted by the XYZ?
b) Explain the retailing processes of XYZ.
c) State and explain the major features of XYZ. Relate them to product mix of retail
marketing.
d) What are the retail marketing mixes that the XYZ should implement to gain
competitive advantage?
Answer:

a) The major retail marketing strategies adopted by the XYZ is multi-channel marketing. It
is a chain store which has operated supermarket, department store, online store, shopping
mall, wholesale mart etc. in various locations of Kathmandu Valley. Similarly,
adoption of different and organized approach for innovation is the another strategy of
XYZ. Again, value and savings concept to its shoppers is the customer oriented strategy
adopted by the company. The consumers were provided by the easy access at one place
for purchase. XYZ facilitates the online shopping facilities through Google, Bing, Yelp,
Face book, LinkedIn. The firms also created the value and adoption of saving concept for
customer. They also provided the bulk buying discount facilities to their regular
customer. Also, the supermarket provided the wide product varieties, about 25000
product varieties.

b) So far, the retailing processes of XYZ is concerned, it has adapted two different
process, i.e. offline store and online store. In offline store, customers directly pay visit
to stores to buy their needs under single roof. Customers can make a choice by
comparing among the varieties of available products. Physical presence of customer in
the store is one of the key features of store retailing. Regarding offline store, they have
operated super market, department store and shopping mall where retail buying process
is quite different because of method of operation of retail business.. Secondly, online
shopping through mart‘s webpage is another retail buying process. Under this retailing
process customers are allowed to search all items available at all pages where they can
save their time by avoiding personal visit at stores. In online shopping, consumers
access to xyz.com through Google, Bing, Yelp, Facebook and LinkedIn.

The Institute of Chartered Accountants of Nepal 102


CAP II Paper 6: Business Communication & Marketing

c) The XYZ has been operating the retail business from different perspectives where
customer can purchase from both personal visit to store for buying and online buying
without visiting actual store. With respect to personal visit the store has provided value
and saving concept to customer by making available all merchandises at one place. The
store has retail and whole mart where customer can save by bulk and discount shopping.
This has also been characterized by easy setting up store at mall for B2B purpose.
Online shopping has been characterized for easy access through different search engine,
easy shopping as easy to search all items at all pages and advertisement facilities or free
ad page on store‘s blog. The store has also equipped with wide product varieties.
The features discussed above can be reflected in terms of product mix. Retailers
provide stores full of products to suit every consumer‘s needs to meet expectations of
varied customers. It offers all the materials including groceries and vegetables, meats.
It also provides ATM lounge and food stalls in Thela concept. The store has also
separate mart for end consumer and business consumer. Besides company‘s webpage is
also equipped with easy shopping as it is easy to search all items.

d) Retail is a service business. Thus, it should include all the seven P‘s while designing
marketing mix to gain competitive advantage. Product, place, price, promotion, people,
process and physical evidence all elements are fundamentally important to retail
management to gain competitive advantage.
In the case, XYZ has operated supermarket, department store, shopping mall, wholesale
mart and online store. First of all, there must be the same retail format and uniformity in
retail operation. If they use different retail format, at that time it is very difficult to
positioning their retail business.
As they are extending their outlets within Kathmandu valley, locations of stores could
be the major decision for its marketing mix. The store should target the area where
proximity of customer is very high. Besides distribution channels should be decided
very carefully, that ease store to manage inventory level smoothly.
Finally, the retailer has already adapted selling through internet, which is being
considered as very important promotional tool. This is the good method of marketing
communication that maintains the link with customers. However, the store should not
rely only on internet as promotional tool.

6. What is product line strategy? Discuss the trading up and trading down
strategies with suitable examples. (4+6=10)

Answer:

If any company offers more than one product items which are closely related, is popularly
known as product line strategy. For example, Nebico Biscuits offers varieties of biscuits
such as Digestive, Marie, Coconut, Glucose, Thin arrowroot, Salty, Creamy biscuits etc.

The Institute of Chartered Accountants of Nepal 103


Suggested Answer - June 2018

Companies make decisions that concern either adding new items in existing product lines,
deleting products from existing product lines, or adding new product lines. Another aspect
relates to upgrading the existing technology either to reduce the product costs or to improve
quality, for stretching (downwards, upwards, or both ways), or line filling. Among them,
trading up and trading down strategies are discussed as follows:
Trading Down or downward Line Stretching: If any company adds new product item in
their product line which is low price low quality in comparison to their existing products, it
is known as trading down or downward line stretching strategy. Companies sometimes
introduce new products with an objective of communicating an image of technical
excellence and high quality and locate at the upper end of the market. Subsequently, the
company might stretch downwards due to competitor‘s attack by introducing a low-end
product in response to competitive attack, or a company may introduce a low-end product
to fill up a vacant slot that may seem attractive to a new competitor. Another possibility is
that market may become more attractive at low-end due to faster growth rate. Downward
stretch sometimes poses risks. Low-end competitors may attack by moving into high-end,
or for a prestige-image company introduction of a low-end model may adversely affect its
product-image.
For example, Tata Nano car offered to price sensitive customer by Tata company is best
example of trading down. Similarly, CG Foods Private limited, producer of popular brand
Waiwai has added Mama instant noodle which is low price low quality in comparison to
Waiwai is the best example of trading down strategy.
Trading Up or Upward Stretching: If any company adds new product item in their product
line which is high price high quality in comparison to their existing products, it is known as
trading up or upward line stretching strategy. In this situation, companies operating at low-
end may opt to enter high- end because of better opportunities as a result of faster market
growth, or the need to create an image of full line company. There may be certain risks
associated with upward line stretching. These may include prospective customers‘
perceptions that the newcomer in the high-end category may not produce high-quality
products, or competitors already well-established in the high-end market may retaliate by
introducing items in the low-end of the market.
For example, Maruti Udyog introduced its medium-priced models such as Maruti Zen,
Maruti Esteem, Wagon R, Alto, and Swift after it had entered the market with its low-end
Maruti 800 and Maruti Omni. Toyota introduced its Lexus luxury car as a standalone
product (with no outward link to Toyota) for just this very reason. It did not want it to be in
any way affected by Toyota‘s no-doubt superb, but mass market image. Similarly, a
Tribhuvan University affiliated college which has been offering BBS is now going to offer
BBA, BHM or BBM with premium fee structure. It is called trading up strategy because
BBA, BHM or BBM is considered as high price high quality program in comparison to
BBS.

7.

The Institute of Chartered Accountants of Nepal 104


CAP II Paper 6: Business Communication & Marketing

a) ―Understanding buyer‘s behavior is a major input for marketing


strategy.‖ Justify this statement. 5
b) Give the meaning of marketing intelligence system and also point out
the sources of marketing intelligence system. 5

Answer:
a) Behavior shown by consumers while they are purchasing, consuming or disposing any
product or services is called buyer‘s behavior. It involves searching for, evaluation of,
purchase and using the product. Behavior also covers the post purchase behavior of
product – i.e. consumer satisfaction or dissatisfaction where it involves disposal of
product.
On other hand, the aim of marketing is to meet and satisfy target customers‘ needs and
wants better than competitors. In this respect, understanding buyer‘s behavior is most
important for every marketer as it is the study of how individuals, groups, and
organizations select, buy, use and dispose of goods, services, ideas, ore experiences to
satisfy their needs and wants. Understanding buyer‘s behavior provides clues for
improving or introducing products or services, setting prices, devising channels,
crafting messages, and developing other marketing activities. Furthermore, marketers
are always looking for emerging trends that suggest new marketing opportunities. And
studying consumer behavior would be the input for marketing strategy for improvement
by understanding issues such as:

 The psychology of how consumers think, feel, reason, and select between different
alternatives (e.g., brands, products, and retailers);
 The psychology of how the consumer is influenced by his or her environment (e.g.,
culture, family, signs, media);
 The behavior of consumers while shopping or making other marketing decisions;
 Limitations in consumer knowledge or information processing abilities influence
decisions and marketing outcome;
 How consumer motivation and decision strategies differ between products that
differ in their level of importance or interest that they entail for the consumer; and
 How marketers can adapt and improve their marketing campaigns and marketing
strategies to more effectively reach the consumer.

b) Marketing intelligence is the systematic collection and analysis of publicly available


information about consumers, competitors, and developments in the marketplace. The
goal of marketing intelligence is to improve strategic decision making by understanding
the consumer environment, assessing and tracking competitors‘ actions, and providing
early warning of opportunities and threats.
There are two sources of marketing intelligence. They are formal and informal.
Salesmen, intermediaries, information centers, experts, private agencies etc. are formal
sources and newspapers, trade journals, magazines, employees etc. are informal

The Institute of Chartered Accountants of Nepal 105


Suggested Answer - June 2018

sources.
1. Formal Sources: Formal sources of marketing intelligence system includes
middlemen, sales force, marketing information Centre, vacancy announcement, private
agencies, stakeholders, and international sources. The formal sources of marketing
intelligence are as follows:
a. Intermediaries: The intermediaries include dealers, distributors, wholesalers,
retailers etc. Middlemen deal with many competitors‘ products at one time.
Therefore, on the one hand, middlemen know about the policy and strategy of new
products, promotion, and pricing etc. of the different companies. And on the other
hand, they have adequate information about the market because of their direct link
with customers. They can be motivated to provide vital market information.
b. Sales-forces (Salesmen): They spot and report new developments in the market-
place. Organizations train and motivate them for marketing intelligence purposes.
Sales forces provide useful information to the business organization.
c. Marketing Information Centre: Organizations can establish a marketing
information centre for marketing intelligence. Then to receive the information
secretly, staff of this centre are sent to the competitors‘ organization. Those staff
will send the information of the competitors about their policy and strategy secretly
and regularly to the centre. They are also called business spy.
d. Vacancy Announcement: It is difficult to receive information openly from
competitors‘ staff. But the competitors‘ staff may also be the important source to get
information. So, the organization announces the vacancy where the competitors‘
staff also applies for the job. These vacancies may be both imaginary and real. In
this way, organization receives the important secret information through the
interview of the applicants.
e. Private Agencies: Private agencies may also be the sources to receive information.
Some of the private agencies collect the secret information regularly and provide
them to the interested organization. Any organization can buy the secret information
from such agencies.
f. Stakeholders: Stakeholder includes customers, government, suppliers, middlemen
and staff. They also provide information about the market. For example, from
customers organization can know about product‘s quality, feature, taste, price,
buying capacity etc. From suppliers, one can know about raw material‘s price,
technology, alternative raw materials and competitors. From government it can
know about product‘s demand, population, inflation, per capita income, etc. From
middlemen one can know about distribution cost, distribution capacity and customer
response, etc. Similarly, through staff organization can know about administrative
problems, ability to implement decision and disability, etc.
g. Specialist/Expert: A business organization may also appoint specialists for
collecting market information. Such specialists are skilled in scanning market

The Institute of Chartered Accountants of Nepal 106


CAP II Paper 6: Business Communication & Marketing

environment.
h. International Sources: Information can be obtained through international
organizations and the articles written by the expert or critics. Similarly, articles
published by WTO, SAFTA, INDP, UNDP, UNIDO, ILO, GATT, World Bank can
also be the sources of information.
2. Informal Sources: The informal sources of marketing intelligence are as follows:

Reading newspapers, magazines, trade journals etc.
 Having interactions with production manager, financial manager, personnel manager,
research and development manager, sales manager, employees etc.
 Having interaction with customers, intermediaries, other concerned persons etc.
8. Briefly explain the following: (5×2=10)
a) Any two features of marketing environment
b) Price discrimination
c) E-commerce
d) Distribution
e) Sales promotion

Answer:
a) Any two features of Marketing environment

Marketing environment consists of surrounding of the firm which affects the marketing
decision including policies and strategies of a firm. Marketing environment is characterized
by following facts.
i) It includes all the factors and forces which influence marketing , sales and consumer
satisfaction.
ii) It is always dynamic in nature.

b) Price discrimination

If any marketer sets different price for different customer for the same product. It is called
price discrimination. It is a flexible pricing policy where different price is determined for
same product on the basis of types of customer, time, quantity, geographical area, season
etc.

c) E-commerce

E-commerce is internet-based marketing. It means that the company or its website offers to
transact or facilitate the selling of products and services online. It has given rise in turn to e-
purchasing and e-marketing.

d) Distribution

The Institute of Chartered Accountants of Nepal 107


Suggested Answer - June 2018

Distribution includes all the various activities the company undertakes to make the product
accessible and available to target customers. It involves the marketing channel and physical
distribution. Marketing channel makes the product available to the customers. Physical
distribution makes the product accessible to the channel members and customers

e) Sales promotion
Sales promotion consists of all promotional activities other than advertising, personal
selling and publicity that stimulate short-term or temporary market demand for products.
The basic purpose is to stimulate on the spot buying by people through short-term
incentives. Incentives such as a samples, price concessions, and prize contests are offered to
customers to encourage buying.

The Institute of Chartered Accountants of Nepal 108


CAP II Paper 6: Business Communication & Marketing

Specific Comments on the performance of the students


Batch: - June 2018
Level: - CAP-II
Subject: Business Communication
Question No. 1
Survey questions follow a format. Most of the student doesn't know this. Students are
not clear on objectives and problem statements.
Question No. 2
Students failed to notice the word 'prefatory parts' of the report. so, 98% students
answered incorrectly.
Question No. 3
Questions are easy and clear. Answers are satisfactory. But very few of them
misunderstood the concept of CV. Instead of writing CV they have written job
application.
Question No. 4
'Channel breakdown' has confused many students. The question is about
communication channel breakdown but many students have understood it as
'organizational breakdown'.
Subject: Marketing
Question No. 5
This part is case analysis, where majority of the candidates did not get the main theme
of the case as well as theory of case, policy matter.
Question No. 6
This is the marketing strategy where majority of the students did not follow the
concept because they did not well-practice in their basic books. Most of the students
have no knowledge about trading up and trading down strategy.
Question No. 7
This part of questions focused on conceptual knowledge where students provided
reasoning while answering the questions. Students have no knowledge about the
understanding buyer's behavior and about marketing intelligence system.
Question No. 8
Here short answers questions were asked and all most all students responded properly.

The Institute of Chartered Accountants of Nepal 109


Suggested Answer - June 2018

Paper 7:

Income tax & VAT

The Institute of Chartered Accountants of Nepal 110


CAP II Paper 7: Income tax & VAT

Maximum Marks - 100


Total No. of Questions - 8 Total No. of Printed Pages -16
Time Allowed - 3 Hours
Marks
Attempt all questions. Working note should form part of the answer.
1. Mustang Cashmere Manufacturing Co. Pvt. Limited is engaged in manufacturing and sale
of high grade cashmere clothes. It provides employment opportunity to 650 Nepalese
people throughout the year. Following are the extracts of the Income Statement of the
company for the year ended on Ashadh 31, 2074.
Particulars Amount Rs.
Income:
Export Sales 6,000,000.00
Domestic Sales 4,000,000.00
Dividend Received (Net of Tax) 150,000.00
Rent Income (Related with Business) 50,000.00
Total Income 10,200,000.00
Expenditure:
Cost of Materials Consumed 3,000,000.00
Manufacturing Expenses 500,000.00
Employee Cost 1,000,000.00
Selling and Administrative Expenses 1,500,000.00
Interest and Bank Charges 500,000.00
Exchange Loss 250,000.00
Loss on Sale of Depreciable Assets 300,000.00
Total Expenditure 7,050,000.00
Operating Profit 3,150,000.00
Less: Provision for Bonus 300,000.00
Profit Before Tax 2,850,000.00
Additional information:
a) Cost of materials is consumed in the ratio of sales.
b) Exchange loss includes Rs 100,000.00 against revaluation of creditors at the year-end.
c) Selling and Administrative Expenses include Rs 70,000.00 donation given to Prime
Minister Disaster Relief Fund and Rs 300,000.00 given for construction of school.
d) Out of total provision for bonus, Rs 200,000.00 was distributed to the employees till
the time of filing income tax return. It has been decided by the management not to
pay the undistributed portion.
e) You are given the following information in regard to some of the expenses:
i) Rs 50,000.00 included in employee cost as staff welfare is personal expenses of
directors.
ii) Manufacturing expenses include Rs 30,000.00 for electricity bill of previous year.

The Institute of Chartered Accountants of Nepal 111


Suggested Answer - June 2018

iii) Selling and Administrative Expenses include Rs 10,000.00 for business


promotion which is not related to business.
Based on the above information, Compute tax liability of the Company.
Answer:
I. Determination of Tax Rates:
As per section 11 of Income Tax Act, 2058 below are the exemptions available to
Manufacturer cum Exporter of cashmere clothes being a Special Industry
i) The Company employs altogether 650 Nepali employees‘ throughout the year. So, it
is entitled to effective tax rate of 90% of applicable tax rate as per section 11(3)(Ka)
for employing 300 or more Nepali Employees
ii) The Company has export sales. So, 25% tax rebate for export Income only as per
section 11(3) (3nga), i.e. effective tax rate is 75% of applicable tax rate
Selection of Tax Exemption
As per section 11(5), if more than one tax exemption is available for the same income,
then only one tax exemption should be availed as per choice of the Taxpayer.
Hence, the Company should choose one of the exemptions mentioned below:
Exemption Effective Tax Rate Effective Tax Rate on
for Export Income Domestic Income
As a result of employing 300 or more 90% of 20% = 18% 90% of 20% = 18%
Nepali Nationals
Concessional rate as a result of export 20% - 20%*25%= 20% -0= 20%
15%
Selection of Exemption (highest rebate 15% 18%
exemption for particular Income)

II. Computation of Taxable Income and Tax Liability of Mustang Cashmere


Manufacturing Co. Pvt. Ltd. for the year ended Ashadh 31, 2074
Particulars Export Domestic Total (Rs.)
Income (Rs.) Income (Rs.)
Income From Business
Export Sales 60,00,000 - 60,00,000
Domestic Sales - 40,00,000 40,00,000
Dividend - - -
Rent Income - 50,000 50,000
Total Income 60,00,000 40,50,000 1,00,50,000
Allowable Expenses:
Interest Expenses (sec. 14) 3,00,000 2,00,000 5,00,000
Cost of Trading Stock (Sec. 15) 20,82,000 13,88,000 34,70,000
Employee Cost (Sec. 13) 5,70,000 3,80,000 9,50,000
Selling and Administrative 6,72,000 4,48,000 11,90,000
Expenses (Sec. 13)
Exchange Loss 90,000 60,000 1,50,000
Loss on Sale of Depreciable - - -
Assets

The Institute of Chartered Accountants of Nepal 112


CAP II Paper 7: Income tax & VAT

Provision for Bonus 1,80,000 1,20,000 300,000


Total Deductions 38,94,000 25,96,000 64,90,000
Assessable income from 21,06,000 14,54,000
Business
Less: Contribution to PM Relief 42,000 28,000 70,000
Fund
Taxable Income 20,64,000 14,26,000
Tax Liability 309,600 256,680 566,280
(@15% for export income and
@18% for domestic income)

Working Notesː
1. Costs of materials consumed have been allocated at the rate 60:40 and in lack of
further information other costs are also allocated in same ratio in the given solution.
2. Cost of Trading Stock
Particulars Amount (Rs.)
Cost of Materials Consumed 30,00,000
Manufacturing Expenses as per Income Statement 5,00,000
Less : Prior Period Electricity Expenses not allowed 30,000
Net Allowable Cost of Trading Stock 34,70,000
Cost of Trading Stock for Export Income – 60% of Total Expenses 20,82,000
Cost of Trading Stock for Domestic Income – 40% of Total Expenses 13,88,000
3. Employee Cost
Particulars Amount(Rs.)
Employee Cost as per Income Statement 10,00,000
Less : Personal expenses of directors not allowed 50,000
Net Allowable Employee Cost 9,50,000
Employee Cost for Export Income – 60% of Total Expenses 5,70,000
Employee Cost for Domestic Income – 40% of Total Expenses 3,80,000
4. Selling and Administrative Expenses
Particulars Amount(Rs)
Selling and Administrative Expenses as per Income Statement 15,00,000
Less : Donation given to School not allowed & PM Relief fund 370,000
Less : Business promotion Expenses not related to business 10,000
Net Allowable Selling and Administrative Expenses 11,20,000
Selling and Administrative Expenses for Export Income – 60% 6,72,000
Selling and Administrative Expenses for Domestic Income – 40% 4,48,000
5. Exchange Loss
Particulars Amount (Rs.)
Exchange Loss as per Income Statement 2,50,000
Less : Exchange Loss on revaluation of creditors 1,00,000
Net Allowable Exchange Loss 1,50,000
Exchange Loss for Export Income – 60% of Total Expenses 90,000

The Institute of Chartered Accountants of Nepal 113


Suggested Answer - June 2018

Exchange Loss for Domestic Income – 40% of Total Expenses 60,000


6. Loss on sale of Assets (1 mark for this note)
As per Income Tax 2058, the realized value of sold assets has to be adjusted in the
depreciation base amount of respective group of assets Schedule 2 Section 2(3)]. It is
assumed that the total realized value has been adjusted accordingly and hence loss on sale
of assets has not been claimed as expenses.

7. Provision for Bonus


As per Income Tax Act, 2058; a company must follow accrual basis of accounting. As
such, bonus expenses incurred during the year is deductible during the year, though the
related distribution is included in income of the employee during the Income Year when
the amount is distributed as a result of application of cash basis of accounting. As per the
latest circular by IRD, any undistributed bonus forms part if income of the person in the
immediately following Income Year (CAP II students are not expected to study circulars
issued by IRD)
Particulars Amount (Rs.)
Provision for Bonus as per Income Statement 3,00,000
Bonus for Export Income – 60% of Total Expenses 1,20,000
Bonus for Domestic Income – 40% of Total Expenses 80,000

8. Depreciation is not deducted in lack of information.

2. Mrs. Kriti is an employee in Nepal Bank Ltd. Besides employment income, she has the
following transactions for Income Year 2073/74. Sort out the income from investment
and tax liability for the year assuming that 0% and 15% tax slabs are already over by
income from employment. 10
S. No. Particulars Amount (Rs)
1 Net gain on disposal of listed shares 100,000.00
2 Purchase of New listed shares 600,000.00
3 Income from Taxi 400,000.00
4 Gain on sale of gold 100,000.00
5 Interest from savings deposit from 'A' class bank 4,000.00
6 Interest from fixed deposit from 'A' class bank 40,000.00
Net gain from disposal of ancestral house (owned and
7 resided since Bikram Sambat 2060) 5,000,000.00
8 Royalty from music video 100,000.00
9 House rent 300,000.00
10 Shares transaction commission expenses 5,000.00
11 Salary to taxi driver and other taxi related expenses 200,000.00
12 Music video expenses 20,000.00
13 House repair and maintenance expenses 20,000.00
10
Answer
S. Particulars Amount(Rs.)

The Institute of Chartered Accountants of Nepal 114


CAP II Paper 7: Income tax & VAT

No.
1 Inclusion 2,00,000.00
1.1 Net gain on disposal of shares 1,00,000.00
1.2 Royalty from music video 1,00,000.00
2 Deductions 20,000.00
2.1 Music Video expenses 20,000.00
3 Assessable income from investment 1,80,000.00

Calculation of Tax liability:

Royalty income 80,000×25% 20,000.00


Income from disposal of listed shares 1,00,000×5% 5,000.00
Tax liability 25,000.00

Working Notes:

1) Purchase of new listed shares forms part of investment income, but will only be
considered while computing net gain from disposal of non business chargeable asset.
2) Income from taxi is taxed at a fixed presumptive tax. So, this is not an investment
income.
3) Gold is personal asset and not an investment asset and is not taxed on gain from it.
4) Interest paid to natural persons from a bank and financial institution is a final
withholding income. So, interests from savings and fixed deposits are not included.
5) House rent income of a natural person not related to business is a final withholding
income, so, not included in income from investment.
6) Shares transaction commission expenses is already adjusted while calculating net gain
on disposal of listed shares, so, no further deduction required.
7) Expenses related to final withholding incomes are not deductible for calculating
assessable income.
8) Disposal of ancestral house owned and resided for 10 or more years is not an
investment activity. So, it is not included in income from investment.
3.
a) Bottlers Nepal Ltd. Kathmandu has paid consulting fees Rs. 30 million in the year
2074/75 to International Consultancy Group, New York (ICGN). As per the
agreement, the consultant should deploy at least 2 employees at the project site
located at Butwal, Province No. 5 throughout the year. Can we define ICGN as a
permanent establishment in Nepal? What about the tax withholding implications in
the following situations: 5
i) If the company is registered in Nepal through its local agent and payment was
made to local agent.
ii) If the company is not registered in Nepal and payment was transferred directly to
their bank maintained in New York.
b) Mr. Ramji Paneru retired in 2075 Baishakh end from 30 years of his service with
Nepal Timber Corporation and received lump sum retirement payments as follows: 5
i) Payment from approved retirement fund Rs. 1,675,852.00.

The Institute of Chartered Accountants of Nepal 115


Suggested Answer - June 2018

ii) Payment from unapproved retirement fund Rs. 675,800.00 to which his
contribution was Rs. 511,256.00. This payment is related only with the period
after Income Tax Act, 2058.
You are required to calculate the tax withholding on these payments along with
reason therefor.
Answers:
a) As per Section 2(Ka Nga) a resident person includes a foreign permanent
establishment of a non-resident person situated in Nepal. Further, section 2(Ka Na)
defines permanent establishment that also includes the place: "one or more than one
place in any country where any person has delivered technical, professional or
consultancy service through an employee or in any other manner for more than ninety
days at one or several times in a period of any twelve months". As per these
definitions, International consultancy group is considered a permanent establishment
in Nepal.
i) If payment was made in Nepal, as since the entity is registered for tax purpose; it
must register for VAT as well. While making payment of service fee to VAT
registered service provider, tax shall be withheld @ 1.5% of payment as per
proviso (gha) of Sec. 88 (1) of the Act. If the permanent establishment is not
registered for VAT purpose, the tax shall be withheld @ 15% as per Sec. 88 (1)
(Alternatively, students may also assume that the payment of consulting fee is not
part of independent consulting contract but part of supervisory activities in
relation to construction contract when Sec. 89 is applicable and TDS @ 1.5%
shall be deducted as per Sec. 89 (1) in both cases when there is VAT registration
or not)
ii) If amount was transferred directly, tax rate shall be 15 % as per section 88(1). As
per Income Tax Act, the PE is required to register in IRD for the transaction
including VAT registration. So, the first option is in compliance of Income Tax
Act.
b) TDS Calculation on retirement Payment to Mr. Ramji Paneru

Approved Unapproved
Particulars Retirement Fund Retirement Fund
Payment 1,675,852 675,800
Less Accrued Prior 2045.02.01 to 18 Chaitra
2058 (13.88/30 Yrs.) 775,361 -
Less 50% Deduction: higher of 500,000 or
50% of (1,675,852-775,361) 500,000 -
Less employees contribution - 511,256
Taxable Amount 400,491 164,544
Rate of Tax 5% 5%

TDS Amount 20,024 8,227.20


4.
a) Shreedhar Kumar is appointed on 2066.04.01 as Accounts Officer of Nepal Telecom
Limited with salary structure of Rs. 25,000.00 - 500.00(6)-1,000.00(12). He has

The Institute of Chartered Accountants of Nepal 116


CAP II Paper 7: Income tax & VAT

received interest free home loan Rs. 1,624,000.00 from the employer. Assume market
rate of interest 9% and make other practical assumptions as necessary. Calculate
monthly withholding tax to be deducted from his salary in Income Year 2074/75. 5
b) AB (Pvt) Ltd has made an agreement with ABC (Pvt) Ltd to arrange the cash flow for
its transactions. Chairman of AB (Pvt) Ltd. is also a representative board member of
ABC (Pvt) Ltd. In Shrawan 1, 2072, loan amounting Rs. 50 million had been
provided by the ABC to the AB. Interest rate mutually agreed was 10 percent. Both
the companies are located at Kathmandu. Tax Officer denied the interest expenses of
Rs. 5 million claimed by AB (Pvt) Ltd. on the ground that this transaction is a related
party's transaction. Answer whether the Tax Officer's assessment is correct.
c) Mr. A and Mr. B contributed Rs. 4 million and Rs. 6 million respectively and
deposited into a common bank account to be used to purchase a plot of land and to
resale the same in the future. After 6 months of depositing the money into the bank
account, they purchased a plot of land for Rs. 9 million and got the land registered in
their joint name. Expenses on registration and commission at the time of purchasing
the land are amounted to Rs. 1 million. Later, they sold the plot of land for Rs. 17.2
million. Also, they paid Rs. 1.2 million as sales commission and other incidental
expenses pertinent to the disposal. The bank has given interest @ 12% per annum on
such deposit.
Compute the amount to be included as income in the hand of Mr. A and Mr. B. 5
d) Mr. Ram Prasad, Nepali resident with single status for taxation, had submitted the
following details of income from business and investment as self-assessment of his
tax liability for the Income Year 2073-74:
Amount (Rs.)
Country Net Assessable Income Tax paid
Nepal 500,000.00 25,000.00
USA 200,000.00 65,000.00
UK 300,000.00 0
Canada 500,000.00 80,000.00
Total 1,500,000.00 170,000.00
Compute any unabsorbed foreign tax credit of Mr. Ram Prasad during
the assessment. 5
Answers:
a) Calculation of monthly withholding tax of Mr. Shreedhar Kumar
(Single Status)
Particulars Amount Remarks
(25000+500*6+1000*2)*
Basic Salary 360,000 12
at the rate of 9% on the
Deemed Income on Interest free loan average loan outstanding
146,160 during the year
Assessable Income 506,160
Taxable Income 506,160
First Rs. 350,000 @1% 3,500

The Institute of Chartered Accountants of Nepal 117


Suggested Answer - June 2018

Next Rs. 100,000@15% 15,000


Balance Rs. 56,160 @25% 14,040
Total Tax Liability for the year 32,540
Monthly Deduction required 2,711.67
Social Security Tax 291.67
Income Tax 2,420

Alternatively,
a. The student may assume couple assessment, in which case, the tax liability and monthly
deduction will be as follows:
First Rs. 400,000 @ 1% - 4,000
Next Rs. 100,000 @ 15%- 15,000
Balance Rs. 6,160 @25%- 1,540
Total tax liability – 20,540
Monthly deduction- 1711.67
SST- 333.33
Income Tax- 1378.33
b. The students may make assumptions of employers‘ contribution to ARF @ 10% of basic
salary and grade and 20% of basic salary and grade deposited in the approved retirement fund
account of the employee. Students further assume that festive allowance equal to one month‘s
basic salary and grade is provided by the employer during the year.

b) Section 2(Ka Na) defines the associated person, to any one or more than one person
or group of persons who act as per the intention of each other, it also includes: any
entity which by itself or jointly with any other person related with it or with an
assisting entity or any other person or entity related with such assisting entity controls
50 % or more of the income, capital or voting right of any entity or derives benefits
therefrom.
If associated person is established as per the definition, then tax officer can, by
issuing a notice in writing, distribute, appropriate or allocate the amounts to be
included or deducted in computing the income between those persons in such a
manner as to reflect taxable income or the payable tax that could be set for them as
per Section 33 of Income Tax Act.
As per these provisions of Income Tax Act, the ground of the tax officer is wrong; the
associated person could not be established due to lacking of holding percentage on
AB Pvt. Ltd. by the ABC Pvt. Ltd. But the tax officer should make assessment of
TDS @ 15 % on interest payment as per Section 88(1). AB Pvt. Ltd should deduct the
TDS of Rs. 750,000 (5 million*.15) on the interest payment.
(Alternatively students can solve the problem as above citing the provisions of Sec. 14
of the Act and demonstrates that the provisions does not restrict the deduction of
interest expenses merely on the basis they are associated, even when the relation of
associated persons are established)

c)
(Rs. in Million)

The Institute of Chartered Accountants of Nepal 118


CAP II Paper 7: Income tax & VAT

Particulars Amount Remarks


Expensesː
MA Total Mr. A Mr. B
Cost of Plot of Land 9
Other Expenses 1
Total cost of Land (A) 10 4 Apportioned
6 on the basis of 2ː3

Sales Income 17.2


Lessː Expenses 1.2
Net sales Income (B) 16 6.4 9.6
Income (B-A) 6 2.4 Apportioned
3.6 on the basis of 2ː3
Apart from aforesaid income, Mr. A and Mr. B have received interest income of
Rs.600,000 (10,000,000×12%×6/12) as the bank interest. This interest income amount
also needs to be shared between Mr. A and Mr. B in the ratio of 2ː3 as followsː
Mr. A ː Rs.240 thousand, Mr. B ː Rs.360 thousand,
The above interest income is subject to deduction of applicable TDS which is final.
d)

Computation of Taxable Income of Mr. Ram Prasad for FY.2073-74


Calculation of Average Tax Rate in Nepal:
Tax liability
First 350,000 0% -
Next 100,000 15% 15,000
Rest 1,050,000 25% 262500

Therefore, Average Tax Rate = Tax liability before foreign tax credit/taxable income
* 100
(277,500/1,500,1000)×100
= 18.5%
Computation of foreign tax credit for the FY 2073-74
Countries Foreign Ass. Tax Avg. rate Maximum Eligible Unaborbed
Income paid (a) of tax in eligible foreign foreign tax
Nepal tax credit (b) credit
minimum of
(a) or (b)
USA 200,000 65,000 37,000 37,000 28,000
UK 300,000 -
55,500 - -
Canada 500,000 80000 18.5% 92,500 80,000 -
5. Write short notes: (4×2.5=10)
a) Deductible expenses from cash payment
b) Interest
c) Amended assessment
d) Permanent Account Number

The Institute of Chartered Accountants of Nepal 119


Suggested Answer - June 2018

Answers:
a) According to Section 21, of Income Tax Act, 2058 any expenditure paid through cash
for more than Rs 50,000 by a taxpayer having annual turnover for more than Rs. 20
lakh cannot be deductible. However, the following payments are deductible even
though, they exceed the cash payment limit of Rs. 50,000.
 Payment to GON, Constitutional bodies, public enterprises and BFI,
 Payment to producer of primary agricultural produces even when the products are
primarily processed,
 Payment of retirement contribution or retirement payment
 Payment made on the day of closure of banking service or on circumstances to be
paid only in cash, or
 Payment made in such area where banking facility is not available
 Amount deposited into the bank account of the receiver.
b) "Interest" means the following payment or profit:
(1) Payment under debt liability except the principal,
(2) Profit made from concession, premium, alteration payment or from similar
payment, and
(3) The amounts referred to in Section 32 receivable as an interest out of the payment
to be made by a person who acquires any property under annuities or installment
sale or of the payment made to any person for the use of any property under a
financial lease.

c) As per Section 101 of Income Tax Act, 2058, Inland Revenue Officer (IRO) can
make an amended assessment of any return filed by any tax payers solely on the
ground that the IRO deemed fit to do such amended assessment based on the IRO‘s
best judgement, and should be done in the manner that is consistent with the intention
of the Act.

In case IRD thinks it proper to do so, the assessment may be done as many times as
the IRD thinks appropriate but within 4 years of:
 In the case of an assessment under section 99, the due date of filling of the return,
or
 In the case of jeopardy assessment, the date on which the notice of assessment is
served to the tax payer under sec.102.
The maximum time limit of 4 years is not applicable in case of fraudulent assessment
identified by IRD, when the assessment shall be completed within one year of the
information of fraud.
Amended assessment is not allowed within 4 years time limit as well when the
matters in relation to the assessment are finalized by revenue tribunal or competent
court unless such courts or tribunal permits the re-assessment.

The Institute of Chartered Accountants of Nepal 120


CAP II Paper 7: Income tax & VAT

Before finalizing the amended assessment, a show cause notice shall be sent to tax
payer for defense by providing a time limit of 15 days.

d) As per Section 78, the following provisions are stipulated about PAN.
a. Subject to this Act, the Department shall issue a person with a number to be
known as a Permanent Account Number for the purposes of identifying the
person.
b. The Department may require a person to show their Permanent Account Number
in any return, statement, or other document used for the purposes of this Act.
c. GON may prescribe situations in which a person is required to show or quote their
Permanent Account Number.
d. Notwithstanding anything contained in subsection 1, person mentioned in
subsection 3 must receive PAN before commencement of business transaction.
e. A person cannot relieve from tax liability by reason of not having PAN.
6. Nepal Surgical House (Pvt) Limited is a principal stockiest for Nepal of Medtronic Inc,
Hong Kong (MIH). The company has policy to sell the items to other distributors with
15% markup at landed cost. Medical and surgical equipment manufactured by MIH are
subject to both VAT attractive and VAT exempt. MIH makes the shipment by billing on
FOB Price and import is done through irrevocable letter of credit. The Company provides
following information for the month of Chaitra 2074:
a) All items are subject to 5% custom duty under HS code 84.
b) Shipment against the LC established in Falgun 2074 having FOB value USD
126,300.00 of which USD 43,500.00 is subject to VAT was released during the
month and entire stock was sold on the same month.
c) Of the total import of taxable goods during the month, items having FOB Price USD
26,400.00 was revalued to USD 33,100.00 by the custom authorities.
d) Terms of Payment 20% during the establishment of LC and 80% on release of
documents.
e) Effective exchange rate per USD was 102.6, 103.8 & 103.45 during the date of LC
establishment, document release and custom clearance respectively.
f) Previous non-VAT stock having landed cost of Rs. 2,526,200.00 was also sold during
the month.
g) The company paid Rs. 146,800.00 inclusive of tax for Birgunj Kathmandu
transportation and insurance of the consignment.
On the basis of above information, prepare VAT statement of Nepal Surgical House (Pvt.)
Limited for the month of Chaitra 2074 considering opening VAT credit of Rs. 73,612.00.
Answers:
Amount (Rs.) Value Added Tax
Particular (Rs.)
Taxable Local Sales 5,491,073.90 713,839.61
Exempt Local Sales 12,978,175.14 -
Taxable Export - -
Exempt Export - -

The Institute of Chartered Accountants of Nepal 121


Suggested Answer - June 2018

Total 18,469,249.04 713,839.61


VAT on Import 708,870.44
VAT on Transportation Cost (Birgunj to Ktm)
(44,743.87*13%) 5,816.70
VAT Dr. for the month 4,969.17
Opening VAT Credit 73,612
Credit balance carried forward for Baisakh
2075 (74,459.53)

Note 1: Calculations of the Sales during the month

Particular Total Taxable Exempt

Previous No VAT stock sold 2,526,200 2,526,200


Sale of the Import Stock During the
month 15,943,049.04 5,491,073.90 10,451,975.14

Total Sales During the Month 18,469,249.04 5,491,073.90 12,978,175.14

VAT Collected on Sales 713,839.61 713,839.61 -

Note 2: Calculations dealers‘ price of the import

Particular Total Taxable Exempt

FOB Price in USD 126,300 43,500 82,800


Equivalent NPR
20% of FOB paid during LC
establishment and effective
exchange rate of NPR 102.6 per
USD 2,591,676 892,620 1,699,056
80% of FOB paid during LC
establishment and effective
exchange rate of NPR 103.8 per
USD 10,487,952 3,612,240 6,875,712

Total FOB Price in NPR 13,079,628 4,504,860 8,574,768

Custom Duty @ 5% 653,981.40 225,243 428,738.40

Transportation & Insurance in Nepal 129,911.50 44,743.87 85,167.64

Landed Cost 13,863,520.90 4,774,846.87 9,088,674.04

15% Markup 2,079,528.14 716,227.03 1,363,301.11

The Institute of Chartered Accountants of Nepal 122


CAP II Paper 7: Income tax & VAT

Sales Price to the distributors 15,943,049.04 5,491,073.90 10,451,975.14

Note 3: Calculations VAT paid on import

Particular Total Taxable Exempt

FOB Price in USD 126,300 43,500 82,800


Additional Revaluation by custom
authorities 6,700 6,700
Revised FOB Price in USD
133,000 50,200 82,800
Applicable Exchange Rate
103.45 103.45 103.45

Total FOB Price in NPR 13,758,850 5,193,190 8,565,660

Custom Duty @ 5% 687,942.50 259,659.50 428,283

Taxable Value 14,446,792.50 5,452,849.50 8,993,943

Value Added Tax Paid in Purchase 708,870.44 708,870.44 -

7.
a) Sussex (Pvt) Ltd, Kathmandu imports goods and directly supplies to
the retailers. From the following information, compute the amount of
VAT on sales.
Total cost of goods declared by importer Rs. 350,000.00
Valuation for cost of goods by Custom Officer Rs. 400,000.00
Freight & Insurance up to custom point Rs. 17,000.00
Freight from custom point to warehouse Rs. 20,000.00
Freight from warehouse to the retailers Rs. 8,000.00
Import Duty @ 10 %, excise duty @ 5% applicable only on import.
Gross profit 10 %. 5
b) Morang Distributers (Pvt.) Ltd is a sole purchaser of VAT attractive
goods from the manufacturer. Per unit cost price of the goods is Rs.
40,000.00. The company distributes the goods to the wholesaler,
wholesaler sells the goods to the retailer. The company and the
wholesaler, each incurs transportation expenses Rs. 500.00 for the
goods and they charge 10 % profit margin to determine the selling
price. The wholesaler provides 5 % discount on selling price to the
retailer. The retailer sells the goods on the same selling price fixed by
the wholesaler. The goods are taxable as per VAT Act, 2052.
Required: 5
i) Actual cost with VAT to the final consumer.

The Institute of Chartered Accountants of Nepal 123


Suggested Answer - June 2018

ii) Total VAT payable to the government.


Ignore the VAT payable by the manufacturer.
Answers:
a)

Taxable value in case of local sales is the cash consideration received or


receivable by the supplier at the time of supply of goods after deducting trade
discounts or trade rebates, if any [Sec. 12 (1) & (2)]
Calculation of sales price and VAT amount
Total cost with duties (As per custom valuation, WN) Rs. 4,81,635.00
Less: Over valuation by custom officer Rs. 50,000.00
Actual Cost at Custom Point Rs.4,31,635.00
Other direct cost
Freight from custom point to warehouse Rs. 20,000.00
Cost of the goods s Rs. 4,51,635.00
Gross profit (4,51,635/.9-4,51,635) Rs. 50,181.67
Sales price Rs. 5,01,816.67
VAT on sales price Rs. 65,236.16
Working note
Calculation of custom valuation and duty of the imported goods
Customs valuation Rs. 4,00,000.00
Freight & Insurance up to custom point Rs. 17,000.00
Transaction value Rs.4,17,000.00
Custom Duty @ 10% Rs. 41,700.00
Value for excise duty Rs. 4,58,700.00
Excise duty @ 5% Rs. 22,935.00
Total cost with duties Rs. 4,81,635.00
b)
i. Actual cost with VAT to final consumer =Selling price + VAT (WN 3)
Rs. 50,556+6,572.28
= Rs.57,128.28
ii. Total VAT payable to government=
Payable by the company (WN1) = Rs. 650
Payable by Wholesalers (WN2) = Rs. 394
Payable by Retailers (WN 3) = Rs. 328.61
Rs. 1,372.61
Working notes:
WN 1: The company's selling price and VAT payable:
Particulars price Rs. VAT Rs.
Cost price 40,000 5,200 input
Transportation cost 500
Actual cost 40,500
Profit Margin 10%
So, selling price 40,500/.90= 45,000 5,850 output
Net Profit and VAT payable 4,500 650

The Institute of Chartered Accountants of Nepal 124


CAP II Paper 7: Income tax & VAT

WN 2: Wholesalers' selling price and VAT payable:


Particulars Price Rs. VAT Rs.
Cost price 45,000 5,850 input
Transportation cost 500
Actual cost 45,500
Profit Margin 10%
Fixed selling price 45,500/.90= 50,556
Less: 5 % discount on Rs. 50,556 2,527.80
Net selling price 48,028.20 6,244 Output
Net Profit and VAT payable 2,528.20 394
WN 3: Retailers to consumers
Fixed selling price 45,500/.90= 50,556 6,572.28 Output
Cost price (Net selling price of W.S) 48,028.20 6,244 Input
Net Profit and VAT payable 2,527.80 328.61
8. Write short notes on the basis of VAT Act, 2052: (4×2.5=10)
a) Timing of goods supplied
b) Person
c) Proportionate credit
d) VAT refund to diplomats
Answers:
a) In case of supply of goods, earliest of following shall be time of supply under
sec. 6(2):
 Date of issuance of invoice, or
 Date of possession of goods/ date of removal of goods by the recipients
from suppliers business place, or
 Date of receiving consideration by the supplier.
b) Person" means any Individual, firm, company, association, institution,
partnership firm, co-operative, joint venture, trust or fund and this term also
includes the government entity, religious organization, charitable institution or
other similar association and their branch or sub-branch thereof engaged with
or without the objective of profit, in Taxable transaction.
c) The following provisions are mentioned in sub-rules (3), (4) and (5) under
rules 40 of VAT rules, 2053.
(3) In case any taxpayer has carried on both the transactions of taxable goods
or services and tax-exempt goods or services such taxpayer may deduct only
the tax paid on purchases or imports directly related to the taxable goods or
services.
(4) In case a taxpayer carrying out the transactions of both taxable and tax
exempt goods or services fails to establish the direct relationship of the
purchased or imported goods with the taxable goods or services, such taxpayer
may deduct the amount of tax paid on his purchases or imports by calculating
the proportion of taxable transaction value out of his total sale value.
(5) While calculating tax pursuant to sub-rule (4), if the tax officer feels that it
cannot be calculated proportionally, he may seek direction from the
Department to calculate it through another alternative method.

The Institute of Chartered Accountants of Nepal 125


Suggested Answer - June 2018

d) According to Section 25(1) (ka) of the VAT Act, the tax amount paid within
Nepal by a diplomat recognized by the Ministry of Foreign Affairs of GON on
the basis of mutual recognition by a foreign nation shall be refunded to the
extent of the amount consumed in Nepal. The tax refund application should be
submitted within 3 years from the date of transaction. The VAT credit is not
provided in any purchase less than Rs. 5,000 at a time.

The Institute of Chartered Accountants of Nepal 126


CAP II Paper 7: Income tax & VAT

Specific Comments on the performance of the students


Batch: - June 2018
Level: - CAP-II
Subject: Income Tax & VAT
Question No. 1
Mostly students solved the question but only few students answered by segregating
expert for local business. Most of the students failed to compute income differently or
export and domestic sales.
Question No. 2
Mostly student unable to which items should be included and which items should be
excluded as income from investment. Most students treat interest income as taxable
although it is FWT.
Question No. 3
Most students treat all the retirement payment as taxable income and does not
segregate the R.B before Chaitra 18, 2058
Question No. 4
Students answered the questions who have concept of PI, Aug tax rate, joint
investment but only few have knowledge about what constitute related party and
implication of transaction between related parties
Question No. 5
Students have solved who had studied the IT Act.
Question No. 6
Not a single student answers this question accurately. Probably, due to custom Act
provision, students confused how to solve it.
Question No. 7
Although this question is basically arithmetical based, students are unable to solve it
properly.
Question No. 8
Students answer the question who had studied the VAT Act.

The Institute of Chartered Accountants of Nepal 127


Suggested Answer - June 2018

The End

The Institute of Chartered Accountants of Nepal 128

You might also like